MBE KAPLAN--CONLAW Flashcards Preview

Texas Bar Exam > MBE KAPLAN--CONLAW > Flashcards

Flashcards in MBE KAPLAN--CONLAW Deck (175)
Loading flashcards...
1
Q
  1. During the final days of his presidency, many of the President’s Cabinet members began leaking rather embarrassing stories about the President to the press. In one such story, which appeared in a newspaper, an unidentified Cabinet member was quoted as saying that the President pardoned the owner of a professional baseball team, for a previous felony conviction because the owner promised to give the President a lifetime pass to all future games. Upset by these news “leaks,” the President, in his last official act, issued an executive order banning all executive employees from having conversations with members of the press unless prior permission had been obtained from an administrative supervisor. The executive order called for the dismissal of any employee willfully in violation of that order.

After the President left office, the press made repeated references to the executive order and derisively referred to it as the President’s legacy. Nonetheless, after the new President took office he promised to abide by the order. Thereafter one of the Vice President’s senior advisers, leaked a story to a magazine stating that the Vice President told staff members that the only reason he visited a city following an earthquake was because he wanted to play golf at a nearby golf course. The magazine printed the story and named the senior adviser as the source. In compliance with civil service regulations, a hearing was conducted and the senior adviser was found to have violated the executive order barring unauthorized conununications with print media. Subsequently, the senior adviser was summarily dismissed from his executive staff position.

The senior adviser brings an appropriate action seeking reinstatement and challenges the constitutionality of the executive order. If this case reaches the U.S. Supreme Court, the justices should rule the executive order is

(A) constitutional, because governmental employment is a privilege and not a right.

(B) constitutional, because the President has plenary power to set forth employment regulations for executive department employees.

(C) unconstitutional, because Congress, not the President, has authority to regulate the terms and conditions of federal employment.

(D) unconstitutional, because the President cannot broadly limit all executive employees’ freedom of speech and association.

A
  1. (D) Choice (A) is incorrect. As a general rule, the President may terminate, without Congress’s approval, those appointees, like his advisers. However, the President may not violate the federal Constitution in doing so. In this instance, the President’s order requiring his appointees to obtain prior permission to speak probably violates the FirstAmendment. Choice (B) is incorrect. The President may not draft regulations that violate the constitutional rights of the employees. After all, the President is no mere employer; he or she is a state actor, and the Constitution protects people, including governmental employees, from constitutional violations by state actors in their official capacity. Choice (C) is too narrow. The President has the authority to regulate the terms and conditions of federal employment for certain employees, especially those, like his advisers who are directly and solely responsible to the President. While the President has the power both to issue executive orders and to remove purely executive officers appointed by him without being subject to any Congressional limitation, the scope of these powers must not contravene federal constitutional limitations, such as the First Amendment rights of freedom of speech and association. The executive order banning all executive employees from having conversations with members of the press unless prior permission has been obtained from an administrative supervisor probably violates the First Amendment as an overbroad prior restraint limiting otherwise protected speech. The executive order is, therefore, unconstitutional. Therefore, choice (D) is the correct answer.
2
Q
  1. A devastating earthquake struck a foreign country. The earthquake destroyed hundreds of homes and buildings. Thousands of residents were killed, injured, and left homeless by the earthquake. The leader of the foreign country sought the assistance of the United States in providing emergency aid.

The President initially wanted to send troops from a west coast military base but learned that they were out on training maneuvers. As a result, the President issued an executive order directing members of a federal agency under the supervision of the Immigration and Naturalization Service, to travel to the foreign country to engage in emergency rescue operations. This federal agency’s main function is to patrol the borders to prevent illegal aliens from entering the United States.

A member of this federal agency refused to comply with the executive order and assist in the relief effort. He claimed that participating in such operations in a foreign country is beyond the scope of his job responsibilities. He brought suit in federal court challenging the validity of the President’s executive order. Assuming the member of the federal agency has adequate standing, the court should rule the executive order

(A) valid, because as commander-in-chief, the President has unfettered authority to require federal employees to participate in an international relief effort.

(B) valid, because employees of an executive agency can be required to perform such emergency functions.

(C) invalid, because the executive order is beyond the scope of presidential power absent congressional authorization.

(D) invalid, because the director of the INS, not the President, has the power to issue directives to federal agencies under its supervision.

A
  1. (B) The Constitution gives the President broad authority with respect to foreign affairs. Article II, Section 2 explicitly enumerates a number of powers given him in this area (e.g., commander-in-chief power, treaty-making power, and the right to appoint am bassadors). However, the President’s power over foreign affairs is not plenary or absolute. Rather, the President and Congress have concurrent powers over foreign affairs. To be sure, Congress is given war power authority and treaties must be ratified by two-thirds of the Senate. In this regard, choice (A) is not the best answer for two reasons. First, it is doubtful that the President would have authority as commander- in-chief to mobilize nonmilitary federal employees in this manner. Second, the President’s authority to direct foreign affairs is not “unfettered” or absolute. Choice (B) would be a better answer, since the President as Chief Executive would probably have authority to direct executive employees to engage in such an international relief effort. Choice (C) is incorrect. This is a little too strong. To avoid violating the principle of the separation of powers, it is wise for the President to seek congressional approval for an executive order. However, it is not necessary in all instances that the President seek approval from Congress before issuing his executive order. This is especially true for issues that are related to foreign affairs. Choice (D) is a bit too narrow. The INS is encompassed under the Department of Homeland Security, which, in turn, is housed under the Executive Branch.
3
Q
  1. Congress enacted a statute wherein the government would purchase and operate all of the nation’s airlines.

In all likelihood, Congress’s power to enact this legislation will derive from

(A) its power to tax and provide for the general welfare.

(B) its power to raise and support an army and declare war.

(C) its power to regulate commerce.

(D) its power to make laws regarding territory and other property belonging to the United States.

A
  1. (C) Since Congress has the power to regulate interstate commerce under Article I, Section 8, choice (C) is the best answer. Choice (B) is incorrect because the facts do not state that the United States is at war. Similarly, you may not assume that the airline takeover was a wartime or war-related act. Choice (A) is incorrect. In order to purchase aLL of these airlines, it’s possible that Congress will have to levy higher taxes on Americans. But the question asks under what power Congress may purchase the airlines, not how and if it will pay for them. And Congress can use its commerce clause powers to purchase the airlines. Finally, choice (D) is incorrect. The airlines didn’t belong to the United States at the time of purchase.
4
Q
  1. Congress enacted a statute designed to make the stabilization of rents more effective throughout the United States. The statute will be applicable to all leasehold contracts hereafter entered into between landlords and tenants involving residential and commercial properties. The new federal statute is intended to protect tenants from spiraling rents and “profiteering” by landlords without curtailing the supply of rental units or depriving landlords of substantial constitutional rights.
    The new federal statute would be most clearly constitutional as applied to

(A) the rental of a state-owned office building by a state to a privately owned business.

(B) the rental of residential property located in the District of Columbia by an individual landlord to a family of six.

(C) the rental of an apartment located by a real estate company to an individual tenant.

(D) the rental of an office building to a city by an individual landlord.

A
  1. (B) ArticLe IV, Section 3, Clause 2 gives Congress power “to dispose of and make all needfuL ruLes and regulations respecting” lands of the United States. Since the District of Columbia is under federal jurisdiction, choice (B) is the best answer. Choice (A) is incorrect. ArticLe IV, Section 3, Clause 2 gives Congress power “to dispose of and make all needful rules and regulations respecting” Lands of the United States. For purposes of this clause, a state does not fall within Congress’s federal jurisdiction. Choice (C) is incorrect. Article IV, Section 3, CLause 2 gives Congress power “to dispose of and make all needful rules and regulations respecting” Lands of the United States. For purposes of this clause, a state does not faLL within Congress’s federaL jurisdiction. Choice (D) is not the best choice. Article IV, Section 3, Clause 2 gives Congress power “to dispose of and make all needful rules and regulations respecting” Lands of the United States. For purposes of this clause, a city does not faLl within Congress’s federal jurisdiction.
5
Q
  1. Congress has recently enacted a statute designed to prevent price gouging of gasoline by private retail sellers. To help administer the new statute Congress created a seven-member agency, four members of which were appointed by the President, two by the Senate, and one by the Speaker of the House. The agency was authorized to issue rules interpreting regulations prescribed under existing federal law. It was also authorized to issue “cease and desist” orders after hearings against private retail sellers of gasoline who “intentionally engaged in price gouging.” In this regard, violations of the cease and desist orders were made punishable by fine and/or imprisonment.

Which of the following is the strongest constitutional argument against the authority of this agency?

(A) Congress may not delegate to the agency power to make interpretations of existing federal laws.

(B) Congress may not delegate to the agency power to make “cease and desist” orders, the violation of which is punishable by imprisonment.

(C) Congress may not delegate enforcement of a federal law in any governmental body other than the U.S. Attorney General or the courts.

(D) Congress may not retain the power to appoint even a minority of members of such an agency.

A
  1. (D) Article II, Section 2 specifies that the President shall appoint “with the advice and consent of the Senate,” all “Ambassadors, other public Ministers and Consuls, judges of the Supreme Court, and all other Officers of the United States.” Note that although Congress may appoint officials to exercise such investigative power as it might delegate to one of its own committees, it maynotappoint members oran agency or commission with administrative powers. Such persons are deemed “officers of the United States” and must be appointed pursuant to Article II, Section 2 [Buckleyv. Valeo, 424 U.S. 1 (1976)]. Choice (A) is incorrect. In the fact situation, Congress authorized the agency to do more than interpret the laws. Congress authorized it to punish violators. But such authorization goes beyond what Congress is permitted to do. Under Article II, Section 2, the President shall appoint “with the advice and consent of the Senate,” all “Ambassadors, other public Ministers and ConsuLs, judges of the Supreme Court, and all other Officers of the United States.” In other words, only the President may appoint those with administrative powers to punish law violators. Choice (B) is a good choice, but it’s not the best choice. The problem is larger than one of “cease and desist” orders. For Congress lacks the authority to appoint members to an agency that has any administrative powers, including any powers to punish law violators. Congress would do no better if it had substituted the right to issue cease and desist orders with some other form of punishment. Only the President has authority to appoint members who may exercise administrative power [Buckleyv. Valeo, 424 U.S. 1 (1976)], construing Article II, Section 2. Finally, choice (C) is incorrect. The Attorney General certainly has enforcement powers, given that he is an officer of the Executive Branch. So, too, the courts may issue cease and desist orders. Congress also has the authority under Article I to create agencies that may enforce the law. Congress may not, however, appoint members to these agencies; only the President may do so [Buckleyv. Valeo, 424 U.S. 1 (1976)], construing Article II, Section 2.
6
Q
  1. The navy wanted to build a naval base on the north shore of an island in Hawaii. Situated along the north shore of this island were coral reefs which are the home of a very rare species of fish that are found only along the north shore area.
    Congress conducted hearings to decide whether to authorize the construction of the naval base. During the hearings, one of the speakers who addressed the congressional committee was a famous expert on oceanography and marine biology. The expert vehemently opposed the naval plan and stated that the construction would, in his opinion, result in the extinction of the rare species of fish. Congress thereafter approved the construction of the naval base and passed a bill providing necessary authorization and funding for the project.

The expert has filed an action in federal district court seeking to enjoin the construction of the naval base on ecological grounds. Does the expert have adequate standing?

(A) Yes, because he has a personal stake in the litigation.

(B) Yes, because he is a recognized expert on marine biology and he testified at the congressional hearings.

(C) No, because the suit presents a nonjusticiable political question.

(D) No, because he is not suffering any actual harm or injury.

A
  1. (D) The Supreme Court has said that the core of Article Ill’s requirement for cases and controversies is found in the rule that standing is limited to those who allege that they personally have suffered or imminently will suffer an injury. In City of Los Angeles v. Lyons, 461 U.S. 95 (1983), the Court stated that “the plaintiff must show that he has sustained or is immediately in danger of sustaining some direct iniury as a result of the challenged action.” With respect to standing in environmental cases, the Supreme Court has handed down two conflicting rulings. In Sierra Club v. Morton, 405 U.S. 727 (1972), the Sierra Club sought to prevent construction of a ski resort in Mineral Valley in California. The Court ruled that the Sierra Club lacked standing because there was no allegation that any of the Sierra Club’s members ever had used Mineral Valley. By contrast, in United States v. Students Challenging RegulatoryAgency Procedures, 412 U.S. 669 (1973), the Court upheld the standing of a group of George Washington law students alleging that an administrative rate hike would decrease recycling, which would lead to more use of natural resources and increased pollution. The students maintained that their enjoyment of the forests, streams, and mountains in the Washington, D.C., area would be lessened as a result. The Court upheld the group’s standing because the plaintiffs actually used the environmental area for hikes, biking, and other recreational purposes. In this hypo, there is no showing that the expert is making any personal use of the coral reef area. Therefore, he would lack standing on environmental grounds. Choice (A) is incorrect. The Supreme Court has said that the core of Article Ill’s requirement for cases and controversies is found in the rule that standing is limited to those who allege that they personally have suffered or imminently will suffer an injury [City of LosAngeles v. Lyons, 461 U.S. 95 (1983)]. Professor has not suffered nor will suffer some personal injury. His grievance can be shared by anyone who is interested in preventing damage to the environment. Answer (B) is incorrect. This would seem an intelligent answer, but the expert still lacks standing because standing is limited to those who allege that they personally have suffered or imminently will suffer on injury [City of Los Angeles v. Lyons, 461 U.S. 95 (1983)]. The expert’s expertise cannot compensate for his lack of personal injury. Choice (C) is not correct. The political question doctrine would not permit the court to consider the case because of one or more of the following reasons. One, if the U.S. has appropriated the decision- making on this issue to another branch of the federal government. For example, the judiciary cannot hear cases interpreting the rules for impeachment; only the Senate may do so [U.S. v. Nixon, 418 U.S. 683 (1974)]. Two, if the court feels that, for practical reasons, it should not hear the case. For example, the Supreme Court felt it impractical to supervise electoral districts to prevent partisan gerrymandering. [ Vieth vjubelirer, 541 U.S. 267 (2004)].The extinction of the rare species of fish may present serious environmental problems but does not present issues that are related to the political question doctrine.
7
Q
  1. There are 2,000 insurance agents licensed under the general licensing laws of a state. The state legislature recently passed a highly controversial bill that, inter alia, reduces “good driver” automobile insurance rates and prohibits price-fixing and discrimination by insurance companies. This bill passed despite a well-financed and intense lobbying effort mounted by the insurance industry. After this law was enacted, an insurance agent who is a resident of the state, was interviewed by the local newspaper and publicly stated, “the legislature is a bunch of self-serving crooks.” After the statement made newspaper headlines, the state legislature enacted a statute providing that “the state insurance license of the insurance agent is henceforth revoked.” Astonished by the legislature’s action, the insurance agent, who remains unrepentant, seeks your legal advice.

You should advise the insurance agent that the state statute revoking her state insurance license is

(A) constitutional, because a state license is a privilege and not a right and, therefore, it is not protected by the due process clause.

(B) unconstitutional, because it denies her the equal protection of the laws.

(C) unconstitutional, because it is a prohibited bill of attainder.

(D) unconstitutional, because it is a denial of a privilege or immunity protected by Article IV.

A
  1. (C) A bill of attainder is any form of legislative punishment of a named group or individual without judicial trial. Under Article I, Sections 9 and 10, federal and state governments are prohibited from enacting bills of attainder. The statute revoking the state insurance license of the insurance agent is a sufficient form of legislative “punishment” to constitute an impermissible bill of attainder. Choice (A) is incorrect. A state-issued license can be a right protected under the due process clause. Here, the insurance agent did not receive fair notice and a hearing, normal requirements of due process. Choice (B) is incorrect. The equal protection clause provides heightened protection for groups whose legal classifications are suspect (e.g., classifications based on race, ethnicity, nationality, and religion) and for groups whose legal classifications are quasi suspect (e.g., gender, illegitimacy). The insurance agent may have been the subject of discrimination by the legislature but not because she is a member of a suspect or quasi-suspect class. Answer (D) is not the best choice. The privileges and immunities clause of Article IV would prevent the state legislature from discriminating against those who are out-of-staters. Even if the legislature discriminated against the insurance agent by revoking her license, it did not discriminate against her because she is an out-of-stater.
8
Q
  1. Evidence at Congressional hearings has indicated that thousands of elderly citizens die each year because they cannot afford proper medical and hospital treatment. Based on the evidence presented at these hearings, a senator wishes to introduce a bill providing free medical and hospital care for all citizens 70 years of age and older. The senator, however, intends to include a provision in the bill denying such medical and hospital care benefits to aliens (in the same age category) unless they have been legally admitted for permanent U.S. residency. The senator has asked for your advice regarding his proposed bill. You are understandably concerned about the legality of the aliens proviso.

Which of the following is the strongest argument in support of the constitutionality of the provision disqualifying aliens from receiving such medical and hospital benefits?

(A) Due to the fact that the medical and hospital benefits will be paid for by the government, they are a privilege not a right and, therefore, are not within the meaning of the Fifth Amendment.

(B) The disqualifying provision does not unduly burden either interstate commerce or the right of aliens to travel freely from state to state.

(C) The principles of equal protection apply against the states and not against the federal government.

(D) The disqualifying provision is reasonably related to legitimate congressional objectives under its immigration, citizenship, and spending powers.

A
  1. (D) In light of the decision in Mathews v. Diaz, 426 u.s. 67 (1976), choice (D) is the best answer. In Mathews, the Court applied “relaxed scrutiny” in upholding the validity of a federal statute that conditioned an alien’s eligibility for participation in a federal medical insurance program on the satisfaction of a durational residency requirement, but imposed no similar burden on citizens. The Court was at pains to emphasize that Congress, as an aspect of its broad power over immigration and naturalization, enjoys rights to distinguish among aliens that are not shared by the states. Although (C) is a correct statement of law, it does not provide any rationale supporting or justifying the federal government’s treatment of aliens. Choice (A) is incorrect. That the government is paying for something does not necessarily turn something from a right (which is presumably protected) to a privilege (which presumably isn’t). We still have “state action” and thus the government is bound to respect the Constitution even as it doles out supposed privileges. Choice (B) is incorrect. This seems to be a plausible answer because the dormant commerce clause only governs states and local governments; here, we have Congress. But the fact that Congress does not violate the dormant commerce clause is not the best answer because the call of the question focuses on aliens and, hence, on issues of equal protection. Choice (C) is incorrect. It’s true that the 5th Amendment’s Due Process Clause does not contain an equal protection clause. But the Supreme Court has read into it a doctrine of equal protection that is analogous to the 14th Amendment’s Equal Protection Clause [Boiling v. Sharpe, 347 u.s. 497 (1954)].
9
Q
  1. A city has undergone a massive redevelopment project aimed at remodeling and beautifying the downtown area. Recently the city council passed an ordinance prohibiting the placement of any sign with dimensions larger than six feet on the exterior of any commercial building. Furthermore, signs within the guidelines of the ordinance could only relate to advertising the business of the property’s occupant. The intended purpose of the ordinance was to advance the municipality’s interests in traffic safety and aesthetics.
    An owner of an office building in the newly developed section of town, placed a 10-foot sign on the outside of his building endorsing his brother’s political candidacy. The city council has ordered the owner to remove the sign from his building, but he has refused to take it down until after the election.
    If the owner challenges the city ordinance, the most likely result is that he will
    (A) prevail, because the ordinance violates his freedom of speech.
    (B) prevail, because such a time, place, and manner restriction on private property is discriminatory and overbroad.
    (C) not prevail, because although commercial speech is protected by the First Amendment, it is subject to greater regulation than other forms of protected speech.
    (D) not prevail, because the ordinance is rationally related to a legitimate state interest.
A
  1. (B) In Metromedia, Inc. v. San Diego, 453 u.s. 490 (1981), the Court struck down a San Diego ordinance which, inter alia, prohibited all billboards containing non-corn mercial messages, except for those messages falling within certain defined categories (e.g., temporary politicaL campaign signs, signs carrying news items or telling the time or temperature, etc.). The Court objected to this handling of non-commercial speech on the grounds that “the city may not choose the appropriate subjects for public discourse”; the city was required to either allow all, or no, non-commercial messages. Here, the Middletown ordinance would have been valid if it were limited to restricting the size of billboards. But since the ordinance also attempted to impose non-content-neutral regulations (by requiring that signs relate only to advertising the business of the owner-occupant), it probably will be struck down under the strict (and often fatal) scrutiny test. Choice (A) is not the best choice. This is an alluring answer, but it’s not the best choice. We want to see what, if anything, is defective with the statute that regulates the speech. So Choice (B) is a stronger choice. Choice (C) is incorrect. It’s true that commercial speech is subject to greater regulation than, say, political speech. But the ordinance in the example indulges in content regulation. So the ordinance will be subject to strict scrutiny and most likely will fail such scrutiny. Choice (D) is incorrect. Strict scrutiny review applies to speech—except for unprotected speech, which is governed by the rational basis test, and time, place, and manner restrictions.
10
Q
  1. For many years, persons engaged in real estate transactions in a state have utilized installment land contracts. The so-called installment land contract has been popular as a substitute for the mortgage or deed of trust. Under such a contract, the seller agrees to accept, and the buyer agrees to pay, the purchase price in installments over a stipulated period of time. The vendor retains legal title as security for the payment of the purchase price; and the vendee has equitable title under the doctrine of equitable conversion. The most important characteristic of the installment land contract, however, is the forfeiture clause, which provides that if the vendee is in default for 30 days, the vendor may, at his option, declare a forfeiture of the contract, in which case all payments made shall be forfeited as “liquidated” damages and the buyer shall become a tenant at will of the seller.
    Over the years, many sellers of property under such installment land contracts have declared forfeitures in situations where the prospective buyers were delinquent in their payments, even when the buyer was late with a single payment after nearly all the other payments had been made. In order to remedy this inequitable situation, the state legislature enacted a statute requiring any seller attempting to declare a forfeiture of an installment land contract to do so by instituting a formal foreclosure action in the courts. The new law also provided that prior to the commencement of such a foreclosure action, the seller under such an arrangement must give the buyer a 60-day grace period to make up any delinquent installment payment with interest. The new law expressly applied both to installment land contracts entered into subsequent to its enactment and to installment land contracts already in existence at the time of its enactment.
    Is this new state statute likely to be held constitutional?
    (A) Yes, because it is a reasonable regulation of the procedure to be followed in such cases and does not substantially diminish the underlying obligations of the buyer.
    (B) Yes, because the authority to enact laws regulating real estate sales transactions occurring within the boundaries of individual states is reserved exclusively to the states by the Tenth Amendment.
    (C) No, because application of the law to installment land contracts entered into prior to its enactment is a violation of the obligation of contracts.
    (D) No, because application of the law to installment land contracts entered into before or after its enactment is a deprivation of a proprietary interest without due process of the law.
A
  1. (A) Although nowhere mentioned in the Constitution, the concept of police powers in the 10th Amendment has developed closely with that of the dormant commerce power to enable a state to regulate issues relating to health, safety, morals, and welfare, including economic welfare. As a general rule, “where the state regulates even-handedly to effectuate a legitimate local public interest, and its effects on interstate commerce are only incidental, it will be upheld unless the burden imposed on such commerce is clearly excessive in relation to the putative local benefits” [Pike v. Bruce Church, Inc. 397 U.s. 137 (1970)]. The state law requiring sellers under “installment land contracts” to institute foreclosure actions to relieve the harsh effect of forfeiture by allowing the buyer a 60-day reinstatement period serves a legitimate purpose and leaves unchanged the initial monetary obligations of both parties. Therefore, it will be upheld. Choice (A) is correct. In order to protect the health, safety, morals, and welfare of its citizens, a state may enact legislation that impairs contracts under certain conditions. The action of the Utah legislature may be viewed reasonably as a permissible police power modification of the “installment land contract” because no prior underlying contractual obligation has been impaired, since the buyer must still pay in full the original bargained-for purchase price. In general, private parties who enter into contracts may not stop the legislature from exercising its proper police power for the public good. Nowak, p. 376. Choice (B) is incorrect because the authority to enact laws regulating intrastate real estate sales transactions is not reserved exclusively to the states under the 10th Amendment. Such transactions have a cumulative impact on interstate commerce and are subject to the federal commerce power. The court has refused to use the 10th Amendment to reserve subject areas of commerce for state regulation. Choice (C) is also incorrect. Remember that there is no fundamental right to contract in the Constitution. The state’s regulation thus will be subject to rational review. Under rational review, the court probably will find that there is a legitimate government interest in the state’s desire to protect buyers against unfair contracts. Next, the court probably will find that there are rationally related means. In fact, for the right of contract, the Supreme Court has gone so far as to surmise potential justifications under rational review forthe state. [Williamson v. Lee Optical, 348 U.S. 483 (1955)]. Choice (D) is incorrect. Subsequent to the state’s legislation, the bank still retains its right to foreclose on the property. Accordingly, there is most likely no deprivation of property under the due process clause.
11
Q
  1. Congress enacted a statute aimed at “countries assisting or furnishing aid or support to nations or movements engaged in hostilities with the United States.” Section 5 of that act authorized and directed the Treasury Department “to issue orders barring entry into the United States any category of goods the sale of which in the United States is likely to improve the economic or balance of payments posture of an assisting country.” The Secretary of State was authorized by Section 6 to define “assisting countries.” Pursuant to Section 5, the Treasury Department issued a regulation, which provided in part that:
    “Imports of the following categories of goods from assisting countries are hereby prohibited:
    • . (c) Bulk shipments for resale within the United States of books, pamphlets, flags, decorations or other symbols, excepting, however, scientific, technical, and literary works intended for scholarly purpose…”
    The State Department designated a foreign country, as an “assisting country,” on the basis of its determination that medical supplies collected by public donation in this country had been sent to another country currently engaged in hostilities with the United States. As a consequence, the Treasury Department issued an order barring practically all products and goods from this foreign country into the United States.
    A distributor of state and foreign flags, has had a lucrative contract with the foreign country for the importation and sale of their flags in the United States. However, because of the Treasury Department’s order, the foreign country is now barred from importing any of its flags into the United States.
    In an appropriate federal court, the distributor brings a suit against the Secretary of State and the Treasury Department to set aside the order barring the foreign country’s imports on the grounds that it is inconsistent with the principles of our constitutional form of government. Which of the following is the most proper disposition of the distributor’s suit by the federal court?
    (A) Suit dismissed, because the distributor does not have standing to bring this action.
    (B) Suit dismissed, because there is no adversity between the distributor and the defendants.
    (C) Suit dismissed, because it presents a nonjusticiable political question.
    (D) Suit dismissed, because it presents a moot issue.
A
  1. (C) The doctrine of political questions precludes federal judicial review of nonjusticiable issues (i.e., matters not capable of judicial resolution due to inherent separation of powers limitations). The leading case of Baker v. Carr, 369 U.S. 186 (1962), established a test to identify such nonjusticiable issues. For example, where a textually demonstrable commitment to a coordinate political branch of government has been made, or where there is a lack of judicially manageable standards to resolve the issue, or where judicial resolution would create the potential for embarrassment from multifarious pronouncements by various branches of government, then the judiciary will decline resolution of such nonjusticiabie political questions. Choice (C) is correct because foreign affairs is a nonjusticiable political question over which federal courts will not exercise jurisdiction. Choice (A) is incorrect. Actually, the distributor does have standing insofar as it can show that: The distributor has suffered a direct harm from the governmental law; the governmental law is a “but for” cause of the distributor’s harm; and there is a “substantial likelihood” that the relief sought from the court by the distributor, if granted, would remedy the harm. Choice (B) is incorrect. In order to sue, the distributor would have to show that it has suffered a material harm under the adversity requirement. The facts tell us that the distributor has a “lucrative” contract with the foreign country. Hence, we may assume that the governmental law, by prohibiting this contract, would satisfy the adversity requirement. Choice (D) is incorrect. The case will not be dismissed for mootness because an actual controversy does exist, however the court will refrain from hearing that controversy because it is a nonjusticiable political question.
12
Q
  1. A state enacted a statute prohibiting any motor vehicle traveling within the state from having window tinting or glass coating of any kind. The bill passed the state legislature at the urging of state and local law enforcement agencies who argued that tinted windows prevented them from observing interior car activity. Most citizens also supported the bill, especially after a state trooper was killed by an occupant in a window-tinted limousine. The trooper was unable to see that his assailant was armed and dangerous when he approached the vehicle for a speeding infraction.
    A limousine company operates a limo service in a neighboring state and has a fleet of 68 limos, all of which have tinted windows. Each year, the company makes thousands of trips into the state to transport passengers to the international airport situated 10 miles from the border separating the two states. Since all of its limos are manufactured with tinted windows, the company will incur great expense in ordering limos without tinting.
    The company brings suit to challenge the constitutionality of the state statute. Assuming that the company has proper standing to assert such an action, which of the following is their strongest constitutional argument to invalidate the aforesaid statute?
    (A) Because window tinting is permitted on vehicles in neighboring states, this law denies the company the equal protection of laws.
    (B) Because this law burdens interstate commerce by prohibiting all vehicles with window tinting from entering the state, this law violates the commerce clause.
    (C) Because window tinting on vehicles is legal in their home state, this law violates the contract clause by preventing the company from fulfilling its obligation to transport passengers into a neighboring state.
    (D) Because interstate travel is a fundamental right that may not be burdened by state law, it violates the company’s substantive due process rights by arbitrarily and unreasonably regulating economic activity.
A
  1. (B) Under Article I, Section 8, Congress shall have the power “To regulate Commerce with foreign Nations, and among the several States, and with the Indian Tribes.” This plenary federal commerce power is held concurrently with the states, which may freely govern matters that do not require uniform national regulation. For a state regulation affecting interstate commerce to pass judicial scrutiny, the statute must be found reasonable and nondiscriminatoryupon balancing the benefit to the state against the burden imposed on interstate commerce such that no less-restrictive alternative means of regulation is available. Where a state statute imposes a discriminatory burden on out-of-state interests, its validity is seriously in question. The statute effectively prevents the company from doing business in its state. The company’s strongest constitutional argument to invalidate the statute would be the Commerce Clause challenge presented in choice (B). Choice (A) is incorrect because all motorvehicles in the state are treated equally under the law. Choice (D) is incorrect because strong support is given in the facts to demonstrate that the purpose for enacting the statute was neither arbitrary nor unreasonable. Finally, choice (C) is incorrect because the contracts clause of Article I, Section 10 states that no state shall impair the obligation of contracts. Here, no state has prevented the company from fulfilling its contractual obligations, although it may now have to pay more to do so.
13
Q
  1. The state registrar of elections refused to put a candidate’s name on the ballot because the registrar believed that the candidate was not a resident of the state. The registrar contended that Article I, Section 2 of the U.S. Constitution specifically required a candidate for the House of Representatives “be an Inhabitant of that State in which he shall be chosen.” As a consequence, the candidate filed suit in state court against the registrar of elections seeking to have her name placed on the ballot. The state court ruled in her favor and determined that the candidate did, in fact, qualify as a resident of the state. The registrar appealed the decision to the state supreme court which, in turn, affirmed the lower court ruling. Thereafter, the candidate’s name was placed on the ballot. After a hotly contested election, the candidate narrowly defeated the incumbent.
    However, at the time that the candidate was to be seated as a House member, the House of Representatives held hearings on her qualifications and eligibility. By a two-thirds vote, the House determined that the candidate was not a resident of the state at the time of her election and refused to seat her. The candidate then brought suit in federal district court against the Speaker and other officers of the House, seeking back pay and an order that she be seated in the House of Representatives. The defendants demurred, claiming lack of subject matter jurisdiction.
    Which of the following is the strongest constitutional argument supporting the defendants’ demurrer?
    (A) There is no case or controversy between the candidate and the officers of the House of Representatives.
    (B) The case presents a nonjusticiable political question.
    (C) The suit should have been brought as an original action in the U.S. Supreme Court.
    (D) Under Article III of the Constitution, the federal courts are not empowered to render advisory opinions.
A
  1. (B) This Constitutional Law question focuses on the specific case of Powell v. McCormack, 395 U.S. 486 (1969), but there is an important twist. In Powell, the House of Representatives refused to seat him (after he had been elected) due to the fact that he had wrongfully diverted House funds for his own personal use. In brief, the Court held that the House did not have authority to exclude him since he met all the requirements for membership expressly prescribed in Article I, Section 2 (namely, age, citizenship, and residency). Thus, the Court held that Powell’s claim did not present a political question. In this example, however the issue is whether Kennedy fulfilled the “residency” requirements set forth in Article I, Section 2. Since the House has the unreviewablepowerto determine the “residency” (as well as “age” and “citizenship”) qualifications for its members, the case presents a nonjusticiable political question. Choice (A) is incorrect. Because Powell had demanded back pay (as the candidate had done), the Court held that “a prayer for specific relief prevented a finding of mootness and demonstrated that there was, in fact, a ‘case or controversy.” See Powell, 395 U.S. at 497. Choice (C) is incorrect. Artide Ill, Section 2 states: “In all cases affecting ambassadors, other public ministers and consuls, and those in which a state shall be party, the Supreme Court shall have original jurisdiction. In all the other cases before mentioned, the Supreme Court shall have appellate jurisdiction, both as to law and fact, with such exceptions, and under such regulations as the Congress shall make.” The candidate’s lawsuit does involve governmental officials but not those who are covered by the Supreme Court’s original jurisdiction. Choice (D) is not the best choice. While this summation of Article Ill is correct, the federal court is not being invited to issue an advisory opinion. The court is being invited to adjudicate an actual controversy.
14
Q
  1. There has been a great deal of news media coverage regarding the problem of underage drinking. Many high school and college students have been purchasing fraudulent driver’s licenses with phony birthdates showing “proof’ of being 21 years of age or older. As a consequence, many teenagers are able to purchase alcoholic beverages at liquor stores, restaurants, and bars. The situation is becoming especially alarming on many college campuses. A magazine article reported that the majority of freshmen between the ages of 18 and 19 at many colleges had illegally purchased evidence of phony “proof.”
    With underage drinking reaching epidemic proportions, Congress enacted a statute establishing a federal commission to monitor and curtail alcoholic beverage sales to underage drinkers. To implement the program on a national scale, the bill required each state to pass legislation establishing a local “watchdog” agency to facilitate compliance with congressional intent.
    A state has filed suit challenging the constitutionality of the federal statute. The law is likely to be held
    (A) valid, because the sale of alcoholic beverages has a substantial impact on interstate commerce.
    (B) valid, because the establishment of a state watchdog agency under the auspices of a federal regulatory scheme is consistent with the provisions of the Twenty First Amendment.
    (C) invalid, because it violates the fundamental principles of state sovereignty embodied by the Eleventh Amendment.
    (D) invalid, because the federal government may not compel state legislatures to enact and enforce a federal regulatory program.
A
  1. (D) The landmark case of New York v. United States, 505 U.S. 144 (1992) clearly limited Congress’s right to interfere with a state’s ability to make and apply laws through legislative, judicial, and administrative functions. Based on principles of state sovereignty under the 10th Amendment, Congress may not simply commandeer the legislative processes of the States by directly compelling them to enact and enforce a federal regulatory program. The Court explained that allowing Congress to commandeer state governments would undermine government accountability because Congress could make a decision but the states would take the political heat and be held financially and legally responsible for a decision that was not theirs. Therefore, if a federal law compels state legislative or regulatory activity, the statute is unconstitutional even if there is a compelling need for the federal action. Chemerinsky, p. 233. Under its commerce power, Congress could clearly pass legislation to establish a federal commission to monitor and curtail alcoholic beverage sales to underage drinkers. However, a 10th Amendment violation arises when implementation of the bill requires each state to pass legislation to establish a local “watchdog” agency to facilitate compliance with congressional intent. Congress cannot compel state regulatory activity. The federal statute will be held invalid under the 10th Amendment. Therefore, choice (D) is the correct answer. Choice (A) is not the best answer. It is true that the sale of alcoholic beverages has a substantial impact on interstate commerce. And the commerce clause of Article I, Section 8 states that Congress may regulate commerce with foreign nations, amongthe states, and with the Indian tribes. Accordingly, it would seem likely that Congress may create a watchdog agency to regulate some aspect of alcohol sales. However, New York v. United States, 505 U.S. 144 (1992) held that Congress’s exercise of its commerce clause powers may not infringe on the states’ rights under the 10th Amendment. Specifically, the Supreme Court explained that Congress may not simply commandeer the legislative processes of the States by directly compelling them to enact and enforce a federal regulatory program. That would be deemed coercive and violate of the 10th Amendment. Choice (B) is incorrect. The 21st Amendment repealed the 18th Amendment, which had prohibited the manufacture, sale, or transportation of intoxicating liquors. Congress probably will not turn to the 21st Amendment to justify its law. Even if Congress did, it would probably violate the states’ 10th Amendment rights against the federal commandeering of state governments. Choice (C) is incorrect. The 11th Amendment states: “The Judicial power of the United States shall not be construed to extend to any suit in law or equity, commenced or prosecuted against one of the United States by Citizens of another State, or by Citizens or Subjects of any Foreign State.” The 11th Amendment thus prevents lawsuits by private citizens against the states; it does not protect states from the federal government.
15
Q
  1. A women’s action group attempted for many months, unsuccessfully, to reach an agreement with the local professional men’s club to admit women to membership. The women’s group instituted a suit for a declaratory judgment in federal court to determine whether the men’s club was subject to the state’s anti-discrimination act.
    Prior to the elections for city officials, four members of the women’s group were sent to picket the offices of the mayor and district attorney, both prominent members of the men’s club. Two members walked outside the front of the mayor’s office building, carrying signs that read, “The mayor is supposed to serve all the people but his lunch club is for men ONLY. So don’t vote for him.” The other two pickets walked outside the rear of the district attorney’s office building, carrying similar signs, telling the public not to vote for him. This picketing was carried on from 9 A.M. to 5 P.M.
    The same day, two more pickets were assigned to carry identical signs in front of the mayor’s official residence. Two pickets also carried duplicate signs in front of the district attorney’s suburban home during the early evening hours. The picketing at all sites was held peacefully without any disturbance. The relevant city ordinances concerning picketing read as follows:
    “Section 201. No picketing shall be permitted inside of, or on any sidewalk or street immediately adjacent or contiguous to, city hall, without express permission of the mayor. Applications for such permission shall be filed at least three days before such picketing is intended to begin and shall state the purpose, place, and time of the proposed picketing.
    Section 202. It shall be unlawful for any person to engage in picketing before or about the residence of an individual. Nothing herein shall be deemed to prohibit the holding of a meeting or assembly on any premises used for the discussion of subjects of general public interest.”
    The federal district court will most likely avoid making a decision on the merits of the suit for declaratory judgment because
    (A) the case lacks adequate ripeness.
    (B) there is no case or controversy.
    (C) the relief sought is essentially for an advisory opinion.
    (D) the women’s group lacks standing.
A
  1. (C) When a federal constitutional claim is premised on an unsettled question of state law, the federal court should “stay its hand” (abstain temporarily), so as to give state courts a chance to settle underlying state law questions. In the present case, the women’s group initially should have brought suit in state court (thereby exhausting all state remedies) before seeking federal judicial review. Clearly, the women’s group is presently “harmed” by the men’s club’s exclusionary membership policy. Therefore, choice (A) is incorrect. Choice (B) is wrong since an actual “case and controversy” is presented by the men’s club’s exclusion of women members. Choice (D) is incorrect. The women’s group would seem to possess standing here: The men’s club has inflicted injury against the women’s group’s members by excluding them; the court can offer a remedy for its injury.
16
Q
  1. A teachers union, a nongovernmental entity, seeks to picket the local city school board for its decision to require higher certification for instructors who wish to teach in the gifted and talented classes in elementary school. After a few days of picketing, the school board seeks a temporary injunction in the state court to restrain further picketing of the school board. The school board insists that the teachers union has violated Section 101 of the city’s picketing ordinance. Section 101 reads as follows:
    “Section 101. No picketing shall be permitted inside of, or on any sidewalk or street immediately adjacent or contiguous to public elementary and secondary schools without express permission of the mayor. Applications for such permission shall be filed at least three days before such picketing is intended to begin and shall state the purpose, place, and time of the proposed picketing.”
    The court will most likely
    (A) grant relief, because the teachers union
    failed to follow the procedure outlined in the
    ordinance.
    (B) grant relief, because the ordinance is
    constitutional.
    (C) deny relief, because the picketing ordinance
    was unconstitutional on its face.
    (D) deny relief, because the ordinance does not
    provide procedural due process.
A
  1. (C) A frequently tested Constitutional Law area deaLs with Licensing statutes. As a
    general rule, whenever an ordinance is “void on its face,” the defendant need not
    apply for a permit. In this situation, he is entitled to deliver his speech and can suc
    cessfuLly defend any subsequent prosecution. An ordinance may be deemed to be
    facially invalid for the following three reasons: (1) “vagueness,” (2) “overbreadth,”
    (3) “unfettered discretion” in the licensing official. On the other hand, when an ordinance is valid on its face (although a permit may be arbitrarily denied), the applicant must seek judicial relief before engaging in his speech. See Poulos v. New Hampshire, 345 U.S. 395 (1953). Choice (A) is incorrect. This would be true if the ordinance were constitutional. Here, the ordinance is invalid on its face because it affords the mayor unfettered discretion to deny permits for picketing. Under the ordinance, the mayor may deny permits to those advocating viewpoints that he finds objectionable. Choice (B) is incorrect. The ordinance is unconstitutional because it affords the mayor unfettered discretion to deny permits for picketing. Under the ordinance, the mayor may deny permits to those advocating viewpoints that he finds objectionable. Choice (D) is not the strongest answer. This is a potentially good answer. However, the ordinance will be struck down on its face because it affords the mayor unfettered discretion to deny permits for picketing. Under the ordinance, the mayor may deny permits to those advocating viewpoints that he finds objectionable.
17
Q
  1. A group of students from a public state college decide to picket the college for raising tuition. After three days of picketing, the college asks a court to grant a temporary injunction against the picketers. The college claims that the picketers have violated Section 201 of the city’s picketing ordinance. Assume that the court granted the temporary injunction against the students for picketing the college.
    “Section 201. No picketing shall be permitted inside of, or on any sidewalk or street immediately adjacent or contiguous to public colleges without express permission of the mayor. Applications for such permission shall be filed at least three days before such picketing is intended to begin and shall state the purpose, place, and time of the proposed picketing.”
    In a subsequent action challenging the constitutionality of Section 201 of the city’s picketing ordinance, the court will most likely rule that the section is
    (A) constitutional, because the ordinance is a valid exercise of the state’s police power.
    (B) constitutional, because the ordinance is within the reserved rights of the states under the Tenth Amendment.
    (C) unconstitutional, because the ordinance is void for vagueness and overbreadth.
    (D) unconstitutional, because the ordinance violates petitioner’s rights under the First Amendment.
A
  1. (D) In Coxv. Louisiana, 379 U.S. 537 (1965), the U.S. Supreme Court held that an analogous ordinance prohibiting “parades or demonstrations along any street except in accordance with a permit issued by the police chief’ was clearly unconstitutional. The Court reasoned that it would be an arbitrary exercise of state police power to enable a public official to determine which expressions of view will be permitted and which will not. Allowing unfettered discretion in local officials in the regulation of the use of streets is an unwarranted abridgement of one’s freedom of speech and assembly as secured by the 1st Amendment and applied to the States by the 14th Amendment. Choice (C) is not the best answer because the statute is neither over- broad nor vague; it is invalid because it gives the mayor the “unfettered discretion” to approve or disapprove permit applications. Choice (A) is incorrect. Courts have stated that the 10th Amendment contains “police powers,” which beLong to local governments. Those police powers permit local governments to regulate speech for purposes of health, safety, welfare, and morals. However, local governments may not pass laws that regulate speech and which are invalid on their face. We have such a law here because it gives the government unfettered discretion to discriminate against certain viewpoints. Such a law is facially invalid. Finally, choice (B) is incorrect. The state is said to have “police powers” under the 10th Amendment, rather than “reserved powers.” Regardless of the term, a state may not use its 10th Amendment powers to pass a law that can permit the state to enjoy unfettered discretion to discriminate against certain viewpoints. Such a law is facially invalid.
18
Q
  1. A city passed a picketing ordinance. Section 202 of that ordinance reads:
    “Section 202. It shall be unlawful for any person to engage in picketing before or about the residence of an individual. Nothing herein shall be deemed to prohibit the holding of a meeting or assembly on any premises used for the discussion of subjects of general public interest.”
    A group of teachers who work for a public school inside of the city were angry that the principal decreased their salaries. The teachers reacted by picketing on the lawn of the principal’s privately owned house. The teachers were charged with violating Section 202 of the ordinance. The teachers have filed suit attacking the constitutionality of Section 202.
    The court will most likely declare Section 202
    (A) constitutional, as a permissible time, place, and manner regulation.
    (B) constitutional, as a valid exercise of a state’s reserved powers under the Tenth Amendment.
    (C) unconstitutional, as discriminatory on its face. (D) unconstitutional, as vague and overbroad.
A
  1. (A) The government may place reasonable restraints on the time, place, and manner of speech in public areas, such as streets, sidewalks, and parks. A public street does not lose its status as a traditional public forum simply because it runs through a residential neighborhood. A three-part test is used to determine the constitutionality of time, place, or manner regulations of speech and assembly in public places. The regulation must: (1) be content-neutral; (2) be narrowlytailored to serve a significant government interest; and (3) leave alternative channels of communication open. Section 202 is content-neutral (the regulation does not prefer some messages over others). It serves the significant government interest of protecting residential privacy. An important aspect of such privacy is the protection of unwilling listeners within their homes from the intrusion of objectionable or unwanted speech. Moreover, Section 202 is narrowly tailored to serve that governmental interest, since, although its ban is complete, it targets and eliminates no more than the exact source of the “evil” it seeks to remedy: offensive and disturbing picketing focused on a “captive” home audience. It does not prohibit more generally directed means of public communication that may not be completely banned in residential areas [Frisby v. Schultz, 487 U.S. 474 (1988)]. Therefore, choice (A) is correct as it comes to the right conclusion and highlights the correct legal issue. Choice (B) is a potentially good answer but certainly it is less precise than (A). Choice (C) is incorrect. For a law that regulates free speech to be declared invalid on its face, it must suffer from one or more of these: (1) “vagueness,” (2) “overbreadth,” (3) “unfettered discretion” in the licensing official. Section 202 does not suffer from any of these. Choice (D) is incorrect. Section 202 is not so vague or overbroad as to be unconstitutional. Alaw is deemed unduly vague if a reasonable person is unable to distinguish what is permitted [City of Chicago v. Morales, 527 U.S. 41 (1999)]. Here, the law is not unduly vague because a reasonable person should be able to determine when picketing is taking place before or about the residence of an individual. A law suffers from overbreadth when it punishes speech that is otherwise protected [Schad v. Borough of Mt. Emphraim, 452 U.S. 61 (1981)]. Here, a person has limited constitutional right of free speech before or about a person’s private property, especially that of a home.
19
Q
  1. A state imposes a graduated income tax upon net income calculated under federal law. Section 22 of the city’s reform act of 2010, which is to become effective on January 1, 2010, provides: “Any parent or guardian financially responsible for the education of his ward may claim a direct tax credit against his income tax liability equal to the amount of tuition of a child or children of high school age who does not attend a public high school.” Other provisions define “tuition” very broadly but limit the credit to tuition paid to schools meeting the educational requirements as determined by the state.
    On December 1, 2009, the parents of two students who attend a public high school in the state sue for a declaratory judgment and injunction in federal court, claiming that Section 22 violates the establishment clause of the First Amendment. The federal court will most likely
    (A) dismiss the action, because the suit involves a political question.
    (B) dismiss the action, because the plaintiffs lack standing as taxpayers.
    (C) dismiss the action, because the issues are not ripe.
    (D) dismiss the action, because the issues being litigated are moot.
A
  1. (C) Since the reform act will not become effective until January 1, 2010, the court will dismiss this action for lack of ripeness. It is crucial that a person asking the court to hold a statute unconstitutional be able to show, not only that the statute is invalid, but also that “he has sustained or is immediately in danger of sustaining some direct injury” as a result of its enforcement. Here, the parents must wait until after the reform act has gone into effect before initiating suit. Choice (A) is incorrect. The court cannot hear the case because the cause of action lacks ripeness. Had the case been ripe, the court would not have been precluded from hearing the case because there is no political question at issue on these facts. A political question is defined as an issue that has been committed to another branch of government. Choice (B) is incorrect because taxpayers generally do not have standing to sue as such [Lujan v. Defenders of Wildlife, 504 u.s. 555 (1992)]. There is a reference to taxes, but the aggrieved parents are not suing as taxpayers. The aggrieved parents are instead arguing that they will suffer a specific harm because they send their children to public schools and thus cannot take advantage of the tax credit. Choice (D) is not the best choice. Article Ill states that the jurisdiction of federal courts only extends to “cases or controversies.” That means federal courts generally may not hear cases that are moot, in other words, those cases where the issues being litigated no longer affect the litigants. For example, there is a probLem of mootness in a suit to overturn a criminal conviction after the convicted dies. In this case, the issues are not moot since the aggrieved parents might suffer harm from the statute.
20
Q
  1. A state passed a statute stating that it will pay for one-half of the tuition for any student attending a private school, whether religious or secular, as long as the school meets the educational requirements established by the department of education.
    The parents of two students who attend a public high school in the state sue for a declaratory judgment and injunction in federal court, claiming that the state statute violates the establishment clause of the First Amendment. Assume that the federal court decides to hear the case on its merits.
    Which of the following is the state’s LEAST persuasive argument for sustaining the validity of the statute?
    (A) The state statute benefits the parents or guardians of all the children in private schools, religious and nonreligious.
    (B) The primary effect of the state statute is not to advance or inhibit religion.
    (C) The administration of the state statute by the department of education does not foster excessive governmental entanglement with religion.
    (D) The state statute is a valid exercise of state regulatory action in the field of education.
A
  1. (D) Choice (D) is the least persuasive argument because it does not address the establishment clause issue clearly raised by the statute, which forms the basis for the parents’ claims. Even if the statute were a valid exercise of state regulatory action in the field of education, the issue of a potential establishment clause violation would remain. Choices (A), (B), and (C) are correct applications of the law and address the claim raised by the parents. Choice (A) would tend to support the view that the state statute does not violate the establishment clause, since both religionists and nonreligionists can benefit. Choice (B) is a plausible argument insofar as both religionists and non-religionists can benefit from the state statute. Finally, choice (C) is a plausible argument insofar as the government is not required to make determinations about what is a religion.
21
Q
  1. A state enacts a statute that will allocate funds generated by state taxes to pay for one-half of the annual salaries of teachers in private schools, whether the schools are public or private, as long as the schools satisfy the requirements established by the United States Department of Education.
    In an action brought by several state taxpayers, all avowed atheists, to challenge the constitutionality of the statute, the enactment will most likely be declared
    (A) unconstitutional, as violative of the First Amendment’s establishment clause.
    (B) unconstitutional, as violative of the First Amendment’s establishment clause as applicable through the Fourteenth Amendment.
    (C) constitutional, as nonviolative of the First Amendment’s establishment clause.
    (D) constitutional, as being within the area of compelling state interest.
A
  1. (B) In Lemon v. Kurtzman, 403 U.S. 602 (1971), the U.S. Supreme Court held unconstitutional a statute that provided for payment of salary supplements to parochial schoolteachers who taught solely secular subjects. The Court concluded that the degree of government surveillance necessary to ensure that the supplements would be restricted to teachers of secular subjects would entangle the government excessively. Choice (A) is not the strongest answer. This is a good answer, but there is one glitch. Remember that the Establishment Clause, on its face, prohibits “Congress” from establishing a religion. The Establishment Clause applies to local governments via the 14th Amendment’s Due Process Clause. Choice (C) is incorrect. Most likely, the statute wilL be deemed to violate the Establishment Clause. The Supreme Court rejected as unconstitutional a statute that provided for payment of salary supplements to parochial schoolteachers who taught solely secular subjects. The Court concluded that the degree of government surveillance necessary to ensure that the supplements would be restricted to teachers of secular subjects would entangle the government excessively [Lemon v. Kurtzman, 403 U.S. 602 (1971)]. Choice CD) is incorrect. It is unlikely that the Court will deem Section 40’s interest to be compelling. The Supreme Court has already rejected as unconstitutional a statute that provided for payment of salary supplements to parochial schoolteachers who taught solely secular subjects. The Court concluded that the degree of government surveillance necessary to ensure that the supplements would be restricted to teachers of secular subjects would entangle the government excessively [Lemon v. Kurtzman, 403 U.S. 602 (1971)].
22
Q
  1. A man entered into a franchise contract with a company to operate a fast-food restaurant in a city. The company’s national headquarters is located in another state. After the contract was executed, the man leased a store in a shopping center where he planned to open his restaurant. City public officials, however, refused to grant the man the necessary food vendor’s license despite the fact that he could prove that his restaurant complied with all provisions of the municipal licensing ordinance. Section 1287 of the city food vending ordinance provides, in part, that “a food vendor’s license shall be issued to any applicant who properly complies with all of the health requirements of this ordinance.” After the man’s application for a food vendor’s license was rejected, he requested a hearing to establish his qualifications. City officials refused this request and also declined to give any reason for his license denial.
    Which of the following is the strongest constitutional argument that the man may use to challenge the refusal of the city officials to grant him a food vendor’s license?
    (A) The city action denies him procedural due process.
    (B) The city action denies him substantive due process by arbitrarily regulating economic activity.
    (C) The city action constitutes an undue burden on the potential interstate commerce between the man and his out-of-state franchisor.
    (D) The city action impairs the obligation of the man’s contract with the franchising company and his rental agreement with the shopping mall.
A
  1. (A) Procedural due process guarantees a fair decision-making process before the government takes some action directly impairing an individual’s life, liberty, or property interests under the 5th and 14th Amendments. Where such a deprivation occurs that is serious in nature, the procedural safeguards of notice and opportunity to be heard (i.e., hearing) are available. Choice (B) is incorrect. There is no fundamental right of contract or “economic activity” under the “substantive due process” clause of the 14th Amendment [West Coast Hotel v. Parish, 300 U.S. 379 (1937)}. Choice (C) is incorrect because in order to establish that a local government is presenting an undue burden on interstate commerce and, thus, violating the commerce clause of Article I, Section 8, there must be a showing that a local government is requiring out-of-staters to comply with a law whose benefits to the local government are outweighed by its burdens on out-of-staters. Choice (D) is incorrect. Article I, Section 10 contains the “contract clause,” which states that no state shall pass any law impairing the obligation of contracts. To sue under this clause, a party would have to show that the state action affected a preexisting contract. The contract at issue in this question does preexist the city’s denial of the license. However, according to Energy Reserves Group v. Kansas Power & Light, 459 u.S. 400 (1973), there must also be a “substantial impairment” of the obligation of contracts. In this instance, the denial does not “substantially impair” the obligation because the company theoretically can still enforce its terms against the man and seek damages.
23
Q
  1. Congress has recently enacted a statute requiring all males between the ages of 18 and 30 to take a physical examination each year. The results of the exam are sent to a government data information center for the purpose of keeping information about men who may be drafted into the military.
    A 25-year-old law school graduate has herpes. He has recently sent resumes to many governmental agencies. Fearful that the information about his herpes condition will become available, he seeks a declaratory judgment that would forbid the government from requiring him to take a physical examination.
    Which of the following is the best constitutional basis in support of the federal law?
    (A) The commerce clause.
    (B) The dormant commerce clause.
    (C) To raise and support an army and navy.
    (D) To provide for the general welfare.
A
  1. (C) Here, choice (C) is the best answer. under Article I, Section 8, Clauses 11 and 12,
    Congress has wide war power authority. Certainly, the draft and selective service
    systems have been repeatedly upheld as a proper exercise of Congress’s war pow
    ers. Based on the decision in United States v. O’Brien, 391 u.s. 367 (1968), indi
    viduals may be constitutionally required to register and submit to examinations
    reasonably required by the selective service system to facilitate the conscription
    of manpower for military service. Choice (A) is incorrect. The Commerce Clause
    of Article I, Section. 8 states that Congress may regulate commerce with foreign
    nations, among the several states, and with the Indian tribes. There does not
    appear to be any commercial transactions in our example, however. Choice (B)
    is incorrect. No congressional statute can be deemed unconstitutional because
    it violates the dormant commerce clause. The concept of the dormant commerce
    clause pertains only to local governments, as when a Local government passes a law
    that discriminates against out-of-staters or places an undue burden on interstate
    commerce. In our example, Congress has passed a law and hence its law cannot be
    deemed to violate the dormant commerce clause. Choice (D) is incorrect. Article I,
    Section 8, reads: “The Congress shall have Power to lay and collect Taxes, Duties,
    Imposts and Excises, to pay the Debts and provide for the common Defence and
    general Welfare of the united States.” The Supreme Court has taken the reference to
    “general welfare” to mean that Congress enjoys great discretion in how it chooses
    to allocate money for the pubLic [United States v. Butler, 297 U.S. 1 (1936)]. There is
    no such allocation of money in our example; just a regulation.
24
Q
  1. The civic auditorium is owned by the city. The auditorium is rented out to various organizations throughout the year. With a seating capacity of 1,500, it is customarily leased for rock concerts, rodeos, sporting events, fashion shows, etc. In January, a rock star held a week-long concert there, drawing a full house each night. Generally, the city leases the auditorium’s facilities for a charge of $2,000 per day.
    In February, a religious sect applied to rent the auditorium for its annual celebration. However, the city council voted 7 to 3 against permitting the group from using the auditorium. When their rental application was denied, the religious sect threatened to take legal action against the city council. They contended unfair discrimination inasmuch as other religious organizations had leased the auditorium in previous months for their religious gatherings. Amid this controversy, the city council passed an ordinance prohibiting the rental of the auditorium to any religious group. The ordinance was passed in a “closed door” session, which did not permit any debate or hearings on the matter.
    Is this newly enacted city ordinance likely to be held constitutional?
    (A) No, because it violates the due process rights of religious groups by not affording them an opportunity for a hearing.
    (B) No, because the ordinance discriminates against religious groups in violation of the free exercise clause of the First Amendment, as applied to the states via the Fourteenth Amendment’s due process clause.
    (C) Yes, because the ordinance treats all religious groups equally.
    (D) Yes, because a city ordinance is not a state action, per Se, and, therefore, is not subject to the limitations of the Fourteenth Amendment.
A
  1. (B) During the 1940s and 19505, the Supreme Court invalidated a number of laws that restricted religious practices primarily on the basis that they interfered with the free speech protection of the 1st Amendment. These decisions overturned statutes regulating the dissemination of religious views because they interfered with both the freedom of speech and religion. In sum, the Court struck down ‘icensing systems for demonstrations or meetings that gave government officials discretion to deny licenses on the basis of the content of speech, while upholding others that had permit requirements based on nondiscriminatory “time, place, or manner” factors. In these cases, the fact that religious meetings were suppressed under discretionary statutes indicated a violation of free exercise rights, but the statutes were invalid in total as they conflicted with the free speech clause. By analogy, the city ordinance would be unconstitutional because it unfairly burdens the freedom of religion (as well as speech). Here, the city ordinance does not further a “compelling state interest,” as less restrictive means are available. Choice (A) is incorrect. According to the 14th Amendment’s due process clause, no state (or other local government) may deprive a person of her life, liberty, or property without due process of law. Generally, no hearing or advance notice is required for laws that are passed and do not target a discrete group of individ ua Is [Bi-Metallic Investment Co. v. State Board of Equalization, 239 u.s. 441 (191 5)1. Here, the religionists, while a defined group, probably do not constitute a group that is sufficiently discrete as to merit a hearing. Of course, this preclusion does not close other legal channels forthe religionists. Choice (C) is incorrect. This may be true, but the equal treatment does not negate the city council’s violation of the religious groups’ free exercise rights under the 1st Amendment. Note, too, the 14th Amendment’s equal protection clause, which states that no person shall be denied the equal protection of the laws. Religious groups are considered to be a “suspect class” under the equal protection clause and discrimination against them will be subject to strict scrutiny. The city council probably will be unable to show that it possesses a “compelling governmental interest” under strict scrutiny. Choice (D) is incorrect. The 14th Amendment, like the other amendments to the Bill of Rights (the first eight Amendments), onLy protect a person from governmental abuse. This is known as the “state action” requirement, even though “state action” refers to any governmental action—including action by any federal or any local governmental body. The city ordinance would thus qualify as state action.
25
Q
  1. A state has enacted a statute imposing a tax on the extraction of all platinum in the state. The extraction of other minerals is not taxed by the state. This is true even though there is considerable mining of silver, turquoise, sulfur, and stone within the state. As a result, the largest platinum mining company in the state has filed suit challenging the constitutionality of the state’s platinum tax statute.
    Which of the following best states the burden of persuasion if the mining company attacks the statute as violating the equal protection of the laws?
    (A) The state must convince the court that the classification in the statute is rationally related to the advancement of a legitimate state interest.
    (B) The state must convince the court that the classification in this statute is the least restrictive means by which to advance a compelling state interest.
    (C) The mining company must convince the court that the classification in this statute is not necessary to advance a compelling state interest.
    (D) The mining company must convince the court that the classification in this statute is not rationally related to the advancement of a legitimate state interest.
A
  1. (D) A highly tested area of Constitutional Law deals with the standards of review used by the courts in equal protection challenges and, specifically, what is the burden of persuasion in each situation? In this question, the state has imposed a statute taxing the extraction of platinum. The tax discriminates against the state’s largest platinum mining company, raising an equal protection challenge, since the state is not taxing the extraction of any other mineral. The standard of review that the court will use in this case will be the traditional rational-basis test. The rational-basis test, which is used for classifications relating to non-suspect classes and to non-fundamental rights, includes the following: classifications based on poverty, wealth, age, public housing, and welfare benefits. See Nowak, Constitutional Law pp. 620-623. under the rational- basis test, the burden is on the plaintiff to prove that the challenged measure is not rationally related to a legitimate state interest. Choice (D) is correct. Note that it is extremely difficult for the plaintiff to prevail under the “mere rationality,” or rational- basis test, because any legitimate interest that is reasonably furthered by enactment of the challenged measure will be purportedly valid. Choice (A) is incorrect. We do not have a suspect or “quasi suspect” cLass that is being created by the law. Hence, the law will be subject to rational review, under rational review, the challenger—not the state—bears the burden of showing that the law does not advance a legitimate governmental interest or that the law’s means are not rationally related. Choice (B) is incorrect. If the state’s law had created a suspect class, it would be subject to strict scrutiny. Under strict scrutiny, one of the things that the state would have to show is that the means were least restrictive. Unlike race, ethnicity, nationality, or religion, the mining company is not a member of a suspect class. Thus, no strict scrutiny is warranted; rational review will be applied. Choice (C) is incorrect. If the state’s law had created a suspect class, it would be subject to strict scrutiny. Under strict scrutiny, one of the things that the state would have to show is that the government’s interests were compelling. Unlike race, ethnicity, nationality, or religion, the mining company is not a member of a suspect class. Thus, no strict scrutiny is warranted; rational review will be applied.
26
Q
  1. A woman attended a private university. One afternoon she was approached by a photographer for a magazine, who asked her if she would be interested in posing nude for an upcoming issue. The woman, who was an aspiring model, agreed and posed for a number of nude pictures, which subsequently appeared in the magazine.
    Afterward, administrators and professors at the university began to harass her for what they considered to be imprudent behavior. During class, her instructors frequently called her “Playmate” and “Stripper.” Consequently, the woman brought suit in federal court against the university, alleging harassment, and seeking an injunction and damages. After this action was instituted, the university signed a stipulation agreeing not to harass her in the future.
    The federal court should now
    (A) hear the case.
    (B) dismiss the action as moot.
    (C) dismiss the action since the issues are no longer ripe.
    (D) dismiss the action because there is no case or controversy.
A
  1. (A) The doctrine of standing requires that a person asserting the violation of a constitutional (or statutory) right must show a direct and immediate personal injury due to the challenged action. Choice (B) is incorrect since it is irrelevant whether the university signed a stipulation agreeing not to harass the woman in the future. The facts clearly indicate that the plaintiff suffered personal injury (humiliation and damage to reputation), which can be traced to the challenged action. The federal court will likely hear the case because a genuine “controversy” is presented. Choice (C) is incorrect. A case is said to lack ripeness and, thus, not justiciable if the alleged injury is only speculative. Here, the woman has already suffered injury from her instructors because of their harassing remarks. Finally, choice (D) is incorrect. In orderfora federal court to review a case under Article Ill, there must be a “case or controversy.” That means that there must be some injury orthreat of an injury. Here, the woman is able to show that she has suffered injury from her instructors because of their harassing remarks.
27
Q
  1. Section 2022(a) of a state’s medical licensing code provides:
    “For the purposes of this statute, only those persons who have graduated from an optometry school located in the state and accredited by the board of optometrists shall be licensed to conduct the practice of optometry within the state.”
    A doctor graduated from a school of optometry located in a neighboring state. The following month, the doctor was granted a license to practice optometry in that neighboring state by its board of optometrists. For approximately one year, the doctor was engaged in the practice of optometry in the neighboring state. Subsequently, the doctor decided to relocate his practice to the state for which Section 2022(a) applies.
    The doctor re-domiciled in the state, where he opened a new office for the practice of optometry. When he initially opened his office, the doctor was unaware of the state’s licensing provision for optometrists. Since the doctor was a licensed optometrist for 10 years, he assumed that he could practice optometry in the state by reciprocity.
    The doctor received notification from the state’s board of optometrists that he was illegally practicing optometry and that he must immediately cease and desist from such practice.
    If the doctor challenges the constitutionality of Section 2022(a) of the medical licensing statute as violating the Commerce Clause, which of the following, if factually true, is the WEAKEST defense that the state could offer?
    (A) The statute will help to ensure that only the most qualified optometrists practice in the state.
    (B) The statute will help protect the optometrists in the state from competition outside the state.
    (C) The statute will help to ensure a continuously available number of practicing optometrists within the state.
    (D) The statute will help to ensure that the practicing optometrists in the state are subject to standards of a regulatory body.
A
  1. (B) The WEAKEST defense to the doctor’s constitutional challenge of the statute under the Commerce Clause is that the statute will help protect optometrists in the state from out-of-state competitors. State regulations, such as the medical licensing statute, that appear primarily to favor local economic interest as against outside competition are generally found to be unconstitutional as unreasonable burdens on interstate commerce. However, if the regulation is perceived as within a legitimate health and safety measure that cannot readily be achieved by less drastic means, a court will uphold even a regulation that is discriminatory. While a state may legislate to protect the health of its citizens, the statutory purpose regarding the licensing provision could be achieved by “reasonable nondiscriminatory alternatives, adequate to conserve the legitimate interests of the local optometrists.” Therefore, the statute could be challenged as a discriminatory burden on interstate commerce. Students should refer to Dean Milk Co. v. Madison, 340 U.S. 349 (1951), where the Supreme Court invalidated a Local health ordinance under the “unreasonable burdens” rule. Choice (A) is not the best answer. By favoring optometrists who are trained in the state, the state is in danger of presenting an undue burden on interstate commerce and, thus, violating the dormant commerce clause of Article I, Section 8. Under dormant commerce clause analysis, the only acceptable justification for such discrimination is safety or health. The state might be able to argue that its requirements, while discriminating against out-of-staters, are likely to produce optometrists who are better skilled and, thus, more likely to render service that will promote health and safety. Choice (C) is not the best choice. By favoring optometrists who are trained in the state, the state is in danger of presenting an undue burden on interstate commerce and, thus, violating the dormant commerce clause of Article I, Section 8. Under dormant commerce clause analysis, the only acceptable justification for such discrimination is safety or health. The state might be able to argue that its requirements, while discriminating against outof-staters, will produce a continuously available number of practicing optometrists within the state; hence, the state’s law will contribute to the visual health and perhaps even safety of the state’s residents. Choice (D) is not correct. By favoring optometrists who are trained in the state, the state is in danger of presenting an undue burden on interstate commerce and, thus, violating the dormant commerce clause of Article I, Section 8. Under dormant commerce clause analysis, the only acceptable justification for such discrimination is safety or health. The state might be able to argue that its requirements, while discriminating against out-of-staters, will produce a continuously available number of practicing optometrists within the state; hence, the state’s law will contribute to the visual health and perhaps even safety of the state’s residents.
28
Q
  1. A security guard relocated to a state which required all persons wishing to practice private security to have graduated from a security training program within the state. The security guard, having already graduated from a program in his former state, wishes to work again as a private security guard. He began to advertise his services but was informed by the state that he must cease and desist from working as a private security guard until he complied with state law and graduated from an in-state program.
    The security guard challenges the constitutionality of the licensing statute on grounds that it violates the due process clause of the Fourteenth Amendment. Which of the following statements is most accurate?
    (A) The security guard has the burden of persuasion to show that the state does not have a compelling state interest in enacting such legislation.
    (B) The security guard has the burden of persuasion to show that the denial of a license to work in private security violates his rights of due process.
    (C) The state has the burden of persuasion to show a compelling state interest in enacting such legislation.
    (D) The state has the burden of persuasion to show that the denial of a license to work in private security does not violate the petitioner’s rights of due process.
A
  1. (B) Since the security guard is challenging the constitutionality of the licensing statute, he has the burden of persuasion to show that the denial of the license violates his rights of due process. Upon such a showing, the burden then shifts to the state to prove that it has a compelling state interest in enacting such legislation. In In re Ruffalo, 390 U.S. 544 (1968), the Supreme Court held that if the government terminates an individual’s ability to engage in a profession, it must grant that individual a procedure to determine his fitness to be a member of the profession. Thus, an individual must be afforded a fair hearing before a government agency may revoke his license to practice a profession, such as, in this case, security. Choice (A) is incorrect. The security guard will first need to show that his due process rights have been violated when the state prohibited him from using his out of state private security license. If the security guard succeeds, the state will have the burden of showing that its licensing requirement furthers a compelling governmental interest. Choice (C) is incorrect. The security guard will first need to show that his due process rights were violated when the state prohibited him from using his out of state private security license. It is only after such a showing that the state bears the burden to prove that its denial furthered a compelling state interest. Choice (D) is incorrect. The security guard has the initial burden to show that the denial of his license violated his fundamental rights of due process. If the security guard is able to show this, the state then bears the burden of showing that its denial constitutes a compelling governmental interest and that the denial was the least restrictive means available to advance the compelling governmental interest.
29
Q
  1. A state recently enacted a statute requiring the following of all firms that do business with and for the state (defined generally as selling goods or providing services to the state, its agencies, or subdivisions):
    — Such businesses must purchase insurance only from insurance companies chartered in the state and thus be subject to regulation by the state insurance commissioner;
    — In hiring any unskilled laborers for employment in connection with state business, preference must be given to citizens of the state;
    — Nonresident aliens shall be prohibited from engaging in any state related business activities;
    — All buildings constructed for the state must have roofs composed of adobe, which is composed of yellow silt or clay deposits found only in the state.
    The proposed statute’s preamble recites that its provisions will ensure:
    — responsible insurance coverage for all those who do business with the state;
    — an increased standard of living for the citizens who comprise the state’s labor force;
    — the lowest possible expenditures by the state government; and
    — a beautiful, uniform aesthetic decor for all new state buildings.
    In evaluating the constitutionality of the state statute under the Supremacy Clause of Article VI, which of the following would be most relevant?
    (A) The number of aliens presently residing in the state.
    (B) The necessity for the enactment of this particular statute.
    (C) The treaties and immigration laws of the United States.
    (D) The overall unemployment rate in the United States.
A
  1. (C) Under the Supremacy Clause of Article VI, federal treaties are the supreme law of the land and supersede any state statutes which are in conflict therewith. According to the Supreme Court, Congress is vested under Article I, Section 8 with the primary authority to regulate immigration and aliens. Any state legislation that affects aliens is thus likely to be preempted (depending, however, in part on the activity regulated). Note that the Supreme Court has upheld a state law forbidding employment of illegal aliens under the state’s police power, since it did not interfere with the exclusive federal power over aliens. Choice (A) is incorrect. According to the Supreme Court, Congress is vested under Article I, Section 8 with the primary authority to regulate immigration and aliens. If Congress passes a law that is valid under this power to regulate immigration and aliens, the law, according to the Supremacy Clause, takes priority over any conflicting state laws. So, too, in some fields, such as immigration, Congress enjoys “field preemption” under the Supremacy Clause; under field preemption, any state statute that overlaps Congress’s field of regulation is also deemed unconstitutional. The number of aliens presently residing in the state is not relevant in determining whether the state statute conflicts with a congressional statute or whetherthere is field preemption. Choice (B) is incorrect. The necessity for the enactment of the state statute is mostly irrelevant in determining its constitutionality under the Supremacy Clause. Even if the state could show that its reasons for the statute were fabulous, the state could not save its statute from being preempted if the statute either conflicted with an otherwise constitutional federal statute or regulated in a field like immigration that was deemed by the Supreme Court to be an exclusively federal domain. Choice (D) is incorrect. The overall unemployment rate in the United States is mostly irrelevant in determining the constitutionality of the state statute under the Supremacy Clause. Much more important is to determine whether the state statute either violates an otherwise constitutional federal statute or whether the state statute regulates in a field like immigration that was deemed by the Supreme Court to be an exclusively federal domain.
30
Q
  1. A state recently enacted a statute requiring the following of all construction companies that do business with and for the state (defined generally as selling goods or providing services to the state, its agencies, or subdivisions):
    Such businesses must purchase insurance only from insurance companies chartered in the state and thus be subject to regulation by the state insurance commissioner;
    — In hiring any unskilled laborers for employment in connection with state business, preference must be given to citizens of the state;
    Nonresident aliens shall be prohibited from engaging in any state related business activities;
    — All buildings constructed for the state must have roofs composed of adobe, which is composed of yellow silt or clay deposits found only in the state.
    The proposed statute’s preamble recites that its provisions will ensure:
    — responsible insurance coverage for all those who do business with the state;
    an increased standard of living for the citizens who comprise the state’s labor force;
    — the lowest possible expenditures by the state government; and
    — a beautiful, uniform aesthetic decor for all new state buildings.
    If the state statute is attacked as violating the commerce clause, which of the following statements is most accurate?
    (A) The statute is a valid exercise of the state’s police power to legislate to protect the health, safety, morals, and welfare of its citizens.
    (B) The statute falls within the reserved powers under the Tenth Amendment.
    (C) The statute should be invalidated if there are nondiscriminatory reasonable alternatives available to serve legitimate local interests.
    (D) The statute is a valid exercise of state action only if the federal government has not previously “occupied the field” in this area.
A
  1. (C) If the state statute is challenged on commerce clause grounds, the court would apply the unreasonable burdens rule (also referred to as the least restrictive means test). State regulations, even in areas generally described as local, cannot be accepted if they impose an unreasonable burden on interstate commerce. In orderto determine the reasonableness of the burden, the court will balance the nature and extent of the burden that the state regulation would impose on interstate commerce against the purposes of the state regulation. Subsection (d) of the statute would place a discriminatory burden on producers of other building materials, both within and outside the state. Choice (A) is incorrect. Under commerce clause analysis, the court will determine whether a state’s benefits outweigh the burdens on interstate commerce. Here, Subsection (d) of the statute would place a discriminatory burden on producers of other building materials, both within and outside the state. Choice (B) is incorrect. The 10th Amendment contains the so-called police powers to regulate health, safety, welfare, and morals. The invocation of police powers, however, will be insufficient to help the state against the conclusion that it violates the dormant commerce clause. Choice (D) is incorrect. Actually, the state statute can still be held invalid for violating the commerce clause, irrespective of whether the state statute has occupied a field that is exclusive to the federal government. The Supremacy Clause of Article VI would invalidate local laws that conflict with the U.S. Constitution, including the commerce clauses therein.
31
Q
  1. A state has recently enacted a statute prohibiting the sale of beer in glass bottles. In accordance with the new law, all beer consumed within the state must be sold in aluminum cans that are recyclable. There is a provision of the statute that does permit breweries to distribute beer to bars, taverns, and restaurants in kegs for “on-site” consumption by patrons.
    Before the passage of the law, approximately 28% of all beer consumed in the state was packaged in glass bottles. Of that total, 75% of the beer was bottled outside the state, while 25% was bottled by companies in-state. The legislature passed the law at the strong urging of the aluminum can industry. It was estimated that one aluminum company alone spent over $5,000,000 in its lobbying campaign for the passage of the bill. Ironically, the new law even received strong support from environmentalists who believed that recyclable cans would help prevent littering and unsightly trash accumulation.
    The strongest federal constitutional argument against the validity of the law is that it violates the
    (A) equal protection clause of the Fourteenth Amendment by discriminating against beer bottlers.
    (B) privileges or immunities clause of the Fourteenth Amendment by preventing out-of-state beer bottlers from conducting their business in the state.
    (C) commerce clause by violating the negative implications on interstate commerce.
    (D) contracts clause by impairing the ability of beer bottlers to honor existing contracts for the sale of bottled beer in the state.
A
  1. (C) Although the federal commerce power is plenary, the states may also regulate commerce among themselves, subject to implications flowing from the Supremacy Clause. State regulation of interstate commerce must be reasonable and non-discriminatory, and the states may not interfere with the free flow of interstate commerce by favoring local interests or burdening out-of-state competition. The state statute, which prohibits the sale of beer in glass bottles, will effectively curtail outof-state production of 21% (75% of 28%) of all the beer consumed in the state. It is arguable that this large proportion is not only discriminatory against out-of-state bottlers, but unreasonable as well, since the urging of the aluminum can industry was the basis on which the state legislature passed the bill. Based on these facts, the strongest argument against the validity of the state statute would be that it vioLates the Commerce Clause and the negative implications flowing from it. Choice CD) is not the strongest answer because the Commerce Clause is a broader and stronger source of power than the contracts clause. Choice (A) is incorrect. The equal protection clause is not as strong an argument because even though the statute does arguably discriminate against out-of-state bottlers, the standard of review the court would use to determine its validity would be the rational-basis test, under which the state could prevail upon a showing of any rational basis for enacting the bill. Choice (B) is incorrect. Remember that the Privileges or Immunities Clause of 14th Amendment (as well as the Privileges and Immunities Clause of Article IV) only applies to U.S. citizens. A company is not a citizen.
32
Q
  1. A state has recently enacted a statute prohibiting the sale of computer printer ink cartridges in plastic bags. In accordance with the new law, all ink cartridges within the state must be sold in paper cartons that are recyclable.
    Before the passage of the law, approximately 28% of all ink cartridges used in the state was packaged in plastic bags. Of that total, 75% of the ink cartridges was packaged outside the state, while 25% was packaged by companies in-state. The legislature passed the bill at the strong urging of the paper industry. It was estimated that one paper company alone spent over $5,000,000 in its lobbying campaign for the passage of the bill. Ironically, the new law even received strong support from environmentalists who believed that recyclable paper would help prevent littering.
    Assume that the state Supreme Court adjudges the law to be unconstitutional on the grounds that it violates the contracts clauses of both the federal and the state Constitutions. The contracts clause of the state Constitution is similar to the one in the federal Constitution. The court so held because, in its judgment, the statute retroactively impairs the ability of plastic bag packagers to honor their existing contracts for the sale of ink cartridges packaged in plastic bags. The state attorney general now seeks review of this decision in the U.S. Supreme Court.
    How should the U.S. Supreme Court rule on this case?
    (A) Refuse to review this case on the merits because there is an adequate and independent state ground for the decision rendered below.
    (B) Reverse the decision on the merits with respect to the state constitutional issue because the federal constitutional holding rendered below makes such a state constitutional decision unnecessary.
    (C) Affirm the decision on the merits with respect to the federal constitutional issue and abstain from reviewing the state constitutional issue.
    (D) Affirm the decision on the merits with respect to both the federal and state constitutional issues because the state constitution must substantially conform with the federal constitution on this issue.
A
  1. (A) The Supreme Court may properly deny review of any matter that rests upon an adequate and independent state ground, since resolution of the “state” issue by the state court might preclude the need for federal review. As a general rule, the Supreme Court, upon reviewing a decision of a state court, reviews only the federal questions and not the state law questions [Nowak, Constitutional Law, p. 85]. Since the law was found by the state court to be unconstitutional as violative of the contracts clause of the state constitution, the Supreme Court would avoid review of the case on the merits because a separate and sufficient state law basis for unconstitutionality exists. Choice (A) is correct in stating this conclusion under the doctrine of adequate and independent state grounds. Choice (B) is incorrect as a misstatement of this doctrine, since the Supreme Court will decline to hear the case, not reverse the decision. Choices (C) and (D) reach the wrong conclusions because the Supreme Court will refuse review altogether, as long as any adequate state ground for state court review exists.
33
Q
  1. A state has recently released medical statistics showing that the number of new AIDS cases within the state has quadrupled from the preceding year. In 2008, the state reported that 2,250 people were diagnosed as being stricken with the AIDS virus. However, in 2009 the state confirmed that over 9,000 new persons contracted the deadly virus. In an effort to improve the health care of AIDS patients in the state, the Legislature has enacted a law providing public funds to assist privately owned hospitals.
    According to the law, every hospital in the state would receive $5,000 annually for each AIDS patient who was admitted to that hospital, and whose period of hospitalization exceeded one week. Although this bill was initially opposed by several churches and other organizations, the state’s legislature re-drafted the bill in a compromise effort to appease the opposition. In its final re-draft, the bill provided that the $5,000 annual subsidy “would not be paid to any hospital performing abortions.”
    Which of the following is the strongest argument against the constitutionality of the state’s statute?
    (A) The statute violates the establishment clause of the First Amendment, as incorporated into the Fourteenth Amendment, by adopting the controversial views of particular churches on abortion.
    (B) The statute violates the Fourteenth Amendment by conditioning the availability of public funds upon the recipient’s agreement to act in a way that makes more difficult the exercise by others of their fundamental constitutional rights.
    (C) The statute violates the equal protection clause of the Fourteenth Amendment by denying nonAIDS patients the same subsidy benefits as those received by AIDS patients.
    (D) The state will be unable to show it is advancing a legitimate governmental interest.
A
  1. (B) This fact situation is extremely representative of both the difficulty of Constitutional Law questions on the bar and of the closeness between long, and often similar, answer choices. The substantive guarantees of due process under the 14th Amendment require that legislation, to be constitutional, have a rational relationship to a legitimate end of government. In the area of fundamental rights, such as privacy (and including abortion), governmental power is limited to the extent that individuals may be afforded freedom of choice in matters relating to their personal life. The state statute limits the availability of funds for AIDS patients only to hospitals refusing to perform abortions. While the statute does not preclude individuals from having abortions performed, it does make more difficult the exercise of that right. Since legislation restricting fundamental rights is viewed under the strict scrutiny standard of review, the state then has a heavy burden to show that the measure is necessary to further a compelling interest. Choice (B) is the strongest argument presented and is, therefore, correct. Choice (A) is not the strongest answer. This is a potentially good choice, but it is uncertain whether the statute was motivated by religious views; the reference to churches in the facts are not conclusive. One could theoretically be opposed to abortion on strictly secular grounds. Choice (C) is incorrect. Under equal protection clause scrutiny, strict scrutiny will be applied to those laws that create suspect classes: race, ethnicity, nationality, and religion. Intermediate review will be applied to legal classifications based on gender and illegitimacy. All other legal classifications are subject to rational review, which is relatively easy for the state to pass. Here, non-AIDS patients will only be entitled to rational review under the equal protection clause. Finally, choice (D) is incorrect. The state statute will probably be subject to strict scrutiny because it is infringing a woman’s fundamental right to an abortion. Hence, the state will have to show that it is advancing something greater than a legitimate governmental interest; the state will have to show that it is advancing a compelling governmental interest.
34
Q
  1. In an effort to improve the health care of cancer patients, a state’s legislature has enacted a law providing public funds to assist privately owned hospitals. According to the law, every hospital in the state would receive $5,000 annually for each cancer patient who was admitted to that hospital, and whose period of hospitalization exceeded one week. Although this bill was initially opposed by several churches and other organizations, the state’s legislature re-drafted the bill in a compromise effort to appease the opposition. In its final re-draft, the bill provided that the $5,000 annual subsidy “would not be paid to any hospital performing abortions.”
    Which of the following is the strongest argument in support of the constitutionality of the state statute?
    (A) The Tenth Amendment reserves to the states plenary power over the allocation of their public funds.
    (B) Public subsidies in hospitals are privileges rather than rights and, therefore, are not entitlements protected by the due process clause of the Fourteenth Amendment.
    (C) The funding limitation in this statute does not directly prohibit or penalize the exercise of a fundamental right and is rationally related to the achievement of a legitimate state interest.
    (D) The statute promotes a compelling state interest in advancing the health, safety, and welfare of its citizenry.
A
  1. (C) Choice (A) is incorrect since the 10th Amendment’s reserved powers do not authorize plenary power to the states regarding allocation of public funds. Choice (B) is incorrect since the court would generally examine the purpose and circumstances underlying the authorization of public subsidies before mechanically applying the rights-privilege rationale. Choices (C) and (D) are both very persuasive. To determine which one is stronger, consider that the primary purpose of the statute itself is to provide public funding to hospitals for cancer patients, not to directly restrict abortion. This purpose promotes a legitimate interestwhich, under equal protection analysis, would be reviewed using the rational-basis standard, as stated in choice (C). Since choice (C) presents a federal constitutional basis of analysis, whereas choice (D) addresses merely a source of state power (i.e., police powers), choice (C) is preferred and, therefore, the correct answer.
35
Q
  1. A company operates passenger buses to all the major cities on the east coast. This service is authorized under a certificate of convenience and necessity issued by the Interstate Commerce Commission, pursuant to federal statute. The certificate does not, however, specify particular highways, streets, or locations for the bus service. The company’s advertising stresses that it picks up and delivers passengers at the center of each city that it serves. The company’s management regards this as a particularly effective advertising point in competition with the airlines and the railroads, because short-haul traffic supplies a major part of the bus company’s revenues.
    One of the major cities on the east coast, acting to relieve traffic congestion and air pollution, has recently enacted an ordinance that prohibits (a) the operation of all trucks and buses in a five- square- mile central business area, known as center city, between the hours of 10:00 A.M. and 4:00 P.M. on weekdays, and (b) all on-street parking of passenger automobiles in center city between the same weekday hours.
    The company’s bus station in this city is located in the heart of the center city area. According to its transportation schedules, more than 75 buses either enter or leave this station between the hours of 10:00 A.M. and 4:00 P.M. each weekday.
    If the company brings suit challenging the constitutionality of the city ordinance, the court will most likely declare the ordinance
    (A) constitutional, because it is within the city’s police power to regulate transportation services in the center city business district.
    (B) constitutional, because it is a valid exercise of municipal regulation in the area of intrastate commerce.
    (C) unconstitutional, because it violates the dormant commerce clause.
    (D) unconstitutional, because the ordinance is discriminatory per Se.
A
  1. (C) In all likelihood, the city ordinance prohibiting the operation of all trucks and buses within its center city business area between 10:00 A.M. and 4:00 P.M. would be violative of the dormant Commerce Clause, if less restrictive alternatives are available. Choice (A) is incorrect. It is true that the city council does have the police power under the 10th Amendment to regulate transportation within its jurisdiction because the state’s police powers of the 10th Amendment include the right to regulate matters concerning health, safety, morals, and welfare. However, the city council does not enjoy an unqualified right to exercise its police powers. Here, the city council appears to have violated the dormant Commerce Clause of Article I, Section 8 by presenting an undue burden on interstate commerce. Choice (B) is incorrect. The city council’s ordinance does regulate activities within its borders. But it also appears to present an undue burden on interstate commerce and, thus, will likely be deemed to violate the dormant Commerce Clause of Article I, Section 8. Choice (D) is incorrect. Actually, the law is not discriminatory, perse, since it affects intrastate commerce as well. However, the law does appear to present an undue burden on interstate commerce and will likely be deemed unconstitutional under the dormant Commerce Clause of Article I, Section 8.
36
Q
  1. A city ordinance provides that it shall be unlawful for any person to litter in a public park, and that a violation of this ordinance shall be punished by a fine of not more than $100 or imprisonment for not more than 30 days.
    A protestor organized a demonstration against federal governmental support to rebels in a foreign country. The rally took place in a city park, and attracted about 100 supporters. During the rally, the protestor delivered a speech to the attendees. At the conclusion of his speech, the protestor said, “I’m sick and tired of the garbage this administration is getting away with. Here’s what I think about its policy of rebel aid …“ At which point, the protestor walked over to a trash can and dumped its contents on the ground. As the crowd cheered wildly, the protestor shouted, “No more rebel aid … let’s stop this garbage now!” After littering the park, the protestor and his supporters left without picking up the trash.
    As a matter of constitutional law, may the protestor be prosecuted under the aforementioned city ordinance for littering the public park?
    (A) No, because littering the park in these circumstances could be construed as symbolic speech and, thus, it is protected from government regulation by the First and Fourteenth Amendments.
    (B) No, because the facts do not indicate that the protestor’s actions presented a clear and present danger that was likely to produce or incite imminent lawless action, thereby necessitating an abridgment of his freedom of speech.
    (C) Yes, because the city ordinance advances an important and legitimate public interest and is not directed at the suppression of communication.
    (D) Yes, because littering the park is conduct, not speech and, therefore, it may not be treated by the law as communication.
A
  1. (C) When the government regulates speech in a traditional public forum, it may only base its restriction on the content of the speech being regulated (1) if that content falls within a category of speech that the court has found unprotected by the 1st Amendment, or (2) if the government can demonstrate a compelling interest in suppressing the speech. However, the government may employ a time, place, or manner regulation to regulate speech in a traditional pubLic forum (streets, parks) so long as the regulation promotes an important interest unrelated to the suppression of a particular message and does not unnecessarily restrict the ability to communicate the message. Nowak, Constitutional Law pp. 975-76. In this question, the test maker is trying to trick students into thinking the Rosemont city ordinance regulates a protected 1st Amendment area. In fact, the littering ordinance regulates neither speech-related conduct nor speech-related content. The regulation promotes a legitimate objective by advancing a health and safety interest under the state’s police powers, and the ordinance passes 1st Amendment scrutiny since it is not directed at the suppression of communication. Choice (C) is correct. Choice (A) is incorrect. Even if the speech were considered as symbolic speech, it can still be regulated under the circumstances. The city has passed a proper time, place, and manner regulation that regulates the secondary effects of speech—not the speech itself—and provides reasonable alternatives for the protestor to say more or less the same thing without littering. Here, the city is advancing an important governmental interest in preventing littering (a secondary effect of the protestor’s speech), not deterring speech. Choice (B) is incorrect. In order to regulate the protestor’s littering, the city does not have to show that his speech constituted a clear and present danger. All that is required is that the city show it has passed a proper time, place, and manner regulation that regulates the secondary effects of speech—not the speech itself—and provides reasonable alternatives for the protestor to say more or less the same thing without littering. Here, the city is advancing an important governmental interest in preventing littering (a secondary effect of the protestor’s speech), not deterring speech. Choice (D) is incorrect. Littering might constitute symbolic speech since it is a combination of both conduct and speech. However, even if it were symbolic speech, the city is justified in regulating it because the city has used a proper time, place, and manner regulation that regulates the secondary effects of speech—not the speech itself—and provides reasonable alternatives for the protestor to say more or less the same thing without littering. Here, the city is advancing an important governmental interest in preventing littering (a secondary effect of protestor’s speech), not deterring speech.
37
Q
  1. A march was held to call attention to the fact that minorities are still disenfranchised from the electoral process. A congressional report revealed that in the certain parts of the country, only 42% of minorities eligible to vote were, in fact, registered. The report also indicated that certain states had residency laws restricting a person’s right to vote. As a consequence, Congress enacted a statute that makes any law denying minorities the right to vote shall be deemed unconstitutional.
    This federal statute will most likely be upheld under which of the following constitutional provisions?
    (A) Thirteenth Amendment.
    (B) Fourteenth Amendment.
    (C) Fifteenth Amendment.
    (D) Twentieth Amendment.
A
  1. (C) The 15th Amendment is a limitation prohibiting the states and the federal government from denying any citizen the right to vote on accountof race orcolor. Note, too, that the 15th Amendment has an “enabling clause” that allows Congress to enact legislation protecting against discrimination affecting the right to vote. Choice (A) is incorrect because the 13th Amendment provides that slavery shall not exist in the United States. Choice (B) is wrong because the 14th Amendment prohibits states from depriving any person of life, liberty, or property without due process and equal protection of the laws. Finally, choice (0) is incorrect because the 20th Amendment deals with presidential and congressional terms, not voting rights.
38
Q
  1. An illegal alien and her three children live in a U.S. city. The city council has enacted an ordinance requiring illegal aliens to pay a $100 “school fee” for each child enrolled in a city public school. Citizens and legal aliens are not required to pay the school fee.
    The city council has enacted this law to raise funds to hire additional teachers who are bilingual. The city determined that over 15% of children attending public schools in the city were illegal aliens. Furthermore, the city conducted a study and found that the overwhelming majority of illegal aliens residing in the city did not pay any local property taxes. As a result, since the city provided educational benefits to the children of illegal aliens, the school fee furthered a significant governmental interest.
    The illegal alien, who is indigent, is unable to pay the city school fee. The city will not allow her children to attend school unless the fee is paid. If she seeks your legal advice regarding the constitutionality of the fee, you should advise her that the ordinance is
    (A) valid, because the city can demonstrate that the school fee is necessary to further a compelling governmental interest.
    (B) valid, because the imposition of the school fee is substantially related to a legitimate governmental interest.
    (C) invalid, because denying educational services to children of undocumented aliens is not substantially related to an important governmental interest.
    (D) invalid, because the cost of educating the children imposed an undue burden on the public school system by requiring the school board to hire additional bilingual teachers.
A
  1. (C) In Plylerv. Doe, 457 U.S. 202 (1982), the Supreme Court held that illegal alien chil
    dren are entitled to a free public education. In Plyler, the Court struck down a Texas
    statute that (1) denied local school districts funds for education of illegal alien
    children, and (2) allowed school districts to deny a free public education to these
    children. The majority of justices determined that the Equal Protection Clause of
    the 14th Amendment was intended to cover any person physically within a state’s
    borders regardless of the legality of his presence. Also, the Court rejected the notion that illegal aliens be treated as a “suspect” class. Instead, the Courtapplied “intermediate-level” scrutiny based on the following two factors: (1) the importance of public education, and (2) the powerless nature of the group. Thus, choice (C) is correct. Choice (A) is incorrect. The city probably will be unable to show that its school fee is necessary—the least restrictive means—to further a compelling governmental interest. In this instance, the city could have raised taxes for everyone, for example, to cover the cost for bilingual teachers. Choice (B) is incorrect. The problem with this answer is that the city’s school fee will be subject to strict scrutiny. The facts here are similar to those in Plyler v. Doe, 457 U.S. 202 (1982), where the Supreme Court rejected as unconstitutional, under the 14th Amendment’s Equal Protection Clause, a Texas statute that denied public education to children who were undocumented aliens. Plyler subjected Texas’s statute to strict scrutiny. The language of “substantially related” and “legitimate means” suggests a level of scrutiny below strict scrutiny. Choice CD) is incorrect. The language of “undue burden” generally comes from dormant commerce clause analysis. But, as phrased, the reference to undue burden in this answer does not involve any dormant commerce clause issues.
39
Q
  1. A state highway runs through the center of a city’s business district. As the highway passes through this business district, there is a stretch where the highway is too narrow to satisfS’ the safety standards for state roadways. Along this narrow stretch and on opposite sides of the street are located two businesses. Situated on the east side of the street is a bar and grill; directly across the street on the west side is a lounge.
    In order to comply with state highway regulations, the state highway department, acting pursuant to state statute, ordered the city to prohibit parking on one side of the highway within the business district. The city council convened a special meeting to determine whether parking should be prohibited on the east or west side of the street. During the meeting, the council was unable to resolve the issue. Finally, the city council president decided the only fair way to resolve the matter was to flip a coin. After the coin flip, the city prohibited parking at all times on the west side of the highway. On account of this decision, parking in front of the lounge was permanently prohibited. This caused the lounge to suffer a substantial decline in business because of the unavailability of on-street parking.
    As a consequence, the owner of the lounge brought suit for declarative relief against the city alleging that the parking ban on his side of the street was unconstitutional. In all likelihood, will the plaintiff be successful in this cause of action?
    (A) No, because as a governmental entity, the city enjoys immunity from such suits under the Eleventh Amendment.
    (B) No, because the city’s action was reasonable.
    (C) Yes, because the action of the city violated the equal protection clause of the Fourteenth Amendment.
    (D) Yes, because responsible government officials cannot conduct or formulate their decisionmaking processes by means of coin flips.
A
  1. (B) In dealing with the area of state economic regulation enacted so as to affect the health or safety of the citizenry, the Court will not strike down the state legislation if the benefit to health and safety outweighs the burden imposed on interstate commerce. The challenged legislation must pass a commerce clause test of “reasonableness” that is stricter than that used for due process and equal protection cases. Nowak, p. 271. Following the holding in South Carolina v. Barnwell, 303 U.S.177 (1938), where the Court upheld a width restriction on trucks using South Carolina state highways due to the safety concern arising from the state’s narrow roads, the city’s parking ban on one side of the highway due to the narrowness of the highway as it passes through the business district promotes a safety interest that is reasonable, and it will be upheld. Choice (A) is incorrect because the 11th Amendment generally prohibits people from suing the state for monetary damages. Here, the lounge is not suing the city for monetary damages. Choice (C) is incorrect. Under equal protection clause scrutiny, strict scrutiny will be applied to those laws that create suspect classes: race, ethnicity, nationality, and religion. Intermediate review will be applied to legal classifications based on gender and illegitimacy. All other legal classifications are subject to rational review, which is relatively easy for the state to pass. Here, the lounge does not constitute either a suspect class or quasi-suspect class. Thus, the city’s discrimination against the lounge, even if true, would only merit rational review under equal protection clause analysis; the city will easily pass such rational review. Choice (D) is incorrect. Under rational basis review, even something as silly as a coin flip may suffice under some circumstances where everything else is pretty much equal.
40
Q
  1. An industrial town with a population of 100,000, is located on the north side of a river. On the south side of the river is situated a rural community with a population of 40,000. For many years, various civic groups have urged that both communities merge into one township with a single governmental body. Independent studies have indicated that such a merger would result in an enormous tax savings to the residents of both municipalities by eliminating the duplication of services. On one previous occasion, proponents of the merger pian succeeded in having the proposal appear as an election referendum in each community. Although the merger referendum passed in the industrial town by a sizable margin, the voters of the rural community rejected the measure, fearing the combined government would be dominated and controlled by its neighbor’s larger representation.
    In order to alleviate the concern of the rural community’s voters regarding under representation in a merged governmental system, the respective city councils of both municipalities appointed a steering committee to formulate a new proposal. Accordingly, the steering committee devised a merger scheme wherein the city council of the united city, would consist of eight members. Within this proposed new system of government, each former municipality would be divided into four districts. With respect to the industrial town, each district would consist of 25,000 persons and each would have one elected city council member. By the same token, the rural community would be divided into four elective districts, each containing 10,000 residents. One city council member would be elected from each of these districts as well. The mayor would be elected at large by a popular vote of all residents in the newly created eight districts.
    Before this merger proposal was placed on the ballot, the state attorney general issued an advisory opinion stating that the measure did not, in her opinion, violate any statutory or constitutional provisions. Thereafter, the proposal was placed on the ballot and was overwhelmingly passed by the voters in both municipalities. After the election, but before the merger had officially been carried out, two taxpayers from the industrial town initiated suit to enjoin the unification, attacking the constitutionality of the disproportionate representative districts.

Which of the following represents the plaintiffs’ strongest constitutional argument in support of their action?
(A) The plaintiffs and other residents of the industrial town have been denied the equal protection of the law.
(B) The plaintiffs and other residents of the industrial town have been denied the due process of the law.
(C) The plaintiffs and other residents of the industrial town have been denied the privileges and immunities of citizenship as guaranteed by Article IV, Section 2.
(D) The merged city would not constitute a republican form of government.

A
  1. (A) State control over the right to vote is not expressly limited by the federal Constitution. However, any inequality in allocating the right to vote based on using electoral districts established on criteria other than street population dilutes the “one man, one vote” principle and will be subject to strict scrutiny review by the Court. In Reynolds v. Sims, 377 U.S. 533 (1964), justice Warren formulated the one person, one vote rule: “If a State should provide that the votes of citizens in one part of the State should be given two times, or five times, or 10 times the weight of votes of citizens in another part of the State, it could hardly be contended that the right to vote of those residing in the disfavored areas had not been effectively diluted. The Equal Protection Clause requires that the seats in both houses of a bicameral state legislature must be apportioned on a population basis.” Nowak, Constitutional Law, p. 754. Choice (B) is incorrect. It seems unlikely that due process was denied to the plaintiffs given that the members of their city council voiced their concerns with the members of the other city council and a steering committee was formulated to represent the collective interests of both cities. Choice (C) is incorrect. The privileges and immunities clause of Article IV has been interpreted by the Supreme Court to prohibit discrimination by a city, even if the city’s policies discriminate against other in-staters, if the effect of the policy is to discriminate against out-ofstaters [United Building & Construction Trades Council v. Mayor and Council of Camden, 465 U.S. 208 (1984)]. However, in this instance, the aggrieved party consists entirely of in-staters. Choice (D) is incorrect. The Supreme Court faced a challenge to the use of statewide initiatives in Pacific States Telephone and Telegraph Company v. Oregon, 223 U.S. 118 (1912). In that case, the Court held that challenges to a state’s republican character are nonjusticiable political questions, and that the decision of whether a state is “republican” in conformance with the guarantee clause may be decided only by Congress, and the Court would not get involved.
41
Q
  1. Before a proposal to merge two towns was placed on a ballot, the state attorney general issued an advisory opinion stating that the measure did not, in her opinion, violate any statutory or constitutional provisions. Thereafter, the proposal was placed on the ballot and was overwhelmingly passed by the voters in both towns. After the election, but before the merger had officially been carried out, two taxpayers from one of the towns initiated suit to enjoin the unification, attacking the constitutionality of the proposal. The suit reached the state supreme court and was found to be constitutional under both state and federal law.
    The plaintiffs now file a motion seeking to have this
    case reviewed by the U.S. Supreme Court. The Court may
    (A) not hear the case, because it was decided on independent state grounds.
    (B) not hear the case, but may have it remanded to federal district court.
    (C) hear the federal issues involved, but decline to rule on the state issue.
    (D) rely on the advisory opinion rendered by the state attorney general and not hear the case on its merits.
A
  1. (A) Where a state court clearly holds that a state law violates (or does not violate) a state law or provision of the state constitution, that decision will be an adequate and independent state ground. This is true regardless of whether or not the opinion also decides that the state law violates a federal law as well. Indeed, where a state court holds that a state law violates both the state and federal constitutions, the doctrine of adequate state grounds will apply. This is precisely the case addressed by the question, because it is evident that the state court found the proposal constitutional under both state and federal law. Without more information (such as, for example, evidence that the opinion is based upon federal interpretation of a similar federal law), there is no reason to assume anything beyond the fact that the Supreme Court will decline the case, because the decision of the state court rests on an independent and adequate state ground. Choice (A) is thus correct. Choice (B) is incorrect, as it confuses the situation presented here with one where it is unclear whether the state made their decision based upon state or federal interpretations of statutes. In that situation, the Supreme Court may take the case, although it also has the power in that situation to dismiss or remand the case for clarification from the state court (not a federal district court). Choice (C) is incorrect because the Court will not hear the case exclusively on federal issues, because it will decline the case altogether, in light of the adequate state ground.
42
Q
  1. A federal grand jury was convened to investigate the alleged bribery by a foreign government of a special adviser to the President. The grand jury was probing the foreign government’s efforts to obtain delivery of transport planes, which were embargoed by the State Department. The Justice Department was trying to ascertain whether the foreign government had offered bribes to the special adviser and other members of the U.S. government in order to secure delivery of the transport planes.
    The special adviser testified before the grand jury that both he and the President had several conferences with the foreign government’s ambassador to the United States. He stated that during these meetings, they discussed problems in their region in general. He denied, however, any involvement in the foreign government’s efforts to secure delivery of the transport planes.
    Two weeks after the special adviser testified, the grand jury returned an indictment, charging him and two other members of the President’s staff with conspiracy to commit bribery and conspiracy to defraud the U.S. government. A special prosecutor was then appointed by the Justice Department to prepare the government’s case.
    Upon motion by the special prosecutor, a subpoena duces tecum was issued directing the President to produce the minutes of his meetings with the special adviser and ambassador of the foreign government. The special prosecutor was able to determine the exact dates of the meetings through appointment records that had been previously subpoenaed. The President released several edited transcripts of these conversations. On the same day, the President’s counsel filed a motion to quash the subpoena duces tecum, claiming an absolute executive privilege.
    Which of the following is the most accurate statement with regard to the President’s claim of executive privilege?
    (A) Under the separation of powers doctrine, the federal judiciary is without authority to review an assertion of executive privilege by the President.
    (B) The need for the confidentiality of high-level communications will sustain an absolute unqualified presidential privilege of immunity from judicial process on all occasions.
    (C) Article III does not vest the federal courts with power to resolve an intrabranch dispute.
    (D) Article II does not vest the President with an absolute, unqualified privilege to withhold evidence from a criminal prosecution.
A
  1. (D) Article II does not vest the President with an absolute, unqualified executive pri vilege as against a subpoena essential to enforcement of criminal statutes. To vest the President with such powers would upset the separation of powers doctrine and gravely impair the role of the courts under Article III. The Supreme Court, in United States v. Nixon, 418 U.S. 683 (1974), held that the courts have the power of judicial review and are the final arbiter ofa claim of executive privilege. Moreover, the Court concluded that the executive privilege may not be asserted as to subpoenaed materials sought for use in a criminal trial, since the Constitution requires the courts to adhere to the due process of law. Choice (A) is incorrect. In United States v. Nixon, 418 U.S. 683 (1974), the Supreme Court held that courts have ultimate authority for determining whether an invocation of executive privilege is justified. Otherwise, the executive branch would undermine the judiciary and thus violate the principle of the separation of powers. Choice (B) is incorrect. While confidentiality is obviously important for the executive branch, there is no unqualified immunity under the Constitution. In United States v. Nixon, 418 U.s. 683 (1974), the Supreme Court held that courts have ultimate authority for determining whether an invocation of executive privilege is justified. Otherwise, the executive branch would undermine the judiciary and thus violate the principle of the separation of powers. Finally, choice (C) is incorrect. Article Ill says: “The [federal] judicial Power shall extend to all Cases, in Law and Equity, arising under this Constitution, the Laws of the United States, and Treaties made, or which shall be made, under their Authority; to all Cases affecting Ambassadors, other public Ministers and Consuls.” Ambassadors and public ministers serve in the executive branch; the President himself is one such example. Thus, the federal courts may review the President.
43
Q
  1. Pursuant to a newly enacted statute, Congress created a nine-member agency empowered to promulgate rules governing air quality standards for the nation. In accordance with the statute, the President was authorized to appoint a majority of six members to the agency, and the other three positions were to be filled by the Senate.
    The nine members were duly appointed to the agency, and all appointees were subsequently approved in confirmation hearings. Thereafter, the agency issued the following regulations:
    (a) Requiring each motor vehicle operating in the United States to be equipped with a specified air/fuel control device;
    (b) Requiring each gas or oil furnace located in the United States to be fitted with a specified device to reduce emissions;
    (c) Requiring each State to establish and maintain a program under which each vehicle and each furnace shall be tested annually for compliance with federal emissions standards.
    Which of the following arguments would provide the strongest constitutional grounds against the authority of the statute?
    (A) The President does not have the constitutional power to appoint a majority of members to an administrative agency.
    (B) Congress does not have the executive authority to appoint members to an administrative agency.
    (C) An administrative agency does not have the constitutional authority to promulgate regulations that unduly burden interstate commerce.
    (D) An administrative agency does not have the constitutional authority to require states to supervise federal regulatory guidelines such as those enumerated in Subsection (c).
A
  1. (B) The power of the President to appoint and remove officers of the United States stems in part from express provisions of the Constitution and in part from the implications of express grants of power. Article II, Section 2, Clause 2 establishes in the President the power to appoint officers of the United States; it also provides that Congress may vest the appointment of inferior officers in either the President alone, in the courts, or in the heads of departments. At no time, however, may the legislative branch exercise executive authority by retaining the power to appoint those who will execute its laws. Thus, in Buckley v. Valeo, 424 U.S. 1 (1976), the U.S. Supreme Court held that Congress had violated Article II in allowing the President pro tern of the Senate and the Speaker of the House to appoint a majority of the voting members of the Federal Election Commission. Choice (A) is incorrect. Article II, Section 2 says that the President “shall nominate, and by and with the Advice and Consent of the Senate, shall appoint Ambassadors, other public Ministers and Consuls, Judges of the supreme Court, and all other Officers of the United States, whose Appointments are not herein otherwise provided for, and which shall be established by Law: but the Congress may by Law vest the Appointment of such inferior Officers, as they think proper, in the President alone, in the Courts of Law, or in the Heads of Departments.” Choice (C) is incorrect. This language of “undue burden” in regard to the commerce clause applies only to states and other local governments when their actions are analyzed under the dormant commerce clause. An administrative agency, acting as the federal government, can regulate commerce under the commerce clause, as long as the agency meets the requirements under Article I, Section 8 of the commerce clause. Choice (D) is incorrect. An administrative agency does have constitutional authority to require states to supervise federal guidelines regarding commerce, as long as the federal agency does not violate the state’s 10th Amendment rights against the federal government conscripting the state’s governmental resources [New York v. United States, 505 U.S. 144 (1992)].
44
Q
  1. Voters in a city passed a referendum legalizing gambling in a certain section of the city. The law established a commission, consisting of five individuals, that was in charge of licensing and overseeing the activities of the casinos. The members of the commission were to be appointed by the mayor. Pursuant to his statutory power, the mayor appointed four private citizens and a clergy member to the Commission.
    There is a constitutional challenge to the appointment of the clergy member as violating the establishment clause of the First Amendment. The mayor’s action is
    (A) unconstitutional, because the appointment of a clergy member to the commission fosters excessive governmental entanglement with religion.
    (B) unconstitutional on its face, because members of the commission are vested with enforcement powers.
    (C) constitutional, because the primary effect of appointing only one religious member to the commission does not, per Se, advance or inhibit religion.
    (D) constitutional, because commission membership is an appointive privilege and not an elective right.
A
  1. (C) Another issue commonly tested on the MBE deals with whetherreligious members are precluded from holding government offices. In McDaniel v. Paty, 435 U.S. 618 (1978), the Supreme Court declared unconstitutional a state law that prevented “Minister(s) of the GospeL, or priest(s) of any denomination whatever from serving as delegates to the state constitutional convention.” Interestingly enough, the disqualification of clergy members from legislative office existed in England and was followed by seven of the original states. The Supreme Court, however, found that this history was not decisive and invalidated the state Law. The majority of justices ruled that the free exercise of religion allowed members of the clergy to hold government office. Based on the holding in McDaniel, the appointment of a member of the clergy to a governmental agency or commission would not, perse, be violative of the establishment clause. Choice (A) is incorrect. In Lemon v. Kurtzman, 403 U.S. 602 (1971), the Court found that there was an establishment of religion if there was “excessive entangLement” between religion and government. For example, there was excessive entanglement if the government had to supervise a religious school to ensure that government subsidies for the teachers were not being used to teach religious subjects. Here, the clergy member is being hired to perform an avowedly religious purpose in maintaining good morals. Thus, no such ongoing supervision is required by the government. Choice (B) is incorrect. Under the 10th Amendment, a state—including local city agencies—is vested with police powers to regulate (and enforce) laws regarding health, safety, welfare, and morals. Choice (D) is incorrect. The distinction between privilege and right would not protect the government from violations against the Establishment Clause. If, say, the government gave $1 million to a church to spread religion, the government aid would clearly violate the Establishment Clause, even though it is a privilege, not a right, for the church to receive the money.
45
Q
  1. A state has enacted an abortion statute in an attempt to reconcile the conflicting interests involved when a woman chooses to terminate a pregnancy by abortion. The state’s statute provided that during the first trimester of pregnancy, a woman’s right to choose to terminate the pregnancy was paramount and could not be restricted in any manner. After the first trimester, the right of a woman to obtain an abortion was limited to cases where it was demonstrated by a physician that an abortion was necessary to protect the life or health of the woman seeking the abortion.
    In all likelihood, this abortion statute is
    (A) constitutional, because the state’s statute
    strikes a proper balance between the funda
    mental right of a woman to choose to terminate
    a pregnancy by abortion and the due process
    right to life of the unborn child.
    (B) constitutional, because the state’s statute is
    substantially related to the important state
    interest in protecting the health and life of the
    mother.
    (C) unconstitutional, because the state’s statute
    imposes an undue burden on the right to obtain
    an abortion.
    (D) unconstitutional, because it is
    irrational to impose virtually no
    restrictions on the right to obtain an
    abortion in the first trimester while
    imposing significant restrictions
    on the right to obtain an abortion
    thereafter.
A
  1. (C) With respect to abortions, the trimester test of Roe has been partially overruled by Planned Parenthood of Southeastern Pennsylvania v. Casey, 505 U.S. 833 (1992). As a result of Casey, the state may restrict abortions so long as they do not place “undue burdens” on the woman’s right to choose. Here, the state’s statute does place an “undue burden” on the right to abortion because after the first trimester, a woman can only have an abortion to protect her health and life. Choice (A) is not the best answer because it utilizes the “fundamental” right to privacy language articulated in Roe v. Wade. However, the Court in Casey appeared to reject the Roe view that abortion was a “fundamental” right and restrictions are no longer to be strictlyscrutinized.
    Choice (B) is wrong because the Court applies the undue burden test to abortions, not an intermediate scrutiny standard as applied in gender discrimination. Choice (D) is incorrect. This option captures the general meaning of Planned Parenthood of Southeastern Pennsylvania v. Casey, 505 U.S. 833 (1992). There, the Supreme Court explained that a state is entitled to place more burdens on a woman’s right to an abortion as she progresses in her pregnancy. The rationale was that after viability, the state may seek to prohibit abortion to save the life of the fetus.
46
Q
  1. Congress recently enacted a statute that prohibits racial discrimination in the sale, transfer, or rental of real estate, either privately or publicly.
    Which of the following constitutional provisions would provide the best rationale for the enactment of this federal statute?
    (A) Under Article I, Congress has the power to enact laws that are “necessary and proper” to the general welfare.
    (B) The enforcement provision of Section 2 of the Thirteenth Amendment.
    (C) The enforcement provision of Section 5 of the Fourteenth Amendment.
    (D) The due process clause of the Fifth Amendment.
A
  1. (B) The 13th Amendment is unique in two respects. First, it contains an absolute bar to the existence of slavery or involuntary servitude; there is no requirement of “state action.” Thus, it is applicable to individuals as well as states. Second, like the 14th and 15th Amendments, it contains an enforcement clause enabling Congress to pass all necessary legislation. In this regard, the Court has held that the enforcement provision of the 13th Amendment has extended Section 1982 of the 1866 Civil Rights Act “to insure minorities the freedom to inherit, purchase, lease, sell, hold and convey real and personal property.” Most important, the 13th Amendment has been construed to prohibit both public and private racial discrimination in housing. Choice (A) is a potentially good option insofar as the necessary and proper clause can be invoked to further Congress’s powers under the commerce clause. But choice (B) is a better option. That is because the 13th Amendment provides an absolute bar to the existence of slavery or involuntary servitude, whether committed by the government or private individuals, and there is a clause in the 13th Amendment that permits Congress to pass all appropriate legislation to enforce the 13th Amendment. Choice (C) is incorrect. Remember that the 14th Amendment only protects a person from the actions of local governments, not private actors. Thus, the 14th Amendment will not help Congress to prevent racial discrimination by private individuals. Finally, choice (D) is incorrect. Like the 14th Amendment, the 5th Amendment only protects a person from discrimination by the federal government. Thus, the 5th Amendment will not help Congress to prevent racial discrimination by private individuals.
47
Q
  1. A staff assistant for a state agency was convicted in federal court of taking bribes from a foreign government for the purpose of influencing an upcoming vote on a waterworks bill. He was sentenced to probation. The staff assistant had served in the agency long enough to become fully qualified for his pension upon retirement under the terms of an agreement between the pensions board and the union. The staff assistant retired and immediately started receiving monthly pension checks.
    Subsequently, the governor signed into law an act, which provided in part:
    “Section 8. Any member of a state agency’s staff who is convicted of… bribery … shall not be
    entitled to receive any retirement or other benefit or payment of any kind from the state … Such conviction shall be considered a breach of the staff member’s employment contract.”
    The staff assistant received a letter from the state which stated that pursuant to this new act the state is immediately discontinuing pension benefit payments to you on account of your bribery conviction.
    The staff assistant contacted an attorney, who challenged the discontinuance of benefits on the grounds that the new law was unconstitutional.
    To counter one of the attorney’s possible arguments regarding the unconstitutionality of Section 8 of the act, the state’s best rebuttal would be that
    (A) the staff assistant was afforded an opportunity to express his views about the new legislation at public hearings, prior to the enactment of the statute.
    (B) deprivation of pension benefits is not cruel and unusual punishment.
    (C) a letter sent through ordinary mail is sufficient notice to satisfj due process for discontinuation of pension benefits.
    (D) it is implicit that one of the conditions of the state’s contract of employment with a state agency staff member is that he shall not engage in bribery.
A
  1. (D) This question requires a two-step analysis to select the best alternative. First, you must determine the constitutional issue involved and then, second, apply the appropriate constitutional principle to the question asked, i.e., the state’s best rebuttal to a constitutional challenge to the pension forfeiture statute. Choice (D) is correct since the argument concerning a condition of employment contract with a agency staff member affects the Contract Clause of the Constitution. The staff member’s attorney, could validly challenge the constitutionality of the statute, alleging unconstitutional impairment of the obligation to contract. The pension forfeiture statute would be violative of the Contract Clause since under the circumstances, the staff member has satisfied the conditions of retirement eligibility (he became fully qualified for his pension on retirement, as stated in the facts). His retirement pay has ripened into a full contractual obligation and has become a vested right. Therefore, the pension forfeiture statute would be an unconstitutional impairment of his vested right to receive retirement benefits (his pension). Choices (A) and (B) are irrelevant to the constitutional issues involved. Answer (C) is inapplicable, since notice is not an issue here.
48
Q
  1. A state’s attorney was convicted in federal court of taking bribes to drop charges against an oil company. He was sentenced to probation. The state’s attorney had served long enough to become fully qualified for his pension upon retirement. The state’s attorney retired and immediately started receiving monthly pension checks.
    Subsequently, the governor signed into law a statute which provided in part:
    “Section 8. Any member of the state attorneys staff who is convicted of… bribery … shall not be
    entitled to receive any retirement or other benefit or payment of any kind from the state . Such conviction shall be considered a breach of the staff member’s employment contract.”
    Following the enactment of the statute, the state’s attorney received a letter from the state which stated they are immediately discontinuing pension benefit payments to him on account of his bribery conviction.
    The state’s attorney contacted an attorney, who challenged the discontinuance of benefits on the grounds that the new law was unconstitutional.
    In order to reinstate the state attorney’s pension on the grounds that the statute is unconstitutional, the attorney’s strongest argument would be that
    (A) the state attorney was retroactively punished.
    (B) the statute is an ex post facto law
    (C) the supremacy clause invalidates the state law, because there is federal legislation regulating pension and profit sharing plans.
    (D) the statute has a chilling effect
    on legislators’ rights to freely discuss pending bills with members of their staff.
A
  1. (B) The two ex post facto clauses in the U.S. Constitution prohibit Congress and the state legislatures from enacting laws that have a retroactive effect. The statute is an example of an expost facto law that renders an act punishable in a manner in which it was not punishable when committed under the facts presented (since the state’s attorney qualified for pension benefits before the pension forfeiture statute was enacted). Thus, the statute, which denied the state’s attorney a pension because of the bribery conviction during his employment applied retroactively to the state’s attorney. Hence, the pension forfeiture statute would be held violative of the expost facto clauses of the U.S. Constitution. Be aware that although choice (A) is a correct statement of fact, choice (B) is preferred, because it is the correct statement of law. When you are confronted with correct statements of fact and law, the latter is the preferred alternative. Choice (C) is incorrect because even if this were factually true, there would not necessarily be any violation of the Supremacy Clause unless the federal and state laws conflicted with each other or the state legislation regulated in a field that was reserved exclusively for the federal government. Neither appears to be the case. Choice (D) is incorrect. Usually, statutes are deemed to violate the 1st Amendment if they are so ambiguous as to scare people—”chill them”—into forgoing their speech. That does not appear to be the situation in our example.
49
Q
  1. A state trooper was convicted in federal court in 2008 of taking bribes. He was sentenced to probation. He had served as a state trooper since 1978, and in 1998, he became fully qualified for his pension upon retirement. In 2006, the trooper retired and immediately started receiving monthly state pension checks.
    In 2009, the governor of the state signed into law a statute, which provided in part:
    “Section 8. Any member of the state law enforcement staff… who is convicted of… bribery
    shall not be entitled to receive any retirement or other benefit or payment of any kind from the state
    Such conviction shall be considered a breach of the staff member’s employment contract
    Section 12. The provisions of this Act shall be retroactive to December 31, 1975.
    Section 14. This Act shall take effect immediately.”
    Following the enactment of the statute, the state trooper received a letter from the state which stated that pursuant to the statute, the state is immediately discontinuing pension benefit payments to you on account of your 2008 bribery conviction.”
    The trooper contacted an attorney, who challenged the discontinuance of benefits on the grounds that the new law was unconstitutional.
    Assume that the state’s highest court holds the statute constitutional. The trooper might still be able to eventually have his pension reinstated if
    (A) he exercises his constitutional right to discretionary review in the U.S. Supreme Court.
    (B) he receives a presidential pardon for his bribery offense.
    (C) he can show that he was convicted before the effective date of the
    statute.
    (D) he can show that the statute violates the dormant commerce clause.
A
  1. (B) Of the four choices listed, only (B) provides a correct statement regarding the possibility of the reinstatement of the trooper’s pension. Article II empowers the President to grant reprieves and pardons for offenses against the United States. The President may pardon absolutely or conditionally commute sentences, and remit fines, penalties, and forfeitures. The facts state that the trooper was convicted of bribery in federal court. In all likelihood, therefore, he was being prosecuted for a federal criminal offense. Choice (A) is incorrect since he does not have a constitutional right to discretionary review in the U.S. Supreme Court. Such right of appeal to the U.S. Supreme Court is not a constitutional right, but a statutory right; Congress has enacted legislation regarding the appellate review of the Supreme Court. Choice (C) is incorrect because even if this were factually true, it would not make any legal difference because the State is not punishing him and, thus, the State is not violating the prohibition against ex post facto laws in Article I, Section 9. Choice (D) is wrong because there is nothing on these facts to suggest that the statute places a burden on interstate commerce.
50
Q
  1. Within the last two years, the number of cases coming before the U.S. Supreme Court has quadrupled. Because of this increased work load, the Court has complained that it is unable to properly review all of its cases. As a consequence, Congress formed a committee to conduct a study on improving the functioning and operation of the Court. Based on the committee’s recommendations, Congress enacted a statute dividing the Court into two panels. One panel would be assigned to handle criminal cases exclusively, while the other panel would handle all non-criminal matters. Each panel would be composed of four associate justices and a chief justice. According to the new law, the decisions of each panel would be final and not reviewable by any other court or judiciary.
    Which of the following is the strongest argument against the constitutionality of this federal statute?
    (A) The statute contravenes the requirement in the Constitution that there be one Supreme Court.
    (B) The statute does not fall within the enumerated powers of Congress and is not necessary and proper for the effectuation of those powers.
    (C) Based on the doctrine ofjudicial supremacy, Congress does not have authority to legislate with respect to the jurisdiction of the Supreme Court.
    (D) Based on the separate sovereignty doctrine, Congress does not have authority to interfere with the procedural machinery of the Supreme Court.
A
  1. (A) The Supreme Court is the only federal court created directly by the Constitution. Article III, Section 1 mandates that judicial power be vested in “one Supreme Court.” See Nowak, Constitutional Law, p. 24. Therefore, the Congressional statute to divide the Supreme Court into two panels is unconstitutional since it contravenesArticle III of the Constitution. Choice (B) is factually true, but its rationale is not as precise as choice (A). Students should be aware that Article III, Section 1 vests judicial power as to the inferior courts—including federal district courts and courts of appeal only “as the Congress may from time to time ordain and establish.” This plenary power of Congress includes not only the establishment of such courts, but also the authorization of their jurisdiction, the power to remove jurisdiction of certain classes of cases, and the power to terminate the courts’ existence. Article Ill judges are appointed for life, and their compensation may not be diminished during their term in office. Choice (C) is incorrect. The concept of “judicial supremacy” finds no explicit mention in the Constitution. In Marbury v. Madison, 5 U.S. (Cranch 1) 137 (1803), the Court stated that it is emphatically the province of the judicial branch to say what the law means. Yet this, in itself, does not explain why the congressional statute in the example is unconstitutional. A better explanation is that Article Ill, Section 1 mandates that judicial power be vested in “one Supreme Court.” Therefore, the Congressional statute to divide the Supreme Court into two panels is unconstitutional since it contravenes Article III of the Constitution. Choice (D) is incorrect. The separate sovereignty doctrine means that different powers of jurisdiction belong to different states and that the federal government’s sovereignty differs from that of the state governments. For example, a person could be prosecuted forthe same crime under both federal and state laws without violating the constitutional prohibition against double jeopardy. Congress and the Supreme Court are governed less by “separate sovereignty doctrine” than by the principle of the separation of powers.
51
Q
  1. The President appointed a delegation to enter into negotiations with representatives of a foreign government to study the problem of preventing the extinction of certain species of seals. The delegation’s goal was twofold: to study the problem and to formulate regulations in a bilateral agreement that would protect the endangered species and provide for a permanent commission that would continually monitor enforcement of the proposed regulations. After compiling their findings and drafting the necessary regulations, the President and the leader of the foreign government entered into a treaty to form a permanent commission to oversee the problem and to grant it the necessary enforcement powers.
    The validity of this treaty would most likely be upheld under which of the following principles?
    (A) The presidential power to conduct foreign affairs.
    (B) An ancillary power of the President under his treaty-making power.
    (C) The treaty-making power, but only if the treaty is ratified by two-thirds of the Senate.
    (D) The treaty-making power, but only if the treaty is ratified by a majority in Congress.
A
  1. (C) Under Article II, Section 2, the only constitutional limitation upon the President’s power “to conduct foreign affairs” is with regard to treaties, which become valid only when ratified by two-thirds of the Senate. Choice (A) is not a bad answer, but a better option is available in Choice (C), given that there is explicit constitutional support in Article II, Section 2 for the President’s power to make treaties, whereas there is no explicit mention of the President’s power to conduct “foreign affairs”. Besides, the President must, under Article II, Section 2, receive ratification by two- thirds of the Senate to complete a treaty; the president cannot rely on his “foreign affairs” powers to do that. Choice (B) is incorrect. The President cannot rely on his ancillary powers under his broader treaty-making power. He does, however, have to rely on the Senate, as Article II, Section 2 requires that he receive ratification by two-thirds of the Senate to complete a treaty. Finally, choice (D) is incorrect. Article II, Section 2 requires that the President receive ratification by two-thirds of the Senate to complete a treaty.
52
Q
  1. The President appointed a delegation to enter into negotiations with representatives of a foreign government to study the problem of preventing the extinction of certain species of rabbits. The delegation’s goal was twofold: to study the problem and to formulate regulations in a bilateral agreement that would protect the endangered species and provide for a permanent commission that would continually monitor enforcement of the proposed regulations. After compiling their findings and drafting the necessary regulations, the President and the leader of the foreign government entered into a treaty to form a permanent commission to oversee the problem and to grant it the necessary enforcement powers.
    Assume that after the treaty goes into effect, a state legislature enacts a statute that provides that “any licensed rabbit-hunter in the state and its surrounding environs may increase his monthly catch of rabbits from 10 to 15 in each of the specified months of the authorized rabbit-hunting season from the first day of October until the last day of February.” If challenged, the enactment of the aforementioned statute would most likely be declared
    (A) constitutional, because the regulation of hunting is within the area of state action.
    (B) constitutional, because the enactment falls within the Tenth Amendment’s reserved
    powers.
    (C) unconstitutional, because it violates the commerce clause.
    (D) unconstitutional, because all treaties are the supreme law of the land.
A
  1. (D) UnderArticleVi, Paragraph 2, alL treaties “which shall be made underthe authority
    of the U.S.” are the “Supreme law of the land” (along with the Constitution itself
    and laws of the U.S. made in pursuance thereof). As a consequence, it is clear that
    any state action in conflict with a treaty is invalid. Choice (A) is incorrect. Even if
    there is state action in this example (and there does appear to be), that alone would
    not make the statute constitutional. In fact, state action simply means that there is
    governmental involvement. Choice (B) is incorrect. As a matter of law, this is true
    because of the state’s police powers under the 10th Amendment to regulate health,
    safety, welfare, and morals. However, in this instance, the state statute conflicts
    with the treaty. Under Article VI, Paragraph 2, aLl treaties “which shall be made
    under the authority of the U.S.” are the “Supreme law of the land” (along with the
    Constitution itself and laws of the U.S. made in pursuance thereof). As a conse
    quence, it is clear that any state action in conflict with a treaty is invalid. Choice (C)
    is not a bad answer, since the state’s actions would seem to have some effect on
    commerce between the state and the foreign government, and under Article I, Sec
    tion 8, only Congress has the authority to regulate commerce with foreign nations.
    But a better answer is available in choice (D) because there is explicit and direct
    support for it in the Constitution.
53
Q
  1. During a three-month period, a city was stunned by a series of mysterious deaths, which claimed the lives of 20 people. Although all the victims had apparently died from poisoning, the police were in a quandary as to who was responsible for the killings. There was finally a breakthrough in the investigation when a police toxicologist determined that all the victims had died after eating poisoned apples which had been purchased at a local market. The apples had all been contaminated with a pesticide. The police then received a letter from a person who claimed responsibility for the poisonings. The letter stated that the killings were in retaliation for the city’s new policy of prosecuting toxic polluters.
    Acting upon an anonymous tip, the police arrested the owner of a pest control company engaged in the manufacture of pesticides, and charged him with 20 counts of murder. Thereafter, the city’s largest newspaper ran a series of articles on the killings and referred to the owner as the pesticide poisoner. After the preliminary hearing, the state trial judge issued an exparte injunction against the newspaper prohibiting it from publishing any news during the trial that might be prejudicial to the owner. The newspaper appealed.
    In light of the U.S. Supreme Court cases to date, the state appellate court should
    (A) dissolve the injunction, because a news story about a matter of public interest is absolutely privileged.
    (B) dissolve the injunction, because it is an impermissible prior restraint on the freedom of the press.
    (C) uphold the injunction, because the inference of guilt in any prejudicial coverage would deny the defendant his constitutional right of a fair trial.
    (D) uphold the injunction, because reference to the defendant by the name pesticide poisoner would be inflammatory and prejudicial.
A
  1. (A) or (B) According to Justice Berger, “the compatibility of a commitment to an uninhibited, robust, and wide-open discussion of public issues in a free press with a commitment to a criminal process in which the conclusions to be reached in a case will be induced only by evidence and argument in open court has been the subject of long standing debate.” See Nebraska Press Association v. Stuart, 427 U.S. 539 (1976). In this area, the rights of the press often conflict with the rights of the accused. AccordingLy, the Supreme Court offered a qualified response to this question when it invalidated a Nebraska District Court “gag order” that prohibited the press from the publication of certain implicative evidence pertaining to a murder suspect until the jury selection process was completed. Based on the Nebraska decision, choices (A) and (B) would both be arguably correct. Choices (C) and CD) are incorrect because these would not necessarily prejudice the defendant’s right to a fair trial because it does not suggest a connection between the owner and the killings.
54
Q
  1. A defendant was charged with murder. The killing took place on February 1. At the time that the crime occurred, this jurisdiction required a unanimous verdict of all 12 jury members for a conviction on a capital offense. On November 1, the state legislature enacted a new law requiring a majority vote of seven of 12 jurors to convict on a capital offense. The defendant’s trial began on December 1. He was subsequently convicted of murder by an eight- to-four vote. Following the trial, the defendant’s attorney filed a motion to set aside the verdict.
    Which of the following would provide the strongest constitutional grounds to overturn the verdict?
    (A) The ex post facto clause.
    (B) The contracts clause.
    (C) The due process clause of the Fourteenth Amendment.
    (D) The Sixth Amendment right to a fair trial.
A
  1. (A) The ex post facto clause forbids both the states and the federal government from enacting retroactive criminal laws. The most common sort of ex post facto law is one that creates a new crime and applies it retroactively to conduct not criminal at the time it was committed. In addition, the ex post facto clause prohibits the retroactive application of an increase in the punishment for a crime that carried a lesser penalty when committed. Another aspect of the ex post facto prohibition is concern with retroactive changes in evidence and procedure that operate to the disadvantage of the criminal defendant by making conviction easier. Thus, a statute that changes the burden of proof on the prosecution from the usual rule of beyond a reasonable doubt to one of the preponderance of the evidence is ex post facto if retroactive. By analogy, in this hypo we have a situation where the legislature changed the unanimity jury verdict requirement for capital offenses afterthe defendant was arrested and charged with murder. As such, choice (A) is correct. Choice (B) is incorrect. Article I, Section 10, the so-called “Contracts Clause,” states that no state shall pass any “Law impairing the Obligation of Contracts.” There are no contracts at issue in this example. Choice (C) is incorrect because the defendant wilL receive due process of law insofar as he will be given a trial and afforded the protections thereof. Choice (D) is not a bad answer, but the defendant will receive a fair trial insofar as he has been guaranteed a jury as required by Article III, Section 2.
55
Q
  1. The Pentagon has recently released a civil defense plan in the event of nuclear war. According to the Pentagon’s study, certain essential citizens would be evacuated once it was determined that a nuclear war was imminent. Essential citizens would include scientists, carpenters, and the young. The study also recommended that certain non-essential citizens such as the elderly, the infirm, and persons in penal institutions not be evacuated since their future contributions would be less important in the rebuilding of the country following a nuclear war.
    An employee of the Pentagon, was instructed to conduct a public opinion survey regarding the controversial plan. Pentagon officials directed the employee to interview citizens in a door-to- door canvass to determine public opinion for the civil defense plan. After the employee conducted his door-to-door interview canvassing, he was prosecuted for not obtaining prior consent of the citizens he interviewed.
    The employee’s strongest argument is that the prosecution
    (A) violates his right to free speech.
    (B) violates the intergovernmental immunity of a federal employee.
    (C) deprives him of his employment interest without due process.
    (D) impairs the obligation of his employment contract.
A
  1. (B) Although a city ordinance may prohibit the business practice of soliciting magazine
    subscriptions door-to-door without prior invitation of the homeowner, in Beard v.
    Alexandria, 341 U.S. 622 (1951), the Court specifically relied on the commercial
    nature of the transactions in question. On the other hand, in Martin v. Struthers,
    319 U.S. 141 (1943), the Court held an ordinance invalid that forbade any person
    to knock on doors, ring doorbells, or otherwise summon any residents to the door
    as violative of the freedom of speech and press. In this regard, the substantive guar
    antee of due process requires that legislation have a rational relationship to the
    Legitimate ends of government. If a law does not have such a relationship, it would
    be an unconstitutional deprivation of liberty as to those persons affected. Here,
    the employee’s strongest argument is that the prosecution violates the intergov
    ernmental immunity of a federal employee. Note that the employee was performing
    essentiaLly a governmental, not a proprietary, function (in the door-to-door canvass
    ing). Thus, choice (B) is the best answer. Choice (A) is incorrect. Although a city
    ordinance may prohibit the business practice of soliciting magazine subscriptions
    door-to-door without prior invitation of the homeowner, in Beard v. Alexandria, 341
    U.S. 622 (1951), the Court specifically relied on the commercial nature of the trans
    actions in question. On the other hand, in Martin v. Struthers, 319 U.S. 141 (1943),
    the Court held an ordinance invalid that forbade any person to knock on doors, ring
    doorbells, or otherwise summon any residents to the door as violative of the free
    dom of speech and press. But choice (B) is the best answer because the employee
    was not exercising his free speech rights as a private citizen but as an employee of
    the federal government. Choice (C) is incorrect, the employee was working for the
    federal government and, thus, was performing a governmental, not a proprietary,
    function. Accordingly, the due process clause is not the best answer. FinalLy, choice
    (D) is incorrect. Article I, Section 10, the so-called “Contracts CLause,” states that no state shall pass any “Law impairing the Obligation of Contracts.” However, the prosecution does not impair the employee’s obligation because he could have conducted the survey without going door-to-door.
56
Q
  1. A state is concerned with the increase in teenage use of alcohol. In an effort to decrease exposure to alcohol, which poses harmful health risk, the state legislature has enacted a statute to restrict various methods of advertising by alcohol manufacturers. One of the provisions of the law states that advertising of alcohol prices is not permitted except by placement of a sticker on the bottle or container. An alcohol company who was a major distributor of alcohol in the state, claims the advertising restriction violates its constitutional rights protected by the First and Fourteenth Amendments.
    If the alcohol company files suit challenging the validity of the state statute, the court should rule the statute
    (A) constitutional, because the state law is rationally related to the health and safety of the state’s citizens.
    (B) constitutional, because the restriction on commercial speech directly advances a substantial government interest.
    (C) unconstitutional, because the regulation on commercial speech is not necessary to further an important government interest.
    (D) unconstitutional, because the state could achieve its objective by a less restrictive means.
A
  1. (D) Quite often, Multistate Constitutional Law questions are based upon case precedent. This question, for example, is based upon the ruling in 44 Liquormart v. Rhode Island, 517 U.S. 484 (1996), in which a Rhode Island statute prohibited all advertising of liquor prices, except for price tags displayed with the merchandise. The Supreme Court invalidated the law because regulations of commercial speech must be “narrowly tailored” and should be no more extensive than is necessary. In both 44 Liquormartand in this question, the state is attempting to prohibit the dissemination of truthful, nonmisleading advertising. Choice (D) is correct because a state will not be permitted to completely ban commercial advertising but must use a means narrowly tailored to achieve the desired objective. Choice (A) is incorrect. Commercial speech, which is what we have in our example, may not get as much protection as political speech, but it may be entitled to more than you might think. Under 44 Liquormart v. Rhode Island, 517 U.S. 484 (1996), the Court rejected a state statute that prohibited all advertising of liquor prices, except for price tags displayed with the merchandise. In doing so, the Court subjected the statute to strict scrutiny. The facts in our example look similar to 44 Liquormart insofar as our example also contains a statute that prohibits nearly all advertising of alcohol. But choice (A) suggests in its reference to “rationally related” that the Court will use rational review, not strict scrutiny. Choice (B) is incorrect. Under strict scrutiny, the court will ask whether the statute furthers a compelling government interest (although sometimes the Court will use the language of “substantial” government interest). The statute’s concern for health would certainly seem to be a compelling (or substantial) government interest, but the statute probably will be unable to satisfy the second part of strict scrutiny: are the means chosen “narrowly tailored”? In other words, are there less-restrictive alternatives? Choice (C) is incorrect. The statute’s concern for health would certainly seem to be a compelling (or substantial or important) government interest.
57
Q
  1. An American franchise operates as an importer and distributor of bicycles manufactured by a foreign country, and maintains several warehouses for the wholesale distribution of the bicycles. A warehouse located in a county handles the distribution of bicycles for several states. The bikes and bike tires are shipped separately to the county warehouse. The tires are mingled, stacked, and stored in the warehouse along with various other tires. The bicycles, on the other hand, arrive completely assembled in their shipping crates and remain on the loading docks. The county imposes an ad valorem property tax on the bikes and tires.
    The county’s ad valorem tax may properly be assessed against the
    (A) tires only.
    (B) bicycles only.
    (C) tires and bicycles.
    (D) neither the tires nor the bicycles.
A
  1. (A) Article I, Section 10, Clause 2, of the U.S. Constitution provides that “No State shall
    without the consent of Congress, lay any Imposts or Duties on Imports or Exports,
    except what may be absolutely necessary for executing its Inspection Laws.”
    In Michelin Tire Corp. v. W. L. Wages Tax Comm., 423 U.S. 276 (1976), the U.S.
    Supreme Court held that while tubes in their corrugated shipping cartons were
    immune from ad valorem taxation, the tires lost their status as imports and became
    subject to taxation because they had been mingled with other tires imported in
    bulk, sorted, and arranged for sale. Similarly, in this hypo, the bikes were immune
    from the tax, since they remained “imports” in transit, whereas the tires lost their
    “import status” once they became part of the tire inventory at the distribution ware
    house. Therefore, choices (B), (C), and (D) are aLl incorrect under this analysis.
58
Q
  1. An American company operates as an importer and distributor of guitars manufactured by a foreign company, and maintains several warehouses throughout the U.S. for the wholesale distribution of the guitars. A warehouse located in a city handles the distribution of guitars for several state areas. The guitars and guitar strings are shipped separately to the city warehouse. The guitar strings are mingled, stacked, and stored in the warehouse along with various other guitar strings. The guitars, on the other hand, arrive completely assembled in their shipping crates and remain on the loading docks. The city imposes an ad valorem property tax on the guitars and strings.
    The power of the city to impose an ad valorem tax on the foreign company’s guitars and/or strings would most likely be upheld under
    (A) the commerce clause.
    (B) the reserved power of the states as granted under the Tenth Amendment.
    (C) the necessary and proper clause. (D) the import and export clause.
A
  1. (B) The state’s power to enact such property taxes is derived from the 10th Amendment’s reserved powers. All other choices are incorrect, since they reflect powers of the federal government. Choice (A) is incorrect. Under Article I, Section 8, only Congress has the authority to exercise commerce clause powers. Choice (C) is incorrect. Under Article I, Section 8, only Congress has the authority to exercise powers under the necessary and proper clause. Choice (D) is incorrect. Article I, Section 10 contains the import and export clause. This clause prohibits the states: “No State shall without the consent of Congress, lay any Imposts or Duties on Imports or Exports, except what may be absolutely necessary for executing its Inspection Laws.” The clause, therefore, does not bestow any powers upon the states.
59
Q
  1. A pharmaceutical company manufactured a new contraceptive sponge. The pharmaceutical company initially intended for the contraceptive to be available for purchase without a doctor’s prescription. However, a study by the Food and Drug Administration revealed that the contraceptive might prove harmful to some users with preexisting health problems. As a result, Congress enacted legislation prohibiting the shipment and sale of the contraceptive across state lines to those who do not have a doctor’s prescription.
    This law is probably
    (A) constitutional, because Congress has the power to provide for the general welfare.
    (B) constitutional, because Congress
    has the power to regulate interstate
    commerce.
    (C) unconstitutional, because it deprives the manufacturer a property right without just compensation.
    (D) unconstitutional, because it interferes with the right of privacy of contraceptive users.
A
  1. (B) Under the commerce clause, Congress has the very broad power to regulate interstate commerce. Generally speaking, Congress has the power to regulate any activity (whether carried on in one state or many) that has any appreciable effect—direct or indirect—upon interstate commerce. This is the so-called “affectation doctrine.” Choice (A) is incorrect. Article I, Section 8, reads: “The Congress shall have Power to lay and collect Taxes, Duties, Imposts and Excises, to pay the Debts and provide for the common Defense and General Welfare of the United States.” The Supreme Court has taken the reference to “general welfare” to mean that Congress enjoys great discretion in how it chooses to allocate money for the public [United States v. Butler, 297 U.S. 1 (1936)]. There is no such allocation of money in our example; just a regulation. Choice (C) is incorrect. The 5th Amendment’s takings clause states: “No person. . . shall have property be taken for public use, without just compensation.” In Lucas v. South Carolina Coastal Council, 505 U.S. 1003 (1992), the Court stated that a taking occurs where “regulation denies all economically beneficial or productive use of the land.” Here, the pharmaceutical company can still sell Autonomy to anyone who has a doctor’s prescription. Choice (D) is incorrect. The Supreme Court has stated that there is a fundamental right to contraceptives [Griswoldv. Connecticut, 381 U.S. 479 (1965) and Eisenstadt v. Baird, 405 U.S. 438 (1972)]. For a law to be deemed invalid as violating this fundamental right, there must be a showing that the law infringes on the right. Here, the federal statute does not appear to be an infringement, since anyone with a doctor’s prescription may obtain the contraceptive.
60
Q
  1. A company created a new brand of pies. However, a study by the federal Food and Drug Administration revealed that the pies contain potentially harmful levels of nuts for some consumers with nut allergies. As a result, Congress enacted legislation prohibiting the shipment and sale of the pies across state lines.
    A state has a statute that regulates the shipment and sale of the pies within its territory. In light of the federal legislation prohibiting the shipment and sale of the pies across state lines, the state statute is probably
    (A) constitutional, because it is within the state’s police power.
    (B) constitutional, because Congress may not regulate an economic activity where both buyer and seller reside in the same state.
    (C) unconstitutional, because it affects interstate commerce.
    (D) unconstitutional, because the federal law preempts any conflicting state legislation regarding the sale and shipment of the pies.
A
  1. (A) In this question, the Vermont statute simply regulates the shipment and sale of the pies
    within the state’s borders (or intrastate). As a result, where Congress has not acted,
    the states do have police powers to reguLate any phase of local business (production,
    marketing, sales, etc.) provided that such regulations neither discriminate against nor
    burden interstate commerce [Parker v. Brown, 317 U.S. 341 (1943)] or violate other
    provisions of the Constitution. Choice (B) is incorrect. Congress, using its commerce
    clause powers under Article I, Section 8, may regulate economic activities where both
    buyer and seller reside in the same state,if such activities have a “substantial effect”
    on interstate commerce. [Katzenbach v. McClung, 379 U.S. 641 (1966)]. Choice (C) is
    incorrect. Even if the state regulation affects interstate commerce, the state is entitled
    to pass such Legislation under its 10th Amendment police powers to regulate health,
    safety, weLfare, and morals. However, the state regulation may not impose any undue
    burdens on interstate commerce; in that case, the regulation would be deemed to
    violate the dormant commerce clause of Article I, Section 8. Choice (D) is incorrect
    because there is no conflict between the federal law and the state law. Nor is Congress
    regulating a field, like immigration, that is exclusive to it.
61
Q
  1. A manufacturer created a new brand of fish tackle which they sold throughout the country. Congress now imposes a tax of 14 cents on each set of tackle sold in the United States.
    This tax is
    (A) unconstitutional, because the tackle is sold in interstate commerce.
    (B) unconstitutional, because it interferes with the sovereign right of state governments to engage in intrastate commerce.
    (C) constitutional, because the supremacy clause validates laws enacted by Congress.
    (D) constitutional, because it is within the power of Congress to raise revenue.
A
  1. (D) Article I, Section 8 provides: “The Congress shall have Power to ay and coLLect Taxes, Duties, Imposts and Excises….” As a generaL rule, if Congress has no power to regulate the activity taxed, the validity of the tax ultimately depends on its validity as a revenue-raising measure. Choice (D) is correct because as long as the dominant intent of the taxis revenue raising, it will be upheld even though the tax may have substantial regulatory effect. Choice (A) is incorrect, as Congress’s ability to tax under the tax and spend clause is not contingent upon whether the taxed object is sold interstate. Choice (B) is incorrect. State governments do have police powers under the 10th Amendment to regulate health, safety, welfare, and morals. Such powers also include the right to regulate intrastate commerce. However, Congress, under its tax and spend powers in Article I, Section 8, can levy taxes on commerce that is intrastate. Choice (C) is incorrect. The Supremacy Clause of Article VI states that the “Constitution, and the Laws of the United States which shall be made in Pursuance thereof, and all Treaties made, or which shall be made, under the Authority of the United States, shall be the supreme Law of the Land.” The Supremacy Clause does not contain any substantive powers for Congress; instead, the clause declares that any state law that conflicts with an otherwise valid federal law will be deemed unconstitutional. Congress’s power to tax derives from the Tax and Spend Clause of Article I, Section 8.
62
Q
  1. A company created a drug to lower high blood pressure. The President issues an executive order prohibiting the shipment and sale of the drug within the United States. Under this executive order, federal agents are empowered to prosecute all interstate shippers and sellers of the drug. The President claims that he is acting upon conclusive evidence that the drug causes cervical cancer in laboratory monkeys.
    This executive order is
    (A) valid, because the President has the authority to ensure that laws are faithfully executed.
    (B) valid, because the President has the authority to impose economic regulations unless overruled by Congress.
    (C) invalid, because it is an unauthorized extension of executive power.
    (D) invalid, because the President does not have the power to regulate interstate commerce.
A
  1. (D) Article I, Section 8 grants Congress (not the President) the power to regulate interstate commerce. The President does not have any constitutionally delegated legislative power that is inherently law-making in nature. See Youngstown Sheet& Tube v. Sawyer, 343 U.S. 579 (1952), holding invalid a presidential order directing seizure of steel miLls to prevent a threatened strike. Note that choice (D) is more narrowly correct than choice (C) because it addresses executive intrusion in the area of interstate commerce. Choices (A), (B), and (C) are incorrect. In Youngstown Sheet & Tube v. Sawyer, 343 U.S. 579 (1952), the Court stated that the President does not possess any legislative powers; the President may act only if there is explicit authorization in the Constitution or in a federal statute. Here, the President lacks such authorization and, thus, violates the principle of the separation of powers.
63
Q
  1. A city imposes a municipal excise tax of $200 per year on commercial photographic studios in the city. It also imposes an excise tax of $100 per year on every itinerant commercial photographer for the privilege of using the streets and sidewalks. A credit is allowed against this latter tax for any excise tax paid to the city by the photographer or his employer in respect to the maintenance of a photographic studio in the city.
    In an action by a studio located in the city challenging the constitutionality of the $200 per year excise tax, the court will most likely declare the tax
    (A) constitutional, as a nondiscriminatory license tax.
    (B) constitutional, as within the powers of the state to tax the instruments of interstate commerce.
    (C) unconstitutional, as an undue burden on interstate commerce.
    (D) unconstitutional, as a discriminatory tax on the privilege of doing business within the state.
A
  1. (A) The $200 per year excise tax on the commercial photographic studios operating in the city would be upheld as a valid license tax. Such license taxes (as well as privilege, franchise, and occupation taxes) when applied to local activities—separate from the interstate commerce, of which they are a part—are generally upheld if nondiscriminatory and not unreasonably burdensome in their impact on the interstate commerce involved. Choice (B) is incorrect. Under the commerce clause of Article I, Section 8, only Congress may regulate interstate commerce, including the instruments thereof. A local government lacks constitutional power to regulate such instruments. Choices (C) and (D) are incorrect. Under the dormant commerce clause of Article I, Section 8, local governments like the city may not unduly burden interstate commerce. In our example, the city is not discriminating against out-ofstaters engaged in commerce. Nor does it seem that the city’s benefits from the tax disproportionately hurt out-of-staters.
64
Q
  1. A city imposes a municipal excise tax of $200 per year on commercial artists’ studios in that city. It also imposes an excise tax of $100 per year on every itinerant commercial figure drawer for the privilege of using the streets and sidewalks. A credit is allowed against this latter tax for any excise tax paid to the city by the drawer or his employer in respect to the maintenance of an artist’s studio in the city.
    A gallery, located in a neighboring state, has been sending two itinerant artists into the city. Their practice is to draw a picture of a pedestrian, ask him to order a finished painting, and collect a payment of $2.00. The drawing is sent to a studio, which frames it and mails the framed painting to the customer. The neighboring state does not impose a tax on artists’ studios.
    The gallery challenges the constitutionality of the $100 per year excise tax that is imposed upon its itinerant artists in the city. The court will most likely declare this tax
    (A) constitutional, as a valid ad valorem tax on interstate commerce.
    (B) constitutional, since the tax was levied not on the commercial art business as a whole but on the local activity of drawing pictures.
    (C) unconstitutional, since a state or municipality may not impose a license tax on orders for goods or services to be perfonned in another state.
    (D) unconstitutional, as a discriminatory privilege tax.
A
  1. (B) The $100 per year excise tax that is imposed upon the itinerant artists would be upheLd. In Dunbar-StanleyStudios v. Alabama (1969), a similar fixed fee on transient photographers was upheld as constitutional when applied to an out-of-state firm taking photographs in the taxing state, the rationale being that the tax was levied not on the business as a whole but on a local activity of taking photographs, as opposed to their development and processing. Under this rationa’e, choice (B) is correct, and choice (C) is incorrect. Choice (A) is also incorrect because, underthe commerce clause of Article I, Section 8, only Congress may regulate interstate commerce. Therefore, a state lacks authority to tax interstate commerce. Finally, choice (D) is incorrect. Under the dormant commerce clause of Article I, Section 8, local governments like the city may not unduly burden interstate commerce. In our example, the city is not discriminating against out-of-staters engaged in commerce. Nor does it seem that the city’s benefits from the tax disproportionately hurt out-of-staters.
65
Q
  1. A state imposes a tax on nonresident photographers who operate photographic studios in the state at a rate of 2% of their state-derived income above $12,000. This income tax exempts taxable income earned outside of the state by photographers who live in the state. Moreover, resident-photographers of the state are not taxed on their in-state earned income. A photographer who operates a studio in the state but is a resident of another state, challenges the constitutionality of this tax.
    Which of the following provisions would furnish the most applicable basis for this constitutional challenge?
    (A) The equal protection clause of the Fourteenth Amendment.
    (B) The due process clause of the Fourteenth Amendment.
    (C) The commerce clause.
    (D) The privileges and immunities clause of Article
    Iv.
A
  1. (D) Article IV, Section 2, so far as relevant, reads as follows: “The Citizens of each State shall be entitled to all Privileges and Immunities of Citizens in the several States.” It was designed to ensure to a citizen of State A who ventures into State B the same privileges that the citizens of State B enjoy. In line with this underlying purpose, it was long ago decided that one of the privileges that the clause guarantees to citizens of State A is that of doing business in State B on terms of substantial equality with the citizens of that State. Like many other constitutional provisions, the Privileges and Immunities Clause is not an absolute. It does bar, nevertheless, discrimination against citizens of other states where there is no substantial reason for the discrimination beyond the mere fact that they are citizens of other states. Therefore, the state tax on nonresident photographers should properly be declared unconstitutional as violative of the Privileges and Immunities Clause. Choice (A) is incorrect. The equal protection clause provides heightened protection for groups whose legal classifications are suspect (e.g., classifications based on race, ethnicity, nationality, and religion) and for groups whose legal classifications are quasi suspect (e.g., gender, illegitimacy). The photographer may have been the subject of discrimination by the Legislature, but he is not a member of a suspect or quasi-suspect class. Only rational review will be applied, which the state probably will pass with ease. Choice (B) is incorrect. Generally, a property interest is not protected under the 14th Amendment’s Due Process Clause unless there is a reasonable expectation to continued receipt of the benefit [Board of Regents v. Roth, 408 U.S. 564 (1972)]. In our example, no such reasonable expectation to continued tax benefits appears to exist. Choice (C) is incorrect. States and other local governments have no powers under the Commerce Clause of Article I, Section 8.
66
Q
  1. A state enacts a statute that prohibits “anyone over
    60 years of age to run for public office.” A state senator has been in office for three terms and wishes to seek re-election. The senator, who is 61, brings suit challenging the constitutionality of the state statute.
    Which of the following best states the burden of persuasion?
    (A) Since a fundamental right is involved, the state must show the regulation is necessary to vindicate a compelling government interest.
    (B) Since no fundamental right is involved, the petitioner must show the age restriction is not rationally related to a legitimate government interest.
    (C) The state must show the age regulation substantially furthers an important government objective and does not impair the fundamental right to vote.
    (D) The petitioner must show the statute violates due process by depriving her of the right to be a candidate.
A
  1. (B) The Constitution contains no express provision that guarantees the right to be a candidate. The states are free, therefore, to create restrictions on the ability to become a candidate. Certainly, states have used several methods to qualify the right to become a candidate. Even though the Supreme Court has not ruled directly on candidacy age restrictions, interestingly enough, this issue was tested on the Multistate exam recently. Choice (B) is correct because in dicta from previously decided cases, the Supreme Court apparently is applying minimal scrutiny to age restrictions. See “Age and Durational Residency Requirements as Qualifications for Candidacy: A VioLation of Equal Protection?,” 1973, U.Ill. Law Review, 161. Choice (A) is incorrect. There is no fundamental right to run for state office. Accordingly, from the perspective of fundamental rights anaLysis, the court will apply rational review. Choice (C) is incorrect because there is no fundamental right to run for state office. Finally, choice (D) is incorrect because the right to run for public office is not a right protected under the due process clause of the 14th Amendment.

Exam Tip: This question needs to be distinguished from Question 67. In Question 67, the state enacted a party affiliation statute placing restrictions on independent candidates. States usually impose demonstrated support requirements on independent candidates or minor political parties. Typically, the demonstrated support statute requires independent candidates or minor parties to submit petitions containing a certain number of signatures from qualified voters before they can receive access to the ballot. In such cases, the Supreme Court generally “has stated that the state needs a compelling or overriding interestto justify classifications and restrictions on political association.” Nowak, pg. 891. On the contrary, this Multistate hypo deals with age restrictions on the right of candidacy. With respect to age classifications, the Supreme Court appears to apply the rational-basis test. Although the Supreme Court has not ruled directly on this age candidacy issue, all related cases involving age discrimination have been adjudged under the minimal scrutiny-rational basistest. In Trafeletv. Thompson, 100 S.Ct. 219 (1979), the Supreme Court refused to review a state law imposing a mandatory retirement age for elected state court judges that was challenged as an age classification violative of equal protection.

67
Q
  1. A state has enacted a party affiliation statute prohibiting a person from being an independent candidate in a general election if she had either (1) registered with a political party during the year prior to the immediately preceding primary, or (2) voted in that primary. The state adopted the so-called “disaffiliation” statute in order to have intraparty feuds resolved in primary elections rather than in the general election. Moreover, the state’s elections director strongly supported the law and argued that it was necessary to avoid voter confusion and to ensure that the general election winner received a majority.
    A man, who was a registered Democrat in 2008, now wishes to run as an independent candidate in the November, 2009, general election. However, the elections director ruled that his candidacy violated the state’s “disaffiliation” statute and barred him from appearing on the ballot.
    If the man files suit in federal district court challenging the constitutionality of the state’s election statute, which of the following best states the burden of persuasion?
    (A) The state must demonstrate that the law is necessary to further an important state interest under equal protection analysis.
    (B) The state must demonstrate that the law is necessary to further a compelling state interest under fundamental rights analysis.
    (C) The man must demonstrate that the law is not rationally related to any legitimate state interest under fundamental rights analysis.
    (D) The man must demonstrate under fundamental rights analysis that the state has less restrictive alternative means available for independent candidates to get a ballot position.
A
  1. (B) The right to be a candidate is related to the fundamental right to vote. In general, the state’s interest in limiting ballot access is twofold: (1) to reduce voter confusion, and (2) to maximize the probability that the winning candidate will have received a majority of the popular vote. In Storer v. Brown, 415 U.S. 724, (1974), a California provision that prohibited independent candidates from running in the general election if they had eithervoted in the immediately preceding party primary or registered their party affiliation with a qualified party within one year of the primary was upheld. The Court determined that the “disaffiliation” statute furthered the state’s compelling interest in the stability of the political system and its interest in having “intraparty feuds” resolved in primaries rather than in the general election. Based on the Court’s application of the strict scrutinystandard of review, (B) is the correct answer. The Court noted further in Storer that the state must adopt reasonable alternative means for independent candidates and minor political parties to get a ballot position, and the alternative means must not place too heavy a burden on the right to vote and the right to associate. Choice (A) is incorrect. The language of “important” state interest is usually associated with “intermediate review” in equal protection clause analysis. Intermediate review is reserved for legal classifications based on gender and illegitimacy. In our example, there is no reference to either classification. Choice (C) is incorrect because the right to be a candidate is related to the fundamental right to vote [Storer v. Brown, 415 U.S. 724, (1974)]. Therefore, the court will apply strict scrutiny to any law that infringes such a right. Choice (D) is incorrect. The court will likely apply strict scrutiny (see explanation for Choice B). In that case, the state will have the burden to show that no less-restrictive alternatives exist.
68
Q
  1. A defendant was charged with the crime of rape. The judge denied him bail pursuant to a state law which states that for the crimes of rape, sexual assault on a child, and sexual assault, no person who stands accused thereof shall be entitled to bail prior to a trial in the courts of this state.
    The defendant was brought to trial and found guilty. After being sentenced to five to ten years in prison, the defendant appealed his conviction to the highest court in the state. The ground for his appeal was an argument that he was denied his right to counsel at the time of his arrest.
    While his appeal was pending, the defendant filed a civil rights action in federal court against the judge. The defendant claimed that the judge violated his rights under the excessive bail clause of the Eighth Amendment.
    The federal court should refuse to hear the case, because
    (A) the federal court would violate the principle of the separation of powers.
    (B) the issues are not ripe.
    (C) the case is moot.
    (D) the issue of bail is capable of repetition, yet evading review.
A
  1. (C) In Murphy v. Hunt, 71 L Ed 2d 353, 102 5.Ct 1181 (1982), the U.S. Supreme Court held that a petitioner’s claim against Nebraska’s prohibition of pretriaL bail to a person charged with sexual offenses had violated his constitutional rights under the 8th Amendment was moot, since the petitioner-defendant had already been convicted of the offenses. As a general rule, a case becomes “moot” when the issues presented are no longer “live” or the parties lack a legally cognizable interest in the outcome of the case. Choice (A) is incorrect. Article III, Section 2 states that the judicial power of the federal courts “shall extend to all Cases, in Law and Equity, arising under this Constitution, the Laws of the United States…” Issues pertaining to the 8th Amendment are related to the Constitution. Accordingly, if a federal court were prohibited by the Constitution from hearing this case, the prohibition is not attributable to the principle of the separation of powers. Choice (B) is not the strongest choice. In a sense, this is true. However, the issues in the example will never become ripe; they are moot. Hence, choice (C) is the stronger answer. Choice (D) is incorrect because the bail issue is capable of repetition, but the next aggrieved person can, before conviction, present it for review.
69
Q
  1. In an attempt to promote safe sex a foundation began sending condoms in the mail. Thousands of Americans became incensed and objected to this type of unsolicited advertising. A group of people started a nationwide campaign against the use of condoms. This new organization also started a strong lobbying movement to have Congress pass legislation prohibiting the distribution of condoms by using the U.S. postal system.
    Assume that the lobbying effort was successful, and Congress passed a law prohibiting any unsolicited advertising for condoms to be distributed through the U.S. postal system. The foundation has challenged the constitutionality of this federal statute. The best argument against the constitutionality of this law is which of the following?
    (A) The statute is invalid because it violates the First Amendment protection of commercial free speech.
    (B) The statute is invalid because it unduly burdens interstate commerce.
    (C) The statute is invalid because it violates the Fifth Amendment right of privacy.
    (D) The statute is invalid because it violates the equal protection clause of the Fourteenth Amendment.
A
  1. (A) This is an extremely difficult Constitutional Law Multistate question. Choice (A) is correct because in Bolger v. Young’s Drug Products Corp., 463 U.S. 60 (1983), a law prohibiting the mailing of unsolicited advertisements for contraceptives was held invalid as violating 1st Amendment’s protection of commercial free speech.
    The Court held that the government’s interest in protecting recipients from mailthey find “offensive” is insubstantial. Choice (B) is wrong: this is not a state law that unduly burdens interstate commerce but rather a federal law. Choice (C) is wrong because the use of contraceptives is a fundamental right under the protected zone of “marital privacy.” There is a subtle distinction, because this question does not dealwith a statute restricting use of contraceptives. On the contrary, the Law restricts “unsolicited advertising” for contraceptives. Choice (B) is incorrect. The Commerce Clause of Article I, Section 8 permits Congress (and Congress alone) to regulate interstate commerce. Thus, Congress cannot be said to present an “undue burden” on interstate commerce; only local governments can do that and, thus, violate the dormant commerce clause of Article I, Section 8. Choice (C) is incorrect because the 5th Amendment’s right of privacy pertains to the right against self-incrimination, not the right to be Left alone suggested in our example. Finally, choice (D) is incorrect. The equal protection clause provides heightened protection for groups whose legal classifications are suspect (e.g., classifications based on race, ethnicity, nationality, and religion) and for groups whose legal classifications are quasi suspect (e.g., gender, illegitimacy). The foundation may have been the subject of discrimination by Congress, but they are not members of a suspect or quasi-suspect class. Only rational review will be applied, which Congress probably will pass with ease.
70
Q
  1. In recent years there has been much publicity regarding juries approving excessively high multi- million dollar damage awards in personal injury actions. As a result, Congress enacted a statute that limited recovery in personal injury actions filed in state court(s) to $400,000, and punitive action recovery to a maximum of $750,000.
    A man was injured in an automobile accident when a car driven by a woman drove through a red light and struck his vehicle. The man was paralyzed from the accident and became a paraplegic. The man brought a personal injury action against the woman in state court. The jury returned a verdict on the man’s behalf and awarded him $1,000,000 in damages.
    After the jury verdict, the woman filed an appeal challenging the amount of the award, claiming that it was excessive and violated the federal guidelines set forth in the statute.
    The man has filed suit challenging the constitutionality of the statute. The statute should be ruled
    (A) unconstitutional, because Congress cannot enact legislation involving local matters, such as automobile accidents, unless it involves interstate commerce.
    (B) unconstitutional, because a limitation on damage awards in tort actions would violate the equal protection clause of the Fourteenth Amendment as applicable to the states by operation of the Fifth Amendment.
    (C) constitutional, because under Article III, Congress has plenary power to regulate the jurisdiction and scope ofjudicial review of federal and lower state courts.
    (D) constitutional, because under the supremacy clause, when there is a conflict between federal law and state law, the federal law preempts and takes precedence over the conflicting state law.
A
  1. (A) Under its commerce power, Congress has plenary power to regulate interstate commerce and commerce with foreign nations. Obviously, the power of Congress to regulate commerce is very broad. It does, however, have limits so as not to obliterate the distinction between what is national and what is local. To be within Congress’s power under the Commerce Clause, a federal law must either (1) regulate the channels of interstate commerce; (2) regulate the instrumentalities of interstate commerce; or (3) regulate activities thathave a substantial effect on interstate commerce. Since the Tort Liability Reform Act does not affect interstate commerce, it will be invalidated as an impermissible intrusion on local affairs. Therefore, choice (A) is correct. Choice (B) is incorrect. The equal protection clause provides heightened protection for groups whose legal classifications are suspect (e.g., classifications based on race, ethnicity, nationality, and religion) and for groups whose legal classifications are quasi suspect (e.g., gender, illegitimacy). People like the defendant may have been the subject of discrimination by Congress, but they are not members of a suspect or quasi-suspect class. Only rational review will be applied, which Congress probably will pass with ease. Choice (C) is incorrect. Article Ill, Section 2 contains an “exceptions clause”: “In all cases affecting Ambassadors, other public Ministers and Consuls, and those in which a State shall be Party, the supreme Court shall have original Jurisdiction. In all the other Cases before mentioned, the supreme Court shall have appellate Jurisdiction, both as to Law and Fact, with such Exceptions, and under such Regulations as the Congress shall make.” Congress may not use its exceptions clause powers to curtail those powers that belong to the judiciary; such abuse would violate the principle of the separation of powers [ExParteMcCardle, 74 U.S. 506 (1869)1. In our example, Congress would appearto limit unduly the court’s discretion regarding what the plaintiff is owed. Choice (D) is incorrect because under the Supremacy Clause of Article VI, a federal law preempts a state law only if the federal law is constitutional. Here, the federal law lacks constitutional basis.
71
Q
  1. Litigation arose in the courts of a state when the Secretary of Commerce attempted to stop a man from sending his new advertising brochures through the mail. Under the Secretary’s interpretation, the new brochures contained obscene photographs and sexually suggestive language. The Secretary was acting under authority given to him by Section 1123(b) of state statute, which provided:
    “Whenever the Secretary of Commerce shall determine that a commercial mailing to residents of the state is inappropriate for minors, the Secretary shall have the authority to prohibit such mailing.”
    The case reached the highest court in the state, which held that the statute was unconstitutional, being in violation of both the commerce clause and the supremacy clause.
    If this case is reviewed by the U.S. Supreme Court, it will reach that court
    (A) by appeal.
    (B) by certiorari.
    (C) if five justices vote to review it.
    (D) despite the doctrine of adequate and independent state grounds.
A
  1. (B) In accordance with 28 U.S.C. Section 1257, when astate statute is declared unconstitutional by the highest state court, the route of appeal is by certiorari. The highest court of the state held that Section 1123(b) of the state statute was unconstitutional. Therefore, choice (B) is the best answer. Choice (A) is incorrect because in 1988, Congress practically eliminated obligatory Supreme Court review by appeal with a couple of minor exceptions. (See Constitutional Law Outline, supra, pg. 2.) Choice (D) is incorrect, as the doctrine of adequate and independent state grounds is inapplicable because the state court decision was not based on state grounds. Under this doctrine, the Supreme Court may only review cases involving a “federal question.” Last, choice (C) is incorrect because only four justices are necessary for the Supreme Court to grant certiorari.
72
Q
  1. A state legislature has recently enacted an obscenity statute prohibiting “the selling or offering for sale of any obscene printed or video materials.” Following numerous complaints from a local citizens’ group, the police entered an adult bookstore, examined materials on the shelves, and purchased magazines that depicted actual pictures of sexual intercourse. The owner was subsequently arrested and charged with violating the state obscenity law. At his trial, the owner’s defense was that the sale of the materials complained of was constitutionally protected speech.
    Which of the following, if established, would be most helpful to the owner’s defense?
    (A) The particular materials involved depicted normal, not deviant, sexual conduct.
    (B) The particular materials involved consisted of serious scientific studies of human sexual urges.
    (C) The police did not have a search warrant when they entered the bookstore to purchase the particular materials involved in this obscenity prosecution.
    (D) 85% of the citizens of the state believe that the sale of sexually explicit material does not contribute to antisocial sexual behavior.
A
  1. (B) The case of MilIerv. California 413 U.S. 15 (1973) states the present rule for obscenity. The basic guidelines for the trier of fact must be (a) whether “the average person applying contemporary community standards” would find that the work, taken as a whole, appeals to the prurient interest, (b) whether the work depicts or describes, in a patently offensive way, sexual conduct specificaLly defined by the applicable state law, and Cc) whether the work, taken as a whole, Lacks serious literary, artistic, political, or scientific value. Applying this test, the best defense for the owner would be choice (B), which addresses the third element of Miller. If the materials the police purchased consisted of serious, scientific studies of human sexual urges, then the materiaLs, taken as a whole, would not be obscene. Choice (A) is incorrect since normal sexual conduct is irrelevant to the elements defined by the Miller standard. Choice (C) is incorrect. Probable cause wouLd also suffice to avoid 4th Amendment prohibitions against unreasonabLe search and seizure. The police may have acquired probable cause after reviewing the materials. Choice (D) is incorrect because, even if true, this is irrelevant as a matter of Law in determining whetherthe material is obscene.
73
Q
  1. Congress enacts a statute that makes Pennsylvania Avenue a one-way street. Congress has enacted the statute solely for the purpose of enabling members of Congress to travel back and forth to the Capital without being tied up in heavy traffic each day. Before Congress enacted this law, the Washington, D.C., City Council had previously passed an ordinance designating Pennsylvania Avenue as a two-lane street.
    Which of the following is the most accurate statement regarding the constitutionality of the federal statute designating Pennsylvania Avenue a one-way street?
    (A) It is valid because Congress has exclusive power over the District of Columbia.
    (B) It is valid because it is a proper exercise of Congress’s enumerated property power.
    (C) It is invalid because the Washington, D.C., City Council has exclusive power over the public thoroughfares within the District of Columbia.
    (D) It is invalid because it is a discriminatory burden on interstate commerce.
A
  1. (A) Under Article 1, Section 8, Clause 17, Congress has the powerto exercise exclusive legislation overthe District of Columbia and to govern places where the government has purchased and erected forts, arsenaLs, dockyards, and other needful buildings. Based on this enumerated power, Congress may properLy legislate to make Pennsylvania Avenue a one-way street, and the federal law wILL predominate over any conflicting ordinance passed by the District of Columbia City Council. Choice (A) is correct. Choice (B) is a potentially good answer but there is explicit support for Congress’s power in Article I, Section 8, Clause 17 to regulate Washington, D.C. Choice (C) is incorrect. As stated above, Congress has exclusive power to legislate over the District of Columbia. Choice (D) is incorrect. Under the Commerce Clause of Article I, Section 8, Congress has authority to regulate interstate commerce. Thus, Congress, as a matter of Law, cannot be deemed to present an undue burden on interstate commerce; onLy state and other local governments can present such a burden and, thus, violate the dormant commerce clause of Article I, Section 8.
74
Q
  1. A city housing code limits the occupancy of all dwelling units in the city to members of a single family. The chief purpose of the code is to limit traffic congestion and noise in the neighborhood. One provision in the code provides that a “family” means a number of individuals related to the nominal head of the household or to the latter’s spouse living as a single housekeeping unit in a single dwelling unit, but is limited to the following:
    (a) husband or wife of the nominal head of the household;
    (b) married or unmarried children of the nominal head of the household or the latter’s spouse, provided, however, that such married or unmarried children have no children of their own residing with them;
    (c) father or mother of the nominal head of the household or of the latter’s spouse.
    A resident of the city lives in her home together with her son and his two children. She received a notice of violation from the city which stated that both grandchildren were illegal occupants of her home and that she must comply with the code. After she fails to remove her two grandchildren, the city brings criminal charges against the resident.
    In an action by the resident challenging the constitutionality of the code provision, the court will most likely declare the provision
    (A) constitutional, on the grounds that the housing provision bears a rational relationship to permissible state objectives.
    (B) constitutional, under the state police power. (C) unconstitutional, as violative of the due process clause of the Fourteenth Amendment.
    (D) unconstitutional, as having no substantial relation to the public health, safety, morals, and general welfare of the state.
A
  1. (C) In Moore v. City of East Cleveland, Ohio, 97 S.Ct. 1932 (1977), the U.S. Supreme Court held that a housing ordinance limiting occupancy of a dwelling unit to members of a singLe family and recognizing as a “family” only a few categories of reLated individuals, under which it was a crime for a grandmother to have certain grandchildren living with her, was violative of the Due Process Clause of the 14th Amendment. The Court recognized that freedom of personal choice in matters of marriage and family Life is one of the liberties protected bythe Due Process Clause of the 14th Amendment. Thus, a city may not regulate the occupancy of its housing by selecting certain categories of relatives who may live together and declaring that others may not. This case is distinguished from Village of Belle Terre v. Boraas, 416 U.S. 1 (1914), where the housing ordinance affected only unrelated individuals. Choice (A) is incorrect because the court will apply strict scrutiny. Choice (B) is incorrect. In Moore v. City of East Cleveland, Ohio, 97 S.Ct. 1932 (1977), the U.S. Supreme Court suggested that the right to organize one’s family is a fundamental right under the liberty interest protected by the 14th Amendment’s Due Process Clause. The city ordinance wouLd seem to infringe on this fundamental right. Accordingly, the court will apply strict scrutiny. Under strict scrutiny, the court will ask whether the ends are compelling and the means chosen are the least restrictive with respect to the fundamental right. Here, the ends are to limit traffic congestion and noise; such ends likely will constitute compelling governmental interests. However, the means chosen are not the least restrictive because the city could have created laws to curb traffic congestion (or hired more police to do so) and to limit noise (or had more intense police enforcement of preexisting codes). Choice (D) is incorrect. The code will, most likely, be struck down, but not because it lacks a “substantial relation” to public health, safety, morals, and welfare. Rather, the code will be subject to strict scrutiny because the right to familial organization is a fundamental right protected under the liberty interest of the 14th Amendment [Moore v. City of East Cleveland, Ohio, 97 S.Ct. 1932 (1977)]. As such, the code, which infringes this fundamental right, will be subject to strict scrutiny. Under strict scrutiny, the court asks whether there is a compelling governmental interest and whether the means chosen are the least restrictive. The language in choice (D) suggests that the city will lose because there is “no substantial relation” to health, safety, and so on.
75
Q
  1. A state legislature has proposed a bill setting up a postal service. Under the proposal, the postal service would be established as a separate state agency under the direction of a postmaster general. The postal service would be responsible for the overnight or express delivery of official governmental mailings within the state. The bill further provides that all state government employees may utilize the overnight postal service at no charge for sending mailings to persons or corporations within the state.
    Assume that no other state has established such a postal service to date. Moreover, assume there is uncontradicted evidence that the establishment of such an independent state postal service will diminish the revenues of the U.S. Postal Service.
    Which of the following, if established, is the strongest argument in support of the proposed legislation?
    (A) Under the Tenth Amendment, a state has exclusive authority to regulate transactions that are wholly intrastate.
    (B) Since the proposed bill is not inconsistent with congressional postal power, it is valid under the supremacy clause.
    (C) State employees may be exempt from paying for mailing charges under intergovernmental immunity provided Congress has enacted a similar exemption for federal governmental employees.
    (D) There is a legitimate state interest in ensuring that governmental mailings are delivered expeditiously.
A
  1. (D) Use process of elimination here. Choice (A) is wrong because a state does not have exclusive authority to regulate intrastate transactions. Under the Commerce Clause, Congress has the power to regulate any activity that has a direct or immediate effect on interstate commerce, even though the activity takes place within a single state. Choice (B) is incorrect because the Supremacy Clause does not validate state laws. On the contrary, the effect of this clause is that to whatever extent Congress has exercised its powers, any “inconsistent” state laws are prohibited. Similarly, choice (C) is not the best answer because intergovernmental immunity, in general, places certain limitations on the state’s power to regulate and tax the property and activities of the federal government. By process of elimination, choice (D) is the best answer.
76
Q
  1. A state was facing a fiscal problem. In order to raise funds, the state decided to consider the creation of a state postal service that only its residents may use for intrastate mail. The price for stamps printed by the state postal service would be half the cost of stamps printed by the U.S. Postal Service and would always be guaranteed to arrive within 24 hours. In addition, any employees of the state would be able to send their mail for free through the state postal service.
    Which of the following is the strongest constitutional argument against the proposed legislation?
    (A) It constitutes a denial of equal protection of the laws, because employees of the state receive personal benefits that are not enjoyed by employees of other states.
    (B) The negative implications that flow from the delegation to Congress of the power to establish a post office prohibit such a state postal service.
    (C) It denies citizens of the state who are not state government employees their rights under the privileges or immunities clause of the Fourteenth Amendment.
    (D) It interferes with the sovereign autonomy of the U.S. Postal Service to operate as an independent federal agency.
A
  1. (B) In the area of Constitutional Law, students are often presented with a proposed state statute and are required to determine the strongest argument IN SUPPORT of the legislation and the strongest argument AGAINST it. The strongest argument AGAINST an independent state postal service was held to be the negative implications that flow from the delegation to Congress of its postal power under Article I, Section 8. This is so because the facts indicate that a state postal service will diminish the revenues of the U.S. Postal Service. Choice (A) is incorrect. The equal protection clause provides heightened protection for groups whose legal classifications are suspect (e.g., classifications based on race, ethnicity, nationality, and religion) and for groups whose legal classifications are quasi suspect (e.g., gender, illegitimacy). Those not employed by the state may have been the subject of discrimination by the legislature, but they are not members of a suspect or quasi-suspect class. Only rational review will be applied, which the state probably will pass with ease. Choice (C) is incorrect. In order to show a violation of the 14th Amendment’s privileges or immunities clause, one must show that he is being denied the privilege or immunity by a state because he is not a citizen of the state. In our example, the state confers benefits not along the lines of state citizenship but along the lines of employment by the state. Finally, choice (D) is incorrect. According to Article I, Section 8, Congress has authority to regulate the U.S. Postal Service. Therefore, the U.S. Postal Service does not exercise “sovereign autonomy.”
77
Q
  1. A resident announced his candidacy for state representative. A law in the state requires new political entrants (regardless of party affiliation) to obtain three times the number of signatures as other candidates who have run for office previously. The resident, however, failed to obtain the necessary number of authenticating signatures to have his name placed on the ballot.
    The resident filed a complaint in federal district court alleging the unconstitutionality of the authenticating requirement. Which of the following, if established, is the state’s strongest argument for sustaining the validity of the authenticating requirement?
    (A) The resident’s petition contained a large number of false signatures.
    (B) A similar authenticating statute was held to be constitutional in another state the previous year.
    (C) The authenticating requirement was necessary to further a compelling state interest.
    (D) Two other candidates had successfully petitioned to have their names included on the ballot.
A
  1. (C) In Jenness v. Fortson, 403 U.S. 431 (1971), the U.S. Supreme Court uphe(d a Georgia law requiring candidates for elective office who ran without winning a primary election to file petitions with signatures from qualified voters equaling 5% of the vote cast in the last general election for that office. The Court held that such state requirements (whereby a candidate or new political party demonstrate public support in order to get on the ballot) further a “compelling state interest,” i.e., preserving the integrity of the electoral process by preventing the ballot from becoming unmanageable and confusing. Choice (A) is incorrect. As suggested by jenness v. Fortson, 403 U.S. 431 (1971), the strongest argument by the state is probably its interest in preventing confusion and an unmanageable litany of candidates’ names on the ballet. The Jenness Court recognized such interests to be “compelling” under strict scrutiny. Choice (B) is incorrect because even if this were true, it would not be the state’s strongest argument insofar as the statute and the case facts in the other state case could be different in legally relevant ways from those in our example. Choice (D) is not the best answer because even if this were true, the facts that pertain to the two candidates could distinguish them in legally meaningful ways from the resident.
78
Q
  1. A resident announced his candidacy for state representative. A law in the state requires new political entrants, regardless of party affiliation, to obtain five times the number of signatures as other candidates who have run for office previously. The resident, however, failed to obtain the necessary number of authenticating signatures to have his name placed on the ballot.
    On the substantive constitutional issue regarding the validity of the authenticating requirement, the most probable judicial resolution will be to
    (A) hold the authenticating requirement unconstitutional as violative of the due process clause of the Fourteenth Amendment.
    (B) hold the authenticating requirement unconstitutional as violative of the equal protection clause of the Fourteenth Amendment.
    (C) dismiss the cause of action because state election procedures are a sovereign state function.
    (D) assign the burden of proving the validity of the authenticating requirement to the
    state.
A
  1. (D) The u.s. Supreme Court has upheld the requirement that a candidate receive a rea
    sonable number of signatures before being placed on the ballot. However, in cases
    in which the signature requirement places a severe restriction on being placed on
    the ballot, the burden will be on the state to show that the regulations are narrowly
    drawn to achieve a state interest of compelling importance [Norman v. Reed, 502
    u.s. 279 (1992)]. Choice (A) is incorrect because based on the facts, we do not know
    how the Court will decide. All that we know is that the Court will likely subject the
    law to strict scrutiny [Norman v. Reed, 502 u.s. 279 (1992)]. Choice (B) is incorrect.
    The equal protection clause provides heightened protection for groups whose legal
    classifications are suspect (e.g., classifications based on race, ethnicity, nationality,
    and religion) and for groups whose legal classifications are quasi suspect (e.g., gen
    der, illegitimacy). The resident may have been the subject of discrimination by the
    legislature but not because he is a member of a suspect or quasi-suspect class. Only
    rational review will be applied, which the other state probably will pass with ease.
    Choice (C) is incorrect. While there is no constitutional right to run for public office, the voters possess fundamental rights under the 1st and 14th Amendments to vote in political elections, including state elections [Norman v. Reed, 502 U.S. 279 (1992)]. Thus, state elections are not immune from scrutiny under the u.s. Constitution.
79
Q
  1. An avowed corrimunist was elected vice president of a union. A senator, in his investigation of communist infiltration of national labor unions, found a provision in a statute passed by Congress, which makes it a crime for a member of the communist party to act as an official of a labor union. After a subsequent legislative hearing, the communist is dismissed from his position by the union.
    Which of the following most accurately summarizes the applicable rule of constitutional law regarding the aforementioned provision of the statute?
    (A) The statutory provision is a form of unconstitutional prior restraint on a person’s First Amendment right of free association.
    (B) Making it a crime for a Communist to hold a union office is a suspect classification, which violates the equal protection clause.
    (C) The statutory prohibition is a reasonable method of discrimination since the benefit to the public outweighs the injury or restrictions that would be inflicted upon the person.
    (D) The statutory provision in the act is a form of legislative punishment violative of the Constitution as a bill of attainder.
A
  1. (D) In United States v. Brown, 381 u.s. 437 (1965), the u.s. Supreme Court found that a provision in the Landrum-Griffin Act making it a crime for a member of the Communist Party to act as an officer or employee of a labor union to be legislative punishment for Party membership, and hence, a bill of attainder. Choice (A) is incorrect. Prior restraints are generally characterized by requirements for licensing or advanced permission from the government. Here, we have a generic criminal law that limits a person’s rights of speech and association. Choice (B) is incorrect. Since we have a congressional statute, not a state statute, the 14th Amendment’s Equal Protection Clause does not apply. We apply instead the equal protection principle that the Supreme Court has read into the 5th Amendment’s Due Process Clause. The equal protection principle in the 5th Amendment provides heightened protection for groups whose legal classifications are suspect (e.g., classifications based on race, ethnicity, nationality, and religion) and for groups whose legal classifications are quasi suspect (e.g., gender, illegitimacy). The communist may have been the subject of discrimination by the legislature, but he is not a member of a suspect or quasi-suspect class. Only rational review will be applied, which Congress probably will pass with ease. Choice (C) is incorrect. The statutory clause will likely be struck down as violating Article I, Section 9, which prohibits a state from passing a bill of attainder. In United States v. Brown, 381 U.S. 437 (1965), the U.S. Supreme Court found that a provision in the Landrum-Griffin Act making it a crime for a member of the Communist Party to act as an officer or employee of a labor union to be legislative punishment for Party membership and, hence, a bill of attainder.
80
Q
  1. An avowed member of an antigovernment organization was elected president of a union. A senator, in his investigation of infiltration by antigovernment organizations of national labor unions, found a provision in a law passed by Congress that makes it a crime for a member of an antigovernment organization to act as an official of a labor union. After a subsequent legislative hearing, the president is dismissed from his position by the executive committee of the union.
    During a legislative hearing, the senator made derogatory statements about the president’s involvement in an antigovernment organization.
    In determining whether the president has a valid cause of action against the senator for defamation, which of the following most accurately summarizes the applicable rule of law?
    (A) Congressional committees do not have the authority to violate a person’s Fifth Amendment privilege against self-incrimination.
    (B) A congressman shall not be questioned in any other place for any speech or debate made by him during a congressional hearing.
    (C) The constitutional requirement for actual malice must be proved by the party defamed in order to recover in a defamation suit.
    (D) It is not a denial of due process or of First Amendment rights for a congressional investigative committee member to make such utterances.
A
  1. (B) Article I, Section 6 of the U.S. Constitution provides that “for any Speech or Debate in either House, they (members of Congress) shall not be questioned in any other place.” Thus, the speech and debate clause would confer immunity upon the senator, exempting him from liability for any remarks made by him during the congressional hearing. Choice (A) is incorrect. In Hutchenson v. United States, 369 U.S. 599 (1962), the Court stated that congressional committees have the authority to issue subpoenas for people to testify. Choice (C) is incorrect. The senator would not be held liable underthe circumstances because Article I, Section 6 of the U.S. Constitution provides that “for any Speech or Debate in either House, they (members of Congress) shall not be questioned in any other place.” Thus, the speech and debate clause would confer immunityupon the senator, exempting him from liability for any remarks made by him during the congressional hearing. Choice (D) is incorrect. There might be due process or 1st Amendment violations by the senator but Article I, Section 6 of the U.S. Constitution, provides that “for any Speech or Debate in either House, they (members of Congress) shall not be questioned in any other place.” Thus, the speech and debate clause would confer immunity upon the senator, exempting him from liability for any remarks made by him during the congressional hearing.
81
Q
  1. A state resident could purchase a license solely for hunting elk for $9.00. A nonresident, in order to hunt elk, was required to purchase a combination license at a cost of $225.00; this entitled him to take one elk, one deer, and one black bear. A nonresident, however, could obtain a license restricted to deer for $51.00. A resident was not required to buy any combination of licenses but if he did, the cost to him of all the privileges granted by the nonresident combination license was $30.00.
    Due to its successful management program for elk, the state has not been compelled to limit the overall number of hunters by means of drawings or lotteries, as have other states. Elk are not hunted commercially in the state. Nonresident hunters seek the animal for its trophy value; the trophy is the distinctive set of antlers. ‘.Vhereas the interest of resident hunters more often may be in the meat, among nonresident hunters, big-game hunting is clearly a sport in the state.
    Two residents of another state bring suit against the state. They assert in their complaint that the disparities between residents and nonresidents in the state hunting license system is unconstitutional. The state’s hunting license system should be found
    (A) constitutional, because it is within the police power of a state to regulate a recreational, noncommercial activity.
    (B) constitutional, because there is a compelling state interest.
    (C) unconstitutional, because it violates the privileges and immunities clause of Article IV, Section 2.
    (D) unconstitutional, because it violates the equal protection clause of the Fourteenth Amendment.
A
  1. (A) In Baldwin v. Montana Fish and Game Commission, 436 U.S. 371 (1977), the U.S. Supreme Court held that the Montana elk-hunting licensing scheme, as applied to nonresidents, was not a fundamental right under the Privileges and Immunities Clause of Article IV, Section 2. Furthermore, the Court majority found no discrimination in the distinctions drawn between residents and nonresidents under the Equal Protection Clause of the 14th Amendment. The Supreme Court concluded that protection of the wildlife of a state is peculiarly within the police power of the state. The court also noted that the elk-hunting licensing scheme did not violate petitioners’“privileges and immunities,” because hunting (on the part of non-Montana residents) was primarily a recreational endeavor. Choice (B) is incorrect. The reference to “compelling state interest” suggests that the court will apply strict scrutiny, but this is doubtful under Baldwin. Choice (C) is incorrect for the reasons stated in the discussion of choice (A), above. Finally, choice (D) is incorrect. The equal protection clause provides heightened protection for groups whose legal classifications are suspect (e.g., classifications based on race, ethnicity, nationality, and religion) and for groups whose legal classifications are quasi suspect (e.g., gender, illegitimacy). The residents may have been the subject of discrimination by the state, but they are not members of a suspect or quasi-suspect class. Only rational review will be applied, which the state probably will pass with ease.
82
Q
  1. A city imposes a tax on nonresident’s city derived income above $1,700 at a 3% rate, except that if the nonresident’s state of residence would impose a lesser tax had the income been earned in that state, the city tax would be reduced to that amount. This income tax exempts taxable income earned by city residents outside of the state. Moreover, residents of the city were not taxed on their in-city earned income.
    A commuter who works in the city but is a resident of a neighboring state, challenges the constitutionality of this statute.
    Which of the following provisions would furnish the most applicable basis for this constitutional challenge?
    (A) The equal protection clause of the Fourteenth Amendment.
    (B) The Fourteenth Amendment’s due process clause.
    (C) The privileges and immunities clause of Article
    IV.
    (D) The commerce clause.
A
  1. (C) In Austin v. New Hampshire, 420 U.S. 656 (1975), the U.S. Supreme Court held that a similar New Hampshire commuters’ income tax on nonresident taxpayers to be violative of the Privileges and Immunities Clause of Article IV, Section 2, which provides that “The citizens of each State shall be entitled to all Privileges and Immunities of citizens in the several states.” Choice (A) is incorrect. The equal protection clause provides heightened protection for groups whose legal classifications are suspect (e.g., classifications based on race, ethnicity, nationality, and religion) and for groups whose Legal classifications are quasi suspect (e.g., gender, illegitimacy). The commuter may have been the subject of discrimination by the city, but he is not a member of a suspect or quasi- suspect class. Only rational review will be applied, which the city probably will pass with ease. Choice (B) is incorrect. Although property is protected by the 14th Amendment’s due process clause, the taxed income probably would not be entitled to due process protection as a taking [New York ex rel. Cohn v. Graves, 300 U.S. 308 (1937)]. Choice (D) is incorrect. The doctrine of the dormant commerce clause is meant to prohibit “economic protectionism—that is, regulatory measures designed to benefit in-state economic interests by burdening out-of-state competitors. . . .“ [Wickard v. Filburn, 317 U.S. 111 (1942)] Notice that no such protectionism exists, since Jones (the out-of-stater) would pay no more than what his own state would require byway of the income tax.
83
Q
  1. A corporation under the authority of a state statute sued to have condemned 1,000 acres of forested land owned by a timber company, which it planned to develop for use as a state recreational area. After a hearing, the state court ordered possession of the land surrendered to the corporation, upon deposit in court of a sum deemed adequate to cover damages that might be awarded. The timber company immediately commenced an action to enjoin the court-ordered sale of their property.
    Which of the following would be the best ground for upholding the state court’s order?
    (A) The power of eminent domain may only be delegated directly to a private enterprise for a public related use or activity.
    (B) The power of eminent domain may only be delegated to a public authority through a legislative determination.
    (C) The injured party has not proved such irreparable injury to use as amounts to a “taking.”
    (D) The Fifth Amendment’s power of eminent domain incorporated by the Fourteenth Amendment as applicable to the states does not require that payment be made prior to condemnation of the property.
A
  1. (A) The power of eminent domain may be delegated directly or indirectly to a private per
    son or enterprise, subject to the requirements that the taking be (a) for a public use,
    and (b) just compensation be given. Therefore, choice (A) is correct, and choice (B)
    is incorrect. Choice (C) is incorrect. In Lucas v. South Carolina Coastal Council, 505
    U.S. 1003 (1992), the Court stated that a taking occurs where “regulation denies
    all economically beneficial or productive use of the land.” Here, 1,000 acres of land
    owned by the timber company has been condemned, thus depriving the company
    of all economic value. Choice (D) is not the best answer. Once the government has
    decided to deprive the owner of all economically beneficial or productive use of the
    land, the government must render just compensation.
84
Q
  1. A corporation under the authority of a state statute, sued to have condemned 1,000 acres of forested land owned by a tree company. The corporation intended to use the land to develop physical endurance and obstacle courses meant to train private security firms and forest rangers. After a hearing, the state court ordered possession of the land surrendered to the corporation upon deposit in court of a sum deemed adequate to cover damages that might be awarded. The tree company immediately commenced an action to enjoin the court-ordered sale of its property.
    Assume that the tree company was not given any notice of the condemnation proceedings by the appropriate state authorities. The tree company’s best argument for challenging the validity of the condemnation proceedings would be
    (A) violation of procedural due process.
    (B) violation of substantive due process.
    (C) unlawful delegation of legislative power because the state’s legislature had no authority to delegate power to a private enterprise for eminent domain.
    (D) the tree company is entitled to a judicial or administrative proceeding in order that the amount of compensation may be determined prior to any “taking.”
A
  1. (A) In addition to the requirements thatthe taking of private property be for a public use and just compensation be given, the condemnee must be given adequate notice and a fair hearing as required by the Due Process Clause of the 14th Amendment. Choice (B) is incorrect. Under the 14th Amendment’s Substantive Due Process Clause, the Supreme Court has identified a set of rights that are organized around bodily autonomy, especially as these relate to reproductive rights, and those rights pertaining to the organization of the family. The tree company cannot argue that such rights are being infringed. Choice (C) is incorrect. The power of eminent domain may be delegated directly or indirectly to a private person or enterprise, subject to the requirements that the taking be (a) for a public use, and (b) just compensation be given. Choice (D) is incorrect. A condemnee is not entitled, as a constitutional matter, to a judicial or administrative proceeding in order to determine the amount of compensation.
85
Q
  1. A state’s constitution reserves to the people of each municipality in the state the power of referendum with respect to all questions that the municipality is authorized to control by legislation. A real estate developer applied for a zoning change to permit construction of a convalescent home on land he owned in a city within the state. While the application was pending, the city charter was amended by popular vote so as to require that any changes in land use agreed to by the city council be approved by a 55% vote in a referendum. The city planning commission and the city council both approved the proposed zoning change. However, the real estate developer’s subsequent application for approval for a proposed home on the grounds was rejected because the rezoning action had not been submitted to a referendum.
    In an action brought in state court seeking a judgment declaring the city charter amendment invalid, the court will most likely declare the amendment
    (A) unconstitutional, as violation of the real estate developer’s due process rights.
    (B) unconstitutional, as an unlawful delegation of legislative power to a regulatory body.
    (C) constitutional, as a valid exercise of the city’s police power.
    (D) constitutional, as a valid exercise of a power reserved by the people to themselves.
A
  1. (D) The amendment to the city charter constituted a valid exercise of power reserved by the people to themselves. Similarly, in Eastlake v. Forest City Enterprises, Inc., 426 U.s. 668 (1976), the U.S. Supreme Court held that a city charter provision requiring proposed land use changes to be ratified by 55% of the votes cast did not violate the due process rights of the 14th Amendment of a landowner who applied for a zoning change. Interestingly enough, the Supreme Court further noted that the amendment to the city charter did not involve a delegation of power by the legislature to a regulatory body. Although the Court did not enumerate the applicable constitutional amendment in reaching its decision, apparently the 10th Amendment acted as the basis for the Court’s ruling. Choices (A), (B), and (C) are incorrect. In Eastlake v. Forest City Enterprises, Inc., 426 U.S. 668 (1976), the Supreme Court announced: “A referendum, which is a means for direct political participation by the people, allowing them what amounts to a veto power over legislative enactments, cannot be characterized as a delegation of power. In establishing legislative bodies, the people can reserve to themselves power to deal directly with matters that might otherwise be assigned to the legislature, and here the power of referendum was specifically reserved to the people under the state constitution.” Accordingly, the state was not deemed to have “taken” the plaintiff’s property; the “people” were presumed to have done so. We have similar facts in our example.
86
Q
  1. A state’s constitution reserves to the people of each municipality in the state the power of referendum with respect to all questions that the municipality is authorized to control by legislation. A real estate developer applied for a zoning change to permit construction of a water park on land he owned in a city within the state. While the application was pending, the city’s charter was amended by popular vote so as to require that any changes in land use agreed to by the city council be approved by a 55% vote in a referendum. The city planning commission and the city council both approved the proposed zoning change. However, the commission rejected the developer’s subsequent application for “recreational area” approval for the proposed water park on the grounds that the council’s rezoning action had not been submitted to a referendum.
    Assume that while the action was pending in the state court, the proposed zoning change was defeated in a referendum. Which would be the most applicable rule of constitutional law with respect to the referendum procedure when applied to a rezoning ordinance?
    (A) The referendum procedure as a basic instrument of the democratic process does not violate the due process clause of the Fourteenth Amendment.
    (B) The referendum procedure is arbitrary and capricious and thus should be held invalid as an unlawful delegation of legislative power.
    (C) The referendum procedure is violative of the due process clause of the Fourteenth
    Amendment.
    (D) The referendum procedure in this context is invalid as against public
    policy.
A
  1. (A) In like manner, the Court in Eastlake held that the referendum process does not,
    in itself, violate the Due Process Clause of the 14th Amendment when applied to a
    rezoning ordinance. Choices (B) and (C) are incorrect. In Eastlake v. Forest City Enter
    prises, Inc., 426 U.S. 668 (1976), the Supreme Court announced: “A referendum,
    which is a means for direct political participation by the people, allowing them what
    amounts to a veto power over legislative enactments, cannot be characterized as
    a delegation of power. In establishing legislative bodies, the people can reserve to
    themselves power to deal directly with matters that might otherwise be assigned to
    the legislature, and here the power of referendum was specifically reserved to the
    people under the state constitution.” Hence, the Court stated that the people—not
    the state—was acting, and there was nothing “arbitrary” or “capricious” about the
    referendum process. Our facts are similar. Choice (D) is incorrect because it is not
    clear how the referendum procedure is against public policy. The Supreme Court
    has held as constitutionally permissible such referendums [Eastlake v. Forest City
    Enterprises, Inc., 426 U.S. 668 (1976)].
87
Q
  1. As a result of the increasing northward movement of the population from a nearby city, a town enacted a zoning ordinance restricting present and future land use in the town to single family dwellings, except in the downtown commercial shopping area, and except for a small area in which multifamily dwellings not over 40 feet in height were permitted. In addition, all new construction or exterior modifications of existing buildings required prior approval of an aesthetic control board. The zoning ordinance defined the term “family” in the context of single family dwellings to mean only one or more persons related by marriage, blood, or adoption, thereby excluding unrelated, unmarried persons from residing in a single residence unit. The enabling legislation contained the following provision:
    “This enactment is necessary to preserve the physical and social homogeneity of this community, and preserve and protect the quality of life of its citizens.”
    A town resident owns a single family residence with five bedrooms. Prior to the enactment of the zoning ordinance, the resident leased the premises on a month-to-month basis to a boyfriend and girlfriend and three other male law students. The five individuals are all unmarried and attend the nearby law school. The resident now brings suit to enjoin enforcement of the ordinance against himself.
    Which of the following is the most accurate statement regarding the constitutionality of the zoning ordinance provision that prohibits unrelated, unmarried persons from residing in a single family residence?
    (A) The zoning ordinance provision would be declared unconstitutional as violative of the due process clause of the Fourteenth Amendment.
    (B) The zoning ordinance would be declared unconstitutional as violative of the equal protection clause of the Fourteenth Amendment.
    (C) The zoning ordinance provision would be declared unconstitutional as violative of the resident’s rights under the contract clause.
    (D) The zoning ordinance provision would be upheld as constitutional under the state’s police power.
A
  1. (D) In Village of Belle Terre v. Boraas, 416 U.S. 1 (1974), the U.S. Supreme Court upheld a similar ordinance that prohibited unrelated, unmarried persons from residing in the same dwelling unit. The Court held that a municipality can enact such an ordinance under its police powers, since the ordinance bears a substantial relationship to the health, safety, welfare, and morals of the citizenry. Students should note, however, that a zoning or housing ordinance that limits occupancy of dwelling units to members of a single family and recognizing as a “family” only a few categories of related individuals is violative of the Due Process Clause of the 14th Amendment. In Moore v. City of East Cleveland, 97 S.Ct. 1932 (1977), the U.S. Supreme Court declared such an (above-mentioned) ordinance unconstitutional, whereby a grandmother was charged with a crime for permitting her grandson to reside in her single family residence. Choice (A) is incorrect. Under the liberty interest of the 14th Amendment’s Due Process Clause, the Supreme Court has recognized a fundamental right to organize one’s family. [Moore v. City of East Cleveland, 431 U.S. 494 (1977)]. Thus, “unrelated, unmarried persons” are not protected by such rights. Choice (B) is incorrect. The equal protection clause provides heightened protection for groups whose legal classifications are suspect (e.g., classifications based on race, ethnicity, nationality, and religion) and for groups whose legal classifications are quasi suspect (e.g., gender, illegitimacy). “Unrelated, unmarried persons” may have been the subject of discrimination by the legislature, but they are not members of a suspect or quasi-suspect class. Only rational review will be applied, which the government probably will pass with ease. Finally, choice (C) is incorrect. The contracts clause of Article I, Section 10 states that no state shall impairthe obligation of contracts. Here, the ordinance does not impair the obligation of contract because of the following. One, the contract took place against the knowledge of both parties that their contract could be subject to heavy governmental regulation. [Energy Reserves Group, Inc. v. Kansas Power & Light Co., 459 U.S. 400 (1983)]. Two, both parties still have remedies (The resident is entitled to evict the renters; the renters are entitled to return of the rental money).
88
Q
  1. A state has a statute requiring all candidates for lieutenant governor to file a petition with the signatures of 5,000 registered voters in order to have their names placed on the ballot. A candidate for lieutenant governor, did not receive the authorized number of signatures and did not have her name placed on the ballot.
    If the candidate brings an appropriate action challenging the constitutionality of the voter petition requirement, the best argument, if established, in support of the statute is that
    (A) it is constitutional under the Fifteenth Amendment.
    (B) it is necessary to further a compelling state interest.
    (C) the candidates’s petition contained 2,000 false signatures.
    (D) only one other candidate successfully petitioned to have his name placed on the ballot.
A
  1. (B) Choice (B) is the correct answer. While choice (C) is a correct statement of fact, a correct statement of law is always preferred. Although a state may have a legitimate interest in creating a so-called support requirement on independent candidates, you must ask yourself, “What is the state’s standard interest in enacting such a statute?” According to the rule enunciated in Williams v. Rhodes, 393 U.S. 23 (1968), there are three state interests protected: (1) the statute promotes the two-party system; (2) the statute helps to prevent voter confusion; and (3) the law ensures that the voters will elect a candidate with a majority vote. Choice (A) is incorrect because the 15th Amendment provides that the right of citizens of the United States to vote shall not be denied or abridged by the United States or by any State on account of race, color, or previous condition of servitude. Choice (D) is incorrect because even if this were factually true, it would not support the extension of some legal argument.
89
Q
  1. A consumer watchdog group presented petitions to the state legislature bearing the signatures of over 10,000 state residents complaining about the recent increases in the cost of cable television. A corporation who provides cable television services successfully persuaded the state legislature to grant it an exclusive right to install cable television lines in all multiple family dwellings in the state, in exchange for the corporation’s promise to freeze cable television rates for the next four years. An owner of several large multifamily apartment buildings in the state brought an action in federal district court challenging the constitutionality of the state legislation. The suit claimed that the space used by the corporation when it subsequently installed cable television lines in one of his apartment buildings amounted to a taking without compensation.
    In this action, the owner will be awarded
    (A) no relief, because easements for utility lines are presumed to be beneficial to the servient estate.
    (B) no relief, because the legislation is merely a regulation of the use of property and not a taking.
    (C) no relief, because the corporation is not a government entity.
    (D) damages for the value of property used by the corporation.
A
  1. (D) A permanent physical occupation of private property by the government or a government regulation that allows someone other than the property owner to have permanent physical occupation of a definabLe part of a piece of property should constitute a taking. Nowak, Constitutional Law, pg. 450. In Loretto v. Teleprompter Manhattan CAWC0rp., 458 U.S. 419 (1982), a city ordinance requiring the landlord-building owner to allow the installation of a cable television receiver on the apartment building and denying the landlord the ability to demand payment in excess of $1 constituted a compensable taking because the ordinance allowed for “permanent physical occupation” of a small part of the building. Choices (A) and (B) are incorrect for the reasons stated above. Choice (C) is incorrect. It does not legally matter if the corporation is not a governmental entity. If the government legally permits the corporation to take another’s property for public use, the government has engaged in a taking that will require just compensation.
90
Q
  1. In which instance would a state, under the enabling clause of the Fourteenth Amendment, be most able to regulate?
    (A) A private individual from discriminating against a person based on race.
    (B) A private individual from discriminating against a person based on nationality.
    (C) A state official from discriminating against a person based on race.
    (D) A federal official from discriminating against a person based on nationality.
A
  1. (C) The most clear case where a state can regulate to protect one’s constitutional rights under the 14th Amendment occurs when state action is involved. Thus, choices (A) and (B) are incorrect because private discrimination is involved. Choice (D) is wrong because a federal official is doing the discriminating. ConsequentLy, choice (C) is the best answer because a state is most able to regulate the actions of a state official when these actions violate one’s constitutional rights.
91
Q
  1. A president of the senior class at a public high school, on his own initiative instituted among the students the practice of invoking divine blessing at the start of the daily luncheon served in the high school cafeteria. Although no teacher or school official either encouraged or discouraged the practice, all but two teachers, when present, joined in the invocation. A student and his parents protested to the school superintendent, but they refused to intervene on the grounds that the matter was entirely up to the students. Moreover, school board officials pointed out that the students who led the recital always sat at a table in the rear of the cafeteria and that no one was required to participate in the blessing.
    In an action by the student’s parents to enjoin the daily luncheon invocation at the high school, the court will most likely
    (A) grant relief, since the invocation violates the establishment clause of the First Amendment.
    (B) grant relief, since the primary effect of the invocation is to advance religious beliefs.
    (C) deny relief, since the lunch hour is not part of the educational process.
    (D) deny relief, since the noncompulsory nature of the invocation would not be violative of the establishment clause.
A
  1. (A) Such voluntary pupil prayer recitals during school hours in the school building have been invalidated as an establishment of religion, despite the fact that no religious sect was preferred or discriminated against, thus in violation of the Establishment Clause of the 1st Amendment and applicable to the states through the Due Process Clause of the 14th Amendment. Choice (B) is incorrect. This may be true but in itself it is not a legal claim. Choice (A) provides such a Legal claim. Choice (C) is incorrect because even if this were true, the religious expression is taking place on governmental property and thus still in danger of violating the Establishment Clause of the 1st Amendment, as applied to locaL governments via the 14th Amendment’s Due Process Clause. Choice (D) is incorrect. Even if noncompulsory, the Supreme Court has struck down practices where a school has, through its religious practices, inadvertently and informally made nonbelievers feel “coerced” into complying with the religious conduct [Lee v. Weisman, 505 U.S. 577 (1992)]. Furthermore, the traditional establishment clause test from Lemon v. Kurtzman, 403 U.S. 602 (1971) only requires “excessive entanglement,” not compulsion.
92
Q
  1. A state enacted a statute that authorized the payment of state funds to any private institution of higher learning within the state that meets certain minimum criteria and refrains from awarding only seminarian or theological degrees. The aid is in the form of annual subsidies to qualified institutions. The grants are non-categorical but may not, under an added provision, be utilized by the institutions for sectarian purposes. A state agency determines the eligibility of applicant institutions. Eight state taxpayers, all avowed atheists, bring suit in federal court to enjoin the payments of subsidies under the statute to all church-affiliated institutions in the state.
    The federal district court will most likely
    (A) dismiss the action for lack of standing.
    (B) uphold the validity of the statute as nonviolative of the establishment clause.
    (C) invalidate the statute as violative of the establishment clause.
    (D) uphold the validity of the statute as a valid exercise of the state’s power to subsidize education.
A
  1. (B) Applying the three-part requirement of Lemon v. Kurtzman, 403 U.S. 602 (1971), state aid such as this must have (1) a secular purpose; (2) a primary effect other than the advancement of religion; and (3) no tendency to entangle the state excessivelyin church affairs. The New Haven statute should be upheld as nonviolative of the Establishment Clause of the 1st Amendment [Roemer v. Board of Public Works of Maryland, 426 U.S. 736 (1976)]. Moreover, it is important that the church-affiliated institutions referred to in this problem must not be so “pervasively sectarian” under Hunt v. McNair, 413 U.S. 734 (1973), that (1) secular activities cannot be separated from sectarian ones, and (2) if secular activities can be separated out, they alone may be funded. Choice (A) is incorrect. We have the requirements for standing: First, we have the danger of imminent injury insofar as the state may violate the Establishment Clause. Second, there’s causation because the state is directly responsible for the potential injury. And plaintiffs are able to show that the state can remedy their harm by refraining from giving money to the schools. Choice (C) is incorrect applying the Lemon test. Finally, choice (D) is incorrect. This answer is unresponsive to the Establishment Clause issue. States do have 10th Amendment poLice powers to regulate matters of health, safety, welfare, and morals, but they must not do so in a way that violates the Establishment Clause.
93
Q
  1. A city has adopted ordinance 172 which provides:
    “Section 1: It shall be unlawful for any person, group, or organization to hold a meeting of 50 persons or more in any city park without first securing a city permit;
    Section 2: The application shall specify the day and hours for which the permit is sought. The fee shall be $10 per hour, with a maximum fee of $50;
    Section 3: Permits shall be issued on a first come basis; provided that the police chief shall deny any application if, after hearing the applicant, it is his considered judgment that (a) the meeting would create serious traffic congestion, or (b) interfere with public enjoyment of the park, or (c) speakers at the meeting would advocate the commission of crime.”
    A foreign exchange student planned to hold a demonstration at park in the city. Although the student’s previous protest rallies attracted fewer than 25 demonstrators, he decided to apply for a permit pursuant to city ordinance 172. After meeting with the student, the chief of police denied his permit application because he believed that the demonstration would incite the protestors and threaten imminent violence or serious disorder.
    Subsequently, the student and his fellow demonstrators staged their rally at the park. The rally attracted only about 20 protestors and was conducted peacefully. As the student was making his final remarks to the gathering, the police arrived at the park and arrested the student and his fellow demonstrators charging them with violating ordinance 172.
    Which of the following would be the most accurate statement with regard to the arrest of the student and the other demonstrators?
    (A) The police were justified in arresting the student, because he and his fellow demonstrators violated the ordinance by staging the rally.
    (B) The police were justified in halting the rally and arresting the demonstrators, because of the threat of imminent physical disturbance.
    (C) The police were not justified in arresting the student and the demonstrators, because they did not violate the ordinance.
    (D) The police were not justified in arresting the student, because the group was not required to comply with the ordinance.
A
  1. (C) Section 1 of the Doral City Ordinance 172 provided that demonstrations of 50 persons or more may not be conducted without first securing a permit from the chief of police. Since the student’s demonstration attracted fewer than 50 demonstrators, there was no violation of the statute. For this reason choice (C) is correct, and choices (A) and (D) are incorrect. Choice (B) is incorrect. There was no evidence that the speaker presented a “clear and present danger,” in which case he would not have been entitled to any constitutional protection. In Brandenburg v. Ohio, 395 U.S. 444 (1969), the Supreme Court stated that a clear and present danger exists if:
    (1) the speaker advocates imminent unlawful conduct, and (2) it is likely that such imminent unlawful conduct will therefore occur.
94
Q
  1. A city has adopted the following ordinance which provides:
    “Section 1: It shall be unlawful for any person, group, or organization to hold a meeting of 50 persons or more in any city park without first securing a city permit;
    Section 2: The application shall specify the day and hours for which the permit is sought. The fee shall be $10 per hour, with a maximum fee of $50;
    Section 3: Permits shall be issued on a first come basis; provided that the chief of police shall deny any application if, after hearing the applicant, it is his considered judgment that the meeting would create serious traffic congestion.”
    An anarchist planned to hold an antigovernment protest demonstration at a city square. Although his previous antigovernment protest rallies attracted fewer than 15 demonstrators, he decided to apply for a permit pursuant to the city ordinance. After meeting with the anarchist, the chief of police denied his permit application because he believed that the demonstration would incite the protestors and threaten imminent violence or serious disorder.
    On October 4, the anarchist and his fellow demonstrators staged their antigovemment protest at a city park. The rally attracted only about 17 protestors and was conducted peacefully. As the anarchist was making his final remarks to the gathering, the city police arrived at the park and arrested him and his fellow demonstrators, charging them with violating the city ordinance.
    In a subsequent action by the anarchist in state court challenging the constitutionality of Sections 1 and 2 of the city ordinance, the court will most likely rule these sections
    (A) constitutional, because a state may regulate the communicative impact of speech related conduct.
    (B) constitutional, because the aforementioned sections are within the ambit of the state’s police power.
    (C) unconstitutional, because the statute is vague and overbroad.
    (D) unconstitutional, because the statute is a content based regulation of
    speech.
A
  1. (B) Sections 1 and 2 of the ordinance should be upheld as constitutional as within the ambit of a state’s police powers to regulate the health, safety, and welfare of its citizens. Note that a state may control parades, processions, and other gatherings in public places by narrowly drawn requirements pertaining to the time, place, size of the group, and duration in the interest of public safety and convenience [Cox v. Louisiana, 379 U.S. 537 (1965)]. However, in this regard, no public official can constitutionally be given unbridled discretion to grant or withhold a permit or license for a parade or assembly as he sees fit. It is important to point out that Section 1, by itself, is a valid exercise of a state’s police powers. Choice (A) is incorrect. It is true that a state may regulate conduct, as well as those aspects of symbolic speech that are imbued with conduct. However, Choice (B) is better insofar as it explicitly articulates the constitutional basis for such state regulation in the state’s 10th Amendment police powers. Choice (C) is incorrect. A court may declare a law to be void-for-vagueness if a reasonable person is unable to determine whether she will be prosecuted under the statute; such laws are deemed to have a “chilling effect” on speech. A law is deemed invalid for overbreadth if the law prohibits speech that is otherwise protected. Neither seems to be the case here. Finally, choice (D) is incorrect. A law that is content-based will be subject to strict scrutiny. Content- based laws discriminate against speech based on its subject matter. That does not appear to be the case here because the law prohibits speech on the basis of secondary considerations about traffic congestion and the like, not subject matter.
95
Q
  1. A city has adopted the following ordinance which provides:
    “Section 1: It shall be unlawful for any person, group, or organization to hold a meeting of 50 persons or more in any city park without first securing a city permit;
    Section 2: The application shall specify the day and hours for which the permit is sought. The fee shall be $10 per hour, with a maximum fee of $50;
    Section 3: Permits shall be issued on a first come basis; provided that the chief of police shall deny any application if, after hearing the applicant, it is his considered judgment that (a) the meeting would create serious traffic congestion, or (b) interfere with public enjoyment of the park, or (c) speakers at the meeting would advocate the commission of crime.”
    A religious fundamentalist who believes that other religions should be violently resisted, planned to hold a protest demonstration against other religions at a city park on July 25. Although his previous anti-religion protest rallies attracted fewer than 25 demonstrators, he decided to apply for a permit pursuant to the city’s ordinance. After meeting with the fundamentalist, the chief of police denied his permit application because he believed that the demonstration would incite the protestors and threaten imminent violence or serious disorder.
    On July 25, the fundamentalist and his fellow demonstrators staged their protest at a city park. The rally attracted only about 20 protestors and was conducted peacefully. As he was making his final remarks to the gathering, the city police arrived at the park and arrested him and his fellow demonstrators charging them with violating the city ordinance.
    If the fundamentalist now brings suit to challenge the constitutionality of Section 3 of the city ordinance, his best argument would be that
    (A) the section is void for vagueness and overbreadth.
    (B) the section is content based.
    (C) the section permits the police chief to exercise unbridled discretion in approving or disapproving permits.
    (D) the section does not fall within the area of compelling state interest.
A
  1. (C) Section 3 would be declared unconstitutional since no public official may exercise unbridled discretion in the issuance of parade or demonstration permits. Choice (A) is incorrect. A court may declare a law to be void-for-vagueness if a reasonable person is unable to determine whether she wilt be prosecuted under the statute; such laws are deemed to have a “chilling effect” on speech. A law is deemed invalid for overbreadth if the law prohibits speech that is otherwise protected. Neither seems to be the case here. Choice (B) is incorrect. A law that is content-based will be subject to strict scrutiny. Content based laws discriminate against speech based on their subject matter. That does not appear to be the case here because the law prohibits speech on the basis of secondary considerations about traffic congestion and the like, not subject matter. Choice (D) is incorrect. The reference to “compelling state interest” implies that the city’s ordinance will be subject to strict scrutiny. But as choice (C) states, the ordinance will be declared unconstitutional on its face for giving too much discretion to the government to deny permits.
96
Q
  1. A state is the only state that imposes a tax on the extraction of pitchblende. This black-colored mineral consists of massive uraninite and contains radium, which is the chief ore-mineral source of uranium. Minute quantities of plutonium are also found in pitchblende. This is particularly significant because plutonium undergoes slow disintegration with the emission of a helium nucleus to form uranium 235, and that is fissionable with slow neutrons to yield atomic energy. As such, pitchblende is vital to the economy of the entire country.
    Congress has recently enacted a statute forbidding any state from imposing a tax on the extraction of pitchblende. Because pitchblende is not mined in any other state, this federal legislation affects only the state. Thus, in practice, this federal law only limits the taxing power of the state.
    In light of the Constitution and this federal law, the state extraction tax on pitchblende is most likely
    (A) invalid, because when Congress exercises its plenary power over interstate commerce, the supremacy clause voids inconsistent state action.
    (B) invalid, because Congress may use its general welfare power to prohibit state legislation that it deems harmful to the nation as a whole.
    (C) valid, because Congress does not have the authority to interfere with the taxing policies of a state.
    (D) valid, because Congress may not enact a law that places one state on an unequal footing with other states.
A
  1. (A) Congress has complete power to permit or forbid state taxation affecting interstate commerce. Where Congress is silent, the Court reviews nondiscriminatory state taxation by balancing the state revenue needs against the burden on interstate commerce. If, however, a state regulation of interstate commerce conflicts with a federal regulation, the state law is invalid under the preemption doctrine. To be sure, Congress, in the exercise of its plenary commerce power, may prohibit a specific form of state regulation. Choices (B), (C), and CD) are incorrect. Article I, Section 8, reads: “The Congress shall have Power to lay and collect Taxes, Duties, Imposts and Excises, to pay the Debts and provide for the common Defense and general Welfare of the United States.” Congress is not levying any taxes in our example. It is, rather, forbidding the states from levying taxes. Congress can best accomplish this prohibition through the Commerce Clause.
97
Q
  1. The Native American Party was founded and chartered in a state to promote the political power of Native Americans. Members pledged themselves to vote only for candidates nominated by the party. At first, membership was open to any voter who pledged himself to those tenets, but after a defeat in an election, the Party expelled all non- Native-American members upon the grounds that experience showed that only Native Americans could be trusted to honor the obligation to vote only for the Party’s nominees. Membership is currently confined to Native Americans.
    In every election since, the Party’s nominees have easily won election in the Sixth Congressional District, an area encompassing a large number of outlying reservations. Subsequently, the Executive Committee proposes that the Party choose its own “Native American Power” candidate for governor of South Dakota in the upcoming election. It will put its slate for U.S. representatives, governor, and U.S. senator before Party members for mail ballot, along with any rival candidates who qualify for the poll. A former non-Native American member of the Party is now challenging his exclusion from the Party.
    Which of the following constitutional provisions would furnish the former member’s best ground for challenging his exclusion from the Party?
    (A) The due process clause of the Fourteenth Amendment.
    (B) The right of assembly as guaranteed by the First Amendment.
    (C) The equal protection clause of the Fourteenth Amendment.
    (D) The voting provisions of the Twenty Fourth Amendment.
A
  1. (C) n Terry v. Adams, 345 U.S. 461 (1953), the State of Texas was held responsible for racial discrimination in the conduct of primary elections that it closely supervised, and which were important in determining who was ultimately elected. Similarly, in the present hypo, the State of South Dakota would be held responsible for the Native American Party’s expulsion of all non-Native-Americans (from membership within the party). Clearly, in this regard, restricting membership to only Native Americans would be racial discrimination (e,g., by “suspect classification”) and, therefore, violative of the Equal Protection Clause of the 14th Amendment, as well as the 15th Amendment. Choice (A) is incorrect. The right to run for public office is not a fundamental right recognized under the 14th Amendment’s Due Process Clause. Choice (B) is incorrect. The 1st Amendment’s Right to Assemble generally refers to the right to organize in public places for the peaceful, lawful expression of political views. This is not necessarily a bad choice, but choice (C) is more precise. Choice (D) is incorrect. The 24th Amendment contains in Section 1 the following language: “The right of citizens of the United States to vote in any primary or other election for President or Vice President, for electors for President or Vice President, or for Senator or Representative in Congress, shall not be denied or abridged by the United States or any state of failure to pay any poll tax or other tax.” There is no poll tax or other tax in our example.
98
Q
  1. The Latino American Party was founded and chartered in a state to promote the political power of Latino Americans. Members pledged themselves to vote only for candidates nominated by the party. At first, membership was open to any voter who pledged himself to those tenets, but after the defeats in a recent election, the Party expelled all nonLatino members upon the grounds that experience showed that only Latinos could be trusted to honor the obligation to vote only for the Party’s nominees. Membership is currently confined to Latinos.
    In every election since, the Party’s nominees have easily won election in their Congressional District. Subsequently, the Executive Committee proposes that the Party choose its own Latino candidate for governor of New Mexico in the upcoming election. It will put its slate for U.S. representatives, governor, and U.S. senator before Party members for mail ballot, along with any rival candidates who qualify for the poil.
    In an action by non-Latino citizens in the relevant
    Congressional District to enjoin the Latino American
    Party from conducting the mail ballot primary, the
    federal court will most likely
    (A) grant relief, because the primary would be proscribed by the Thirteenth Amendment.
    (B) grant relief, because the primary would be proscribed by the Fifteenth Amendment.
    (C) deny relief, because the Party’s primary would be private action and not subject to restriction under the Constitution.
    (D) deny relief, because a pre-primary election is not within the scope of federal election control.
A
  1. (B) Following the holding in Terry v. Adams, 345 U.S. 461 (1953), the court would enjoin the Latino American Party from conducting its mail ballot primary as a flagrant abuse of the 15th Amendment. Under the 15th Amendment, the right to vote cannot be denied by the evasive device of a party primary conducted as an activity of a “private club,” if such primary is a “feeder” election into the election system. Therefore, choice (B) is correct, and choice (C) is not the best answer. Choice (A) is incorrect. The 13th Amendment states: “Neither slavery nor involuntary servitude, except as a punishment for crime whereof the party shall have been duly convicted, shall exist within the United States, or any place subject to their jurisdiction.” There are no issues of involuntary servitude in our example. Choice (D) is incorrect. In Terryv. Adams, 345 U.S. 461 (1953), the Court held that a party primary conducted as an activity of a “private club” would still be considered state action if such a primary is a “feeder” election into the election system. Accordingly, such primaries must adhere to the federal Constitution.
99
Q
  1. A state election code provides that any political organization whose candidate received 20% or more of the vote at the most recent gubernatorial or presidential election is a political party; other political organizations are construed as political bodies. Political parties conduct primaries, and the winning candidates in each office may have their names placed on the ballot. However, nominees of political bodies or any independent candidate may only have their names placed on the ballot if they file a nominating petition signed by not less than 7% of those eligible to vote at the last election for the offices that they are seeking. The time for circulating the petition is 180 days, which is also the deadline governing party candidates in party primaries.
    An independent candidate, who desires to be listed on the ballot for the office of governor, challenges the constitutionality of this election law.
    The court will most likely declare this statute
    (A) unconstitutional, because it is a violation of the First and Fourteenth Amendments’ rights of free speech.
    (B) unconstitutional, because it is a violation of the Fourteenth Amendment’s due process clause.
    (C) unconstitutional, because it violates the Fifteenth Amendment’s voting provisions.
    (D) constitutional, because the election code is nonviolative of the equal protection clause.
A
  1. (D) Choice (A) is incorrect. Injenness v. Fortson, 403 U.S. 431 (1971), the U.S. Supreme Court upheld a similar Georgia statute requiring an independent candidate to file a nominating petition signed by 5% of the electorate as nonviolative of the Equal Protection Clause of the 14th Amendment. The Court sustained the voting statute on the grounds that a candidate had alternative routes to getting his name on the ballot, namely, either by entering the primary of a political party or circulating a nominating petition; therefore, no abridgement of constitutional rights of free speech and association under the 1st and 14th Amendments, nor violation of the equal protection cLause of the 14th Amendment. Choice (B) is incorrect because the right to run for public office is not a fundamental right under the 14th Amendment’s due process clause. Choice (C) is incorrect. The 15th Amendment states: “The right of citizens of the United States to vote shall not be denied or abridged by the United States or by any state on account of race, color, or previous condition of servitude.” In our example, there is no such discrimination.
100
Q
  1. A state labor code provided that no employer in the state shall knowingly employ an alien who is not lawfully residing in the United States if such employment would have an adverse effect on lawful residents’ right to work. A group of immigrant farm workers were continually refused employment contracts by the labor contractors in the state. Instead, the labor contractors had employed many alien workers since they would toil longer hours for less wages. The immigrant farm workers now bring suit in state court pursuant to the above-mentioned statutory provision.
    Which of the following determinations would most likely result as a consequence of the immigrant farm workers’ lawsuit?
    (A) The court would declare the statute unconstitutional as violative of the equal protection clause of the Fourteenth Amendment.
    (B) The court would declare the statute constitutional as within the realm of rights reserved to the states by the Eleventh Amendment.
    (C) The court would declare the statute unconstitutional since the regulation of immigration is preempted.
    (D) The court would declare the statute constitutional since the states are not preempted in the area of economic regulation of illegal aliens.
A
  1. (D) In DeCanas v. Bica, 424 U.S. 351 (1976), the U.S. Supreme Court held a California Labor Code statute that “prohibits an employer from knowingly employing an alien who is not entitled to lawful residence in the U.S. if such employment would have an adverse effect on lawful resident workers” not to be unconstitutional as a regulation of immigration or as being preempted. The Court further noted that it is clearly within a state’s police powers under the 10th Amendment to regulate the employment relationship of illegal aliens. Choice (A) is incorrect. The equal protection clause provides heightened protection for groups whose legal classifications are suspect (e.g., classifications based on race, ethnicity, nationality, and religion) and for groups whose legal classifications are quasi suspect (e.g., gender, illegitimacy). The illegal immigrants may have been the subject of discrimination by the legislature, but they are not members of a suspect or quasi-suspect class. Only rational review will be applied, which the state probably will pass with ease. Choice (B) is incorrect. The 11th Amendment states: “The judicial power of the United States shall not be construed to extend to any suit in law or equity, commenced or prosecuted against one of the United States by citizens of another state, or by citizens or subjects of any foreign state.” In our example, no one is suing the federal government.
101
Q
  1. In a secluded county, where prejudice festers and discrimination flourishes, there is a lovely lake, which the county has developed and maintained for recreational purposes. Although it is not the only lake in the county, it is the largest and most scenic, and it attracts visitors from miles around. One of its biggest assets is the excellent fishing and boating, which is available to the public at large.
    Three years ago, in order to enhance the recreational aspects of the lake, the county leased a sizable portion of the lake and surrounding parkland to a company owned by the most prominent family in the county. The lease required the company to construct and operate a first-rate yacht house and club, complete with bar, restaurant, and private marina, and to pay the county 10% of its net profits as rent. The company set up bylaws, which were reviewed and approved by the county at the time the lease was negotiated. According to the bylaws, the yacht club, complete with its restaurant and bar, would be open to members only, and the membership committee was empowered to set up strict membership “standards,” as well as the cost of membership fees and dues.
    Upon completion of the facilities, the state granted the company a license to sell alcoholic beverages in its restaurant and bar. The membership committee announced that the membership fee was $5,000 and the monthly dues $75. Furthermore, the membership committee had a policy of approving only membership applications from men, while disapproving and denying all applications from women. There were other similar facilities within the county available to women.
    A woman resident of the county brings suit against the company, claiming that her membership application was denied only because she is a woman, and that its policy of excluding women as a group denies her equal protection rights. Which of the following is the most accurate statement?
    (A) The plaintiff will lose, because classifications based on sex have not yet been held to violate the equal protection clause.
    (B) The plaintiff will prevail unless denial of membership to women can be justified by some “compelling interest,” since such discrimination is “suspect” and requires the strictest equal protection test.
    (C) The plaintiff will lose, because other similar facilities are available to women.
    (D) The plaintiff will prevail unless the company can prove some important basis for the exclusion of women.
A
  1. (D) Choice (A) is incorrect because classifications based on sex have been held to violate equal protection. Choice (B) is wrong because such discrimination has not yet been held “suspect” and, therefore, need not be justified by a compelling interest. Choice (C) is incorrect because the existence of similar facilities would not preclude a finding of “state action.” Choice (D) is the correct answer, since discrimination based on sex is subject to a “quasi-suspect” standard of review.
102
Q
  1. A county owns a large expanse of land next to the ocean. Four years ago, in order to enhance the recreational aspects of the land, the county leased most of it to a private company. The lease required the company to construct and operate a first-rate golf course and country club—complete with bar, restaurant, and private marina—and to pay the county 15% of its net profits as rent. The company set up bylaws, which were reviewed and approved by the county at the time the lease was negotiated. According to the bylaws, the golf course and country club, complete with its restaurant and bar, would be open to members only, and the membership committee is empowered to set up strict membership “standards,” as well as the cost of membership fees and dues.
    Upon completion of the facilities, the state granted the company a license to sell alcoholic beverages in its restaurant and bar. The membership committee announced that the membership fee is $5,000 and the monthly dues $75 per month. Furthermore, the membership committee had a policy of approving only membership applications for Latino Men, while disapproving and denying all applications of women, African Americans, white Americans, and other minorities.
    A white resident of the county, upon denial of membership, brings an action against the company seeking injunctive relief to compel his admission claiming that denial of membership to white residents violates his right to equal protection. Which of the following statements is most accurate?
    (A) The company will prevail because its denial of membership lacks the requisite state action.
    (B) The plaintiff will win because even though the company is a privately owned corporation, the state has affirmatively encouraged or facilitated its discriminating acts.
    (C) The company will win, because the plaintiff lacks standing to assert the rights of discrimination against white Americans as a group.
    (D) The plaintiff will win, because denial of membership to white residents cannot be justified by a rational basis.
A
  1. (B) Although the company is a privateLy owned corporation, the state has affirmatively encouraged and facilitated its discriminating acts, thus choice (A) is incorrect because “state action” exists. Choice (C) is an incorrect statement and not applicable to the fact situation. Choice (D) is wrong, since racial discrimination is “suspect” and, therefore, cannot be justified by some “rational basis.” This is true, even if white Americans, a majority race, are the subjects of discrimination. Choice (B) is the strongest answer available.
103
Q
  1. A county owns a large expanse of land next to the ocean. Four years ago, in order to enhance the recreational aspects of this land, the county leased most of it to a company. The lease required the company to construct and operate a first-rate luxury hotel—complete with bar, restaurant, and private marina—and to pay the county 15% of its net profits as rent. The company set up management and operations bylaws for its new hotel, which were reviewed and approved by the county at the time the lease was negotiated.
    Upon completion of the facilities, the state granted the company a license to sell alcoholic beverages in its restaurant and bar. The hotel announced that the least expensive room was $1,000 per night.
    Much of the population in the county cannot afford to stay at the new hotel. One resident who cannot afford to pay the hotel fees brings an action against the company, claiming that the high fees operate to discriminate against the poor, in violation of the constitutional right to equal protection. What is the most likely result of this action?
    (A) The company will lose because social class constitutes a suspect classification under the equal protection clause.
    (B) The company will win, because hotel privileges are not an important or basic enough deprivation, for those unable to pay for them, to be held to violate equal protection.
    (C) The resident will win because all public rights cannot be limited to those who can afford them.
    (D) The resident will win because discrimination against poor people violates the equal protection clause of the Fourteenth Amendment.
A
  1. (B) The equal protection clause provides heightened protection for groups whose legal classifications are suspect (e.g., classifications based on race, ethnicity, nationality, and religion) and for groups whose legal classifications are quasi suspect (e.g., gender, illegitimacy). The poor may have been the subject of discrimination by the legislature, but they are not members of a suspect or quasi-suspect class. Only rational review will be applied, which the county probably will pass with ease. Sometimes, the court may reject a law for violating the equal protection clause if the law denies a very important (albeit not fundamental) right like education. See Plyler v. Doe, 457 U.S. 202 (1982). The right to stay in a luxury hotel is not equivalent to such a right. Choice (A) is incorrect. The equal protection clause provides heightened protection for groups whose legal classifications are suspect (e.g., classifications based on race, ethnicity, nationality, and religion) and for groups whose legal classifications are quasi suspect (e.g., gender, illegitimacy). The poor may have been the subject of discrimination by the legislature, but they are not members of a suspect or quasi- suspect class. Only rational review will be applied, which the county probably will pass with ease. Choice (C) is too general a statement, because not all public rights are protected. Choice (D) is too broad, because not all discrimination against the poor violates equal protection.
104
Q
  1. An indigent man was suffering from a chronic asthmatic and bronchial illness. The man redomiciled to a new state and shortly thereafter suffered a severe respiratory attack and was sent by his attending physician to a nonprofit private community hospital. Pursuant to a state statute governing medical care for indigents, the hospital notified county officials that it had in its hospital an indigent who might qualif’ for county care and requested that the man be transferred to the county’s public hospital facility. In accordance with the approved procedures, the private hospital claimed reimbursement from the county in the amount of $1,069 for the care and services it had provided.
    Under the relevant state statute:
    “Individual county governments are charged with the mandatory duty of providing necessary hospital and medical care for their indigent sick. In order to qualif’ for such hospital and medical care, an indigent shall be resident of the county for the preceding 12 months in order to be eligible for free non-emergency medical care.”
    As a consequence, the county refused to admit the man to its public hospital or to reimburse the private hospital because the man had not been a resident of the county for the preceding year. In an action in federal court against the county challenging the constitutionality of the residency requirement for providing free medical care for indigents, the court will most likely declare the statute
    (A) constitutional, because the statute promotes a compelling state interest.
    (B) constitutional, because the statute is a proper exercise of state action.
    (C) constitutional, because the statute is within the state’s police power to regulate the health, safety, and welfare of its citizens.
    (D) unconstitutional, because it violates the equal protection clause of the Fourteenth Amendment.
A
  1. (D) In Memorial Hospital v. Maricopa County, 415 U.S. 250(1974), the U.S. Supreme Court held that an Arizona statute requiring one year’s residence in a county as a precondition to receiving nonemergency hospitalization or medical care at public expense was unconstitutional as “an invidious discrimination against the poor,” thus violative of the equal protection clause of the 14th Amendment. The Court found that this classification infringed upon interstate travel and in so doing, found that it was irrelevant that the classification also burdened travel by persons within their own state. Under the circumstances, the State of Arizona burdened the right to travel by denying benefits that were essential to the daily life of the new indigent in the state. The Court, in reviewing the state’s justification for this residency requirement, utilized the strict scrutiny test. Therefore, choice (D) is correct, and choice (A) is incorrect. Choice (B) is incorrect. Idiomatically, there is no proper exercise of “state action.” The state action doctrine refers to the idea that the government has acted and is thus theoretically subject to the U.S. Constitution. Choice (C) is incorrect. The state certainly has police powers under the 10th Amendment to regulate issues related to the health, safety, and welfare of its citizens. However, there are limits to what the state may do. The 14th Amendment’s Equal Protection Clause would prohibit the state from denying to the poor essential medical services as described in our example. See Memorial Hospitalv. Maricopa County, 415 U.S. 250 (1974).
105
Q
  1. A state fair is held annually in a county on a large tract of state-owned property. In recent years, many outside organizations have entered the fairgrounds and distributed literature and paraphernalia to the many thousands of patrons visiting the fair. State fair officials did not endorse any of these organizations but permitted them to disseminate their materials throughout the fairgrounds without charge. Lately, however, many families attending the fair have complained about being harassed by canvassers from these various organizations.
    In an effort to protect the safety and welfare of the persons visiting the fair, the state legislature enacted a law prohibiting anyone from selling or distributing materials at the state fair. This new statute provided, however, that groups could pay a $50 license fee and distribute their literature from enclosed booths. These booths would be set up along the entrance to the fairgrounds and rented to anyone wishing to sell or distribute materials or soliciting money during the fair.
    The first year that the statute went into effect, approximately 40 groups rented booth space. There were various organizations paying the $50 license fee. A group of scientists opposed to the use of aerosol spray cans, requested permission to distribute literature at the fairgrounds. The scientists claimed that they simply wanted to warn people of the perils created by the disintegration of the ozone layer from the dispersion of fluorocarbons into the atmosphere. State fair officials offered to lease the scientists a booth at the $50 fee, but refused to permit solicitation activities outside the booth enclosures. The scientists were unwilling to pay the $50 license fee and instituted suit in state court seeking a court order permitting them to distribute literature anywhere in the fairgrounds area.
    Which of the following is the strongest argument in
    support of the constitutionality of the statute?
    (A) The statute applies to the limited area of the state-owned fairgrounds, and does not discriminate among the various organizations by way of their political, religious, or commercial viewpoints.
    (B) The statute applies to representatives of popular organizations, as well as to representatives of unpopular organizations, and is a democratic expression of the will of the people because it was adopted by the state legislature.
    (C) The statute is necessary to protect the safety and welfare of persons using a state facility, and does not discriminate among diverse viewpoints since there is an alternative means by which these organizations can reach their audience.
    (D) The statute protects the patrons of a public facility against unwanted invasions of their privacy by restricting the solicitation activities of those organizations that the patrons do not support.
A
  1. (C) This Constitutional Law question presents some very close answer choices in the area of regulation of 1st Amendment freedom of speech. Regarding noncommunicative aspects of free speech, such as time, place, and manner regulations in public forums, courts will generally uphold reasonable restrictions. Choice (A) is persuasive on this issue. Where speech content is restricted, however, the courts apply a more rigid test whereby a compelling state interest must be justified before government regulation is permitted. Choice (B) addresses this issue, since the restrictions of the Fairgrounds Bill are content-neutral. Choice (D) seeks to balance the rights of free speech against the patrons’ right of privacy. Choice (C), however, presents the strongest argument to support the Fairgrounds Bill. In the area of solicitation, the court uses a balancing test to determine, upon weighing the individual’s rights of free speech against the state’s police powers interest in protecting the safety, welfare, and privacy interests of its citizens, that the challenged measure is reasonable and nondiscriminatory, and that there is no less-drastic alternative means available [Beard v. Alexandria, 341 U.S. 622 (1951)—requirement of homeowner’s consent held a valid restriction on commercial solicitation, whereas a ban on all door-to-door solicitation was found to be too restrictive].
106
Q
  1. A permanent resident alien applied for a position as a state trooper. A state trooper is a member of the state police force, a law enforcement body that exercises broad police authority throughout the state. The position of state trooper is filled on the basis of competitive examinations taken by all of the applicants. After the resident alien applied for the position, the state authorities refused him permission to take the qualifying examination. The state authorities based their refusal on state statute, which provided:
    “No person shall become a member of the state police unless he/she shall be a citizen of the United States.”
    Thus, under this provision, as a prerequisite to becoming a member of the state police, an alien must relinquish his foreign citizenship and become a citizen. In an opinion upholding the validity of the statute, the State Attorney General noted that since police officers fall within the category of important non-elective officials who participate directly in the execution of broad public policy, only citizens of the United States should be qualified to apply for such positions.”
    At the time the resident alien applied for a position as a state trooper, he was a citizen of a foreign country and not currently eligible for citizenship. As a result of a federal statute, Congress has imposed a five-year residency requirement for the attainment of citizenship. Under this federal law, an alien must reside in this country for a period of five years as a prerequisite before applying for citizenship. At this time, the resident alien had only lawfully been residing in the United States for two years, and thus would not be eligible to apply for naturalization until three years later.
    If the resident alien brings suit in federal court challenging the constitutionality of the state statute limiting the membership of its state police force to citizens of the United States, the court will most likely declare the statute
    (A) constitutional, because the statute is within the state’s plenary power to regulate the health, safety, and welfare of its citizens.
    (B) constitutional, because citizenship bears a rational relationship to the special demands of the police function.
    (C) unconstitutional, because it constitutes a violation of the equal protection clause of the Fourteenth Amendment.
    (D) unconstitutional, because it constitutes a violation of the due process clause of the Fourteenth Amendment.
A
  1. (B) In accordance with Foleyv. Connellie, 98 S.Ct. 1067 (1978), the U.S. Supreme Court upheld the validity of a New York statute that limited the membership of the New York State Police Force to U.S. citizens. The Court stated that “in the enforcement and execution of the laws, the police function is one where citizenship bears a rational relationship to the special demands of the particular position.” Thus, in applying the rational basis test, the Supreme Court held that the performance of the police function is an important public responsibility, which can be limited to a particular class (here, U.S. citizens onLy). Students should note that although aliens are extended the right to education and public welfare, along with the ability to earn a livelihood and engage in licensed professions, the right to govern and carry on a governmental function is reserved to citizens only. Therefore, the state police force statutory provision would not be violative of the Equal Protection Clause of the 14th Amendment. Though choice (A) is also correct, choice (B) is the preferred alternative, as it specifies the underlying rationale for upholding the constitutionality of the statute. Choice (C) is incorrect. Generally, classifications by the state that discriminate against lawful aliens are subject to strict scrutiny. Not so here, however. In Foleyv. Connellie, 98 S.Ct. 1067 (1978), the U.S. Supreme Court upheld the validity of a New York statute that limited the membership of the New York State Police Force to U.S. citizens. The Court stated that “in the enforcement and execution of the laws, the police function is one where citizenship bears a rational relationship to the special demands of the particular position.” Thus, in applying the rational basis test, the Supreme Court held that the performance of the police function is an important public responsibility, which can be Limited to a particular class (here, U.S. citizens only). Finally, choice (D) is incorrect because there is no fundamental right to be a police officer.
107
Q
  1. Congress has recently enacted a statute legalizing marijuana. The law, signed by the President, imposes a tax of $1 on each pack of marijuana cigarettes sold in the United States. In an inseverable portion of that same law, the entire proceeds of the tax are appropriated on a continuing basis for direct payments to an art museum. The public museum is dedicated to the collection of pictures, artifacts, weapons, and other historical memorabilia of past wars.
    Which of the following most clearly has standing to attack the constitutionality of this appropriation of the tax monies to the art museum?
    (A) A state, other than the one in which the museum is located, in which several other public museums are located that are not subsidized by this law.
    (B) A nonprofit organization of war veterans that claims it can demonstrate a greater need for the funds than can the museum.
    (C) A purchaser of marijuana cigarettes who is required to pay the tax.
    (D) An association of medical doctors that alleges that the legalization of marijuana will result in a public health hazard.
A
  1. (C) This Multistate question deals with the issue of standing. To satisfy the minimum constitutional requirements imposed by the “case and controversy” limitation of Article Ill, a plaintiff must demonstrate a definite and concrete personal stake in the outcome. First, the plaintiff must show actual injury in fact. Second, she must show causation (namely, that resolution of the grievance in her favor will eliminate the harm alleged). Under these facts, choice (C) is correct because the purchaser of marijuana cigarettes can demonstrate actual injury. Choices (A) and (B) are too remote. Also, the association of doctors would lack standing, since the general rule is against assertion of third-party rights except in limited situations. Choice CD) is incorrect because the doctors lack standing. To satisfy the minimum constitutional requirements imposed by the “case and controversy” limitation of Article Ill, a plaintiff must demonstrate a definite and concrete personal stake in the outcome. First, the plaintiff must show actual injury in fact. The medical doctors cannot show such injury.
108
Q
  1. Congress enacted a statute taxing the sale of automobiles. In an inseverable portion of that same law, the entire proceeds of the tax are appropriated on a continuing basis for direct payments to an education fund. The education fund is dedicated to educating people about the importance of mass transportation as an alternative to automobiles, which the fund considers a major source of pollution.
    As a matter of constitutional law, which of the following statements concerning the continuing federal appropriation to the education fund is most accurate?
    (A) It is constitutional because Congress could reasonably believe that such a subsidy to this particular museum will benefit the cultural life of the nation as a whole.
    (B) It is constitutional because Congress can demonstrate that such a subsidy is rationally related to a legitimate public interest.
    (C) It is unconstitutional because it is not apportioned among the several states on an equitable basis.
    (D) It is unconstitutional because it advances the welfare only of those
    persons who are interested in clean
    air.
A
  1. (A) Choice (A) is correct because Congress can tax an activity, even if such a tax has a regulatory effect, provided that the dominant intent of the tax is fiscal (i.e., revenue-mising). Certainly, Congress has constitutionally taxed bookmakers, guns, and narcotics despite its substantial regulatory effect. In addition, under its spending powers, Congress can spend money (it collects from taxes) to “provide for the common Defense and general welfare of the United States.” In this regard, a reasonable belief by Congress that payment of the tax fund proceeds to the education fund would benefit the cultural life of the nation as a whole would certainly fall within the proper scope of its federal taxing and spending powers. Choice (B) is incorrect. In a sense, the proposition contained in this answer is true. However, choice (A) is better because it references the specific constitutional power that authorizes Congress’s action. Choices (C) and (D) are incorrect because, even if they were factually true, under the Tax and Spend Clause of Article I, Section 8, Congress is not required to apportion its subsidy among the states on an equitable basis. Rather, Congress is given broad discretion in taxing and spending underArticle I, Section 8 for the “general welfare” [United States v. Butler, 297 U.S. 1 (1936)].
109
Q
  1. A state has recently enacted a statute wherein aliens are forbidden from owning more than 10 acres of land within the state. Subsequent to the statute a resident alien enters into a contract to buy 50 acres of land located in the state.
    Assume that the statute empowers the state to bring an ejectment action against any alien who owns more than 10 acres of land. If the resident alien brings an action in federal court to enjoin the state from enforcing the statute against him, his best argument is
    (A) the statute violates the privileges or immunities clause of the Fourteenth Amendment.
    (B) the statute violates the contract clause.
    (C) the statute violates the commerce clause in that it interferes with land ownership.
    (D) the statute violates the equal protection clause of the Fourteenth Amendment.
A
  1. (D) The resident alien’s best argument is the fact that the statute violated his rights under the Equal Protection Clause of the 14th Amendment. Congress has plenary power over the admission of aliens, but once admitted, most state discrimination against them is “suspect” and can only be upheld if necessary to protect a “state’s special interest.” Although there are earlier decisions uphoLding state statutes limiting or barring aliens from owning land (within the state), it is highly unlikely that such a statute would be upheld in light of recent decisions. Since the resident alien is not a citizen, choice (A) is wrong because the Privileges and Immunities Clause is only applicable to citizens of the United States. Choice (B) is wrong because the contract did not take place prior to the enactment of the statute. Choice (C) is a weak argument because the limitation of land ownership is within the state and does not affect interstate commerce.
110
Q
  1. A state has recently enacted a statute wherein aliens are prohibited from owning any commercial real estate within the state. Subsequent to the statute, a lawful resident alien enters into a contract with another lawful resident alien to purchase the latter’s office building.
    Assume that both resident aliens join in a declaratory judgment action to test the validity of the state statute in federal court. The court should rule that
    (A) the state has the burden of proof to show that there is a compelling state interest to support the statute.
    (B) the burden of proof is on the resident aliens to show that there is no compelling state interest to support the statute.
    (C) the resident aliens do not have standing.
    (D) either resident alien has standing, but not both.
A
  1. (A) The federal government has power over aliens, but classifications by a state that are based on alienage are inherently suspect and subject to heightened judicial scrutiny. Since the statute makes a suspect classification, the burden of proving a compelling state interest rests with the state. Choice (B) is wrong because it incorrectly places the burden on the resident aliens. Choices (C) and (D) are incorrect because both resident aliens can establish standing based on the rationale [as expressed by the Supreme Court in Shelleyv. Kraemer, 334 U.S. 1 (1948)] that “the civil rights intended to be protected from discriminatory state action by the Fourteenth Amendment are the rights to acquire, enjoy, own and dispose of property.” Both have standing because the statute abrogates both the right to acquire and alienate property.
111
Q
  1. A state has had a tremendous influx of retired people in recent years. There has been considerable concern among state health officials who foresee that many of the senior citizens will become victims of price gouging on certain medical supplies and services. In an attempt to curb such fraudulent sales practices, the state legislature has enacted a law prohibiting the sale of hearing aids by non-physicians. The measure provides, however, that all non-physician sellers who are presently engaged in the business of selling hearing aids will not be affected.
    Assume that after the statute goes into effect, a non-physician moves to the state and wants to open a business selling hearing aids. After being advised that the state law prohibits him from doing so, he brings suit challenging the constitutionality of the statute. The most likely result is that the state law will be declared
    (A) constitutional, because there is a rational basis for distinguishing between non-physicians are not so engaged.
    (B) constitutional, because a state has the power to regulate any phase of local business, even though such regulations may have some effect on interstate commerce, provided that Congress has not enacted legislation regarding the subject matter.
    (C) unconstitutional, because it denies non- physicians who are not presently engaged in the business of selling hearing aids the equal protection of the law, in violation of the Fourteenth Amendment.
    (D) unconstitutional, because it violates the commerce clause, since Congress has plenary power to regulate any activity that has any appreciable effect on interstate commerce.
A
  1. (A) The U.S. Supreme Court has long recognized that almost all statutes and other forms of government regulation classify (or discriminate) people. As a result, the Court has established several different tests for determining their permissibility under the equal protection clause. The two major tests are the “traditional,” or rational basis test, and the “strict scrutiny,” or compelling interest test. For virtually all economic and social regulations, the Court employs the traditional equal protection test, usually defined as follows: “The classification (or discrimination) is valid if it is rationally related to a proper (or constitutionally permissible) state interest.” Under this rational basis test, a classification is presumed valid and will be upheld unless the person challenging it proves that it is “invidious” or “wholly arbitrary.” Thus, the challenger has the burden to prove that the regulation is not rationally related to a legitimate state interest. [McGowan v. Maryland, 366 U.S. 420 (1961)]. Conversely, government action that intentionally discriminates against racial or ethnic minorities is “suspect” and, thus, subject to “strict scrutiny.” Since this question deals with a matter of “economic and social welfare,” it will be reviewed under the basic rationality test. Therefore, choice (A) is the best answer. Choice (B) is incorrect because the commerce clause alone can limit state regulation. Choice (C) is incorrect. The equal protection clause provides heightened protection for groups whose legal classifications are suspect (e.g., classifications based on race, ethnicity, nationality, and religion) and for groups whose legal classifications are quasi suspect (e.g., gender, illegitimacy). Non-physicians may have been the subject of discrimination by the legislature, but they are not members of a suspect or quasi-suspect class. Only rational review will be applied, which the state probably will pass with ease. Finally, choice (D) is incorrect because the mere recitation that Congress has plenary powers to regulate interstate commerce is insufficient to invalidate the statute.
112
Q
  1. Over the last several years, the economy of a state has substantially changed. The state’s economy used to be based solely on heavy industry. However, the state legislature approved legalized gambling within the state. As a consequence, many casinos and new hotels were built and the state’s economy boomed. These moves were often induced by the granting by the state of special tax benefits for the construction of new casinos and hotels under state statutes.
    Recently, however, neighboring states have legalized gambling and offered greater tax incentives to the gaming industry. As a result, many of the casino and hotel owners have begun to leave the state. The unemployment and social welfare benefits the state has had to pay have substantially increased, burdening the remaining casinos, and also making it difficult for the state to lower its taxes to remain competitive with other states.
    On account of this predicament, the state legislature passed, and the governor duly signed, an emergency bill into law. According to the statute, the state imposed a one cent tax on the playing of any slot machine in any gambling casino. Since virtually all the slot machines required a payment of either a dime, quarter, or dollar, the imposition of this tax required a major costly adaptation on each slot machine to allow for the deposit of the additional one cent tax. Although many casino owners have complained about the tax, their only alternative is to absorb the tax themselves and lose one cent per game. As a consequence of the tax, fewer slot machines are purchased in the state by the casino owners. No manufacturer of slot machines is located in the state.
    Which of the following constitutional provisions provide the strongest ground to attack the validity of the state tax bill?
    (A) The commerce clause.
    (B) The equal protection clause of the Fourteenth Amendment.
    (C) The due process clause of the Fourteenth Amendment.
    (D) The privileges and immunities clause of Article IV, Section 2.
A
  1. (A) Pursuant to the Commerce Clause, Congress has complete power to authorize or forbid state taxation that affects interstate commerce. Undoubtedly, the state tax adversely affects interstate commerce because the facts indicate that all manufacturers of slot machines are out-of-state. As such, the Commerce Clause affords the strongest constitutional grounds to attack the state tax. Choice (B) is incorrect. The manufacturers may have been the subject of discrimination by the legislature, but they are not members of a suspect or quasi-suspect class. Only rational review will be applied, which the state probably will pass with ease. Choice (C) is incorrect. There is no fundamental right under the due process clause to purchase affordable slot machines. Also, property has not been taken without due process of law because the owners still legally possess all of their slot machines. Choice (D) is incorrect. The privileges and immunities clause of Article IV, Section 2 only protects a U.S. citizen from discrimination by a state if that state’s discrimination against out-of-staters is likely to discourage them from traveling to the state [Saenz v. Roe, 526 U.S. 489 (1999)]. Saenz involved a California law that would have made moving to California financially prohibitive for many people receiving welfare from other states and who would continue to need welfare in California. No such facts are present in our example.
113
Q
  1. A state recently imposed a one cent tax on the playing of any coin-operated video game in any restaurant. Since virtually all of the video games required a payment of either a dime, quarter, or dollar, the imposition of this tax required a major costly adaptation on each video game to allow for the deposit of the additional one cent tax. Although many video game owners have complained about the video game tax, their only alternative is to absorb the tax themselves and lose one cent per game. As a consequence of the tax, fewer video games are purchased by the restaurant owners. No manufacturer of video games is located in the state.
    Which of the following is most likely to have standing to bring suit challenging the constitutionality of the video game tax bill in an appropriate federal court?
    (A) A manufacturer of video games who is attempting to sell them to a restaurant owner in the state.
    (B) A state resident who frequently plays video games at restaurants.
    (C) A national video game association whose members travel to the state to play video games at restaurants.
    (D) The director of the state’s youth organization who wants to provide video games that only play for free at recreation centers within the state.
A
  1. (A) With respect to standing, a person asserting the violation of a constitutional or statutory right must show a direct and immediate personal injurydue to the challenged action. The facts clearly indicate that fewer slot machines are being purchased by casino owners on account of the tax. As a result, choice (A) is correct because a manufacturer of sLot machines can show a “direct injury” from application of the tax. Note that associations of individuals may have standing to assert the rights of its members, at least so long as the challenged infractions adversely affect its members’ associational ties. However, choice (C) is wrong because the associational members have not suffered a “direct” and “immediate” harm, since their tax liabiLity is optional (namely, they are not required to play the video games). Choice (B) is not the strongest answer because a state resident is not required to play the slot machine. Hence, it is hard for him to show a direct and immediate injury. Choice (D) is incorrect. The youth organization is not required to play the video games. Hence, it is hard for it to show a direct and immediate injury.
114
Q
  1. The board of a state university has adopted the following rule concerning residency requirements:
    “A student will be considered a legal resident of the state for the purpose of registering at the university if such person is over the age of 21 and has established a legal residence in the state for at least one year next preceding the last day of registration for credit.”
    A student moved to the state and immediately enrolled at the state university. Since he did not fulfill the university residency requirement, the student was required to pay $1,800 tuition each semester, which was $400 more than the tuition of state resident-students.
    In an action by the student challenging the constitutionality of the provision governing the determination of residency for the purpose of fixing a fee differential for out-of-state students in public college, the court will most likely declare the provision
    (A) unconstitutional, because it constitutes a violation of the equal protection clause of the Fourteenth Amendment.
    (B) unconstitutional, because it constitutes a violation of the privileges and immunities clause of Article IV, Section 2.
    (C) constitutional, because the fee differential promotes a compelling state interest.
    (D) constitutional, because the provision does not trigger strict scrutiny.
A
  1. (D) In Arizona Board of Regents v. Harper, 495 R2d 453 (1972), the Arizona Supreme Court held that a board of regents has the authority to adopt a rule requiring residence of one year before a student may be classified as a resident of the state to qualify for lesser charges. Thus, the one-year residency requirement, in order for a student to qualify for lesser tuition charges, did not violate the Due Process, Equal Protection, or Privileges and Immunities Clauses of the U.S. Constitution. In this regard, lower tuition rates at state universities are valid and do not trigger strict scrutiny [Starns v. Malkerson, 326 F. Supp. 234 (1971)]. Choice (A) is incorrect. The student may have been the subject of discrimination by the legislature, but he is not a member of a suspect or quasi-suspect class. Only rational review will be applied, which state university probably will pass with ease. Choice (B) is incorrect. The Supreme Court has never recognized a right under the privileges and immunities clause for in-state college tuition. Choice (C) is not the best answer. The reference to “compelling state interest” implies that strict scrutiny will be used, but that in turn implies that the student has a fundamental right under the Privileges and Immunities Clause to the same tuition as the in-staters. But, as elaborated in choice (B), he doesn’t. Accordingly, rational review will be used, which state university will easily pass.
115
Q
  1. During a violent electrical storm one night, a bolt of lightning struck a public high school building and set it ablaze. The high school was severely damaged and needed to be rebuilt. As a consequence, the city council held an emergency meeting to determine what measures should be taken to locate an appropriate alternative facility in which to conduct classes. Thereupon, the city council passed the following resolution: “During restoration of the high school building, classes shall be conducted at the most suitable facility which submits the lowest bid. In determining a ‘suitable’ facility, the city council shall consider such factors as location and available classroom space.”
    Several bids were submitted. The lowest bid was submitted by a church. The church was located on the same block as the high school and contained sufficient seating capacity for all students. In addition, there were a sufficient number of separate rooms to allow different classes to meet at the same time. The city council voted unanimously to accept the church’s offer. Furthermore, the church agreed to remove all religious symbols and paraphernalia from the classrooms utilized by the students. Only the main chapel was exempt, so that it could remain open for prayer. No high school classes or activities were to be held in the main chapel.
    A parent of one of the high school students is upset at this arrangement. On his son’s behalf, the parent has filed suit in federal district court to challenge the constitutionality of permitting public school classes to be held in a church. Judgment for whom?
    (A) The city, because the classroom arrangement does not inhibit or advance religion.
    (B) The city, because the church was the lowest bidder in accordance with the emergency ordinance.
    (C) The plaintiff, because the present arrangement for conducting classes in a church-owned facility constitutes excessive entanglement with religion.
    (D) The plaintiff, because the emergency measure was not necessary to further a compelling state interest.
A

115.(A) Underthe Lemon test, as a general rule, a government program will be valid under the establishment clause if it (1) has a secular purpose; (2) has a primary effect that neither advances nor inhibits religion; and (3) does not produce excessive government entanglement with religion. Many of the cases involving the establish ment clause involve religious activities in public schools (e.g., prayer and Bible reading). This question has an interesting twist: namely, whether public school classes can be held in a church building. Under the circumstances, there does not appear to be an establishment clause violation because the city’s action neither advances nor inhibits religion. Therefore, choice (A) is correct. Choice (B) is incorrect, as it ignores the religious establishment clause issue discussion, above. Choice (C) is incorrect. Excessive government entanglement with religion does not appear to be the case given that: (1) the church agreed to remove all religious symbols and paraphernalia from the “classrooms” utilized by the students; (2) only the main chapel was exempt so that it could remain open for prayer; and (3) no high school classes or activities were to be held in the main chapel. Finally, choice (D) is incorrect. The plaintiff will not win, and the city does not have to prove that there was a “compelling state interest.” Under the circumstances, there does not appear to be an establishment clause violation, because the city’s action neither advances nor inhibits religion.

116
Q
  1. An owner had a record store in the downtown business area of a city. A famous rock group was scheduled to perform at the local civic center and the owner featured the band’s records in a special sale for the two weeks prior to the concert. In order to promote his sale, the owner installed loudspeakers on the outside of his store window so that he could play the band’s records for people walking by to hear. It was the owner’s hope that when they heard the records, the passersby would turn into customers and buy the band’s records.
    Subsequently, the owner was cited for violating a city ordinance which provides that:
    “An owner of property located within the city limits shall not permit to be used on his property any device which causes sounds, other than clock chimes, to be heard upon the street or sidewalk. Violation of this ordinance shall subject the property owner to a fine of $50.00 for each occurrence.”
    If the owner is successful in challenging this ordinance in court, the court would most likely reason that
    (A) the ordinance violates equal protection because some sounds are permitted, while others are not.
    (B) the ordinance violates the owner’s rights of freedom of speech, because there is not valid interest to support the ordinance.
    (C) the ordinance violates the owner’s rights of freedom of speech, because a municipality may not regulate the use of sound amplification equipment.
    (D) the ordinance violates the owner’s rights under the First and Fourteenth Amendments, because it is vague in defining unpermitted sounds.
A
  1. (D) Choice (D) provides the best legal reasoning if the owner is successful in challenging the city’s sound amplification ordinance. In accordance with the 1st Amendment’s guarantee of freedom of speech, a state or municipality may regulate the use of sound amplification equipment, depending on the interests involved [Kovacs v. Cooper, 336 U.S. 77 (1949)]. The test applied by the courts in this 1st Amendment area (regarding the constitutionality of such an ordinance) is that “the government action must further an important governmental interest unrelated to the message being communicated.” According to the facts presented here, the municipality would not have an overriding interest in prohibiting the use of any sound device, except clock chimes, by a property owner in the city of Wilton. Choice (A) is incorrect, as the equal protection clause protects people, not sounds. Choice (B) is an incorrect statement of law. Choice (C) is incorrect because a state may regulate the use of sound amplification devices (e.g., sound trucks) in the interests of privacy and public tranquility.
117
Q
  1. A man outraged by the recent church decision to clear a famous philosopher of charges of heresy, decided to present a lecture, open to the public, disproving the philosopher’s theories. A state statute provides that: “state universities can permit the use of their lecture halls to the public for worthwhile programs of public benefit, upon approval of the school board.”
    The appropriate school board refused to make a university lecture hall available to the man on the grounds that the proposed lecture was not of worthwhile benefit to the public.
    As a result, the man brought suit in a state court against the school board and requested injunctive relief requiring the board to allow him the use of the lecture hall. The trial court denied relief and dismissed the suit. The judgment was affirmed by the state appellate court, and is now before the U.S. Supreme Court.
    In analyzing the state statute, which of the following statements is least accurate?
    (A) The statute is unconstitutionally overbroad, because it may result in the exclusion of protected speech as well as unprotected speech.
    (B) The statute, as applied to the man, does not violate his First Amendment rights because his proposed speech is not political and, therefore, not among the classes of speech that are protected.
    (C) Indirect speech, regulations are only permissible if necessary to serve compelling state interests.
    (D) The statute is a prior restraint on speech, which unconstitutionally vests unfettered discretion in the school board to decide who may use university lecture halls.
A
  1. (B) Choice (B) incorrectly suggests that only certain classes of speech are protected. Speech is GENERALLY protected; however, there are certain classes of speech that are unprotected (e.g., obscenity, fighting words, and slander). Choice (A) isan accurate statement because the statute is likely to be over-inclusive; it is vulnerable to a facial attack as unconstitutionally overbroad. Choices (C) and (D) are also accurate statements and, therefore, incorrect. Indirect speech regulations are permissible only if necessary to serve compelling state interests. Prior restraints on speech are not, per Se, unconstitutional, but where the power to decide which speech will be affected is vested in an individual or entitywho may decide, at their own discretion, and where there are no specific guidelines or oversight for such a decision, the discretion is considered unfettered, and the restraint on speech will not withstand constitutional muster.
118
Q
  1. For the past 20 years a city by the beach has been a popular location for surfboarding. City residents have recently complained that the surfers are creating a public nuisance by littering the beaches, harassing sunbathers, and injuring swimmers with their surfboards. As a consequence, the city adopted an ordinance prohibiting all surfing on its beaches. The newly enacted ordinance further prohibited the sale of surfboards within the city’s limits. An out of state surfboard manufacturer had planned to sell a new line of fiberglass surfboards in the city in the upcoming year. This is now precluded by the recently adopted measure.
    If the manufacturer seeks to enjoin application of the city ordinance, which of the following is the WEAKEST defense for the city?
    (A) There is no case or controversy.
    (B) The manufacturer’s case is moot.
    (C) The manufacturer lacks standing.
    (D) The case is not ripe.
A
  1. (B) As a general rule, a case is “moot” when there is no case or controversy once the matter has been resolved. In our example, the city’s WEAKEST defense is that the manufacturer’s case is moot because the matter (whether the manufacturer is precluded from selling his surfboards in the city) has not been resolved. Choice (A) is incorrect. Article III, Section 2 requires the existence of a “case or controversy” before a federal court may hear the case. In other words, there must be an actual dispute. This is not the weakest defense by the city, because the manufacturer has not tried to violate the law and, therefore, does not yet know whether he would be prosecuted under it. Furthermore, since the manufacturer had not meant to sell the boards until a year later, we do not know if the city would have changed its mind and repealed the ordinance. Choice (C) is incorrect, as the manufacturer would not appear to lack standing. One of the requirements for standing is the causation requirement. The manufacturer cannot be certain that he would actually be prosecuted under the law. Furthermore, since the manufacturer had not meant to sell the boards until a year later, we do not know if the city would have changed its mind and repealed the ordinance. Choice (D) is incorrect. The ripeness doctrine holds that “a claim is not ripe for adjudication if it rests upon contingent future events that may not occur as anticipated or, indeed, may not occur at all” [Texas v. United States, 523 U.S. 296 (1998)]. Here, since the manufacturer had not meant to sell the boards until a year later, we do not know if the city would have changed its mind and repealed the ordinance.
119
Q
  1. A group advocating the resumption of U.S. diplomatic relations with certain foreign countries planned to hold a rally at a park in the downtown section of a city. The group secured a rally permit in accordance with a local ordinance. Several members of the group, including a political science professor at a state university and one of the group’s leaders, were scheduled to give speeches. Other members of the group were assigned to walk among the crowd to solicit signatures for a petition, which the group planned to present to the President.
    A large crowd gathered in the park at the appointed date and time, anxiously waiting for the speeches to begin. As the professor, the first speaker, began addressing the gathering, a television news team started filming her presentation, which was to be shown on the local news that evening.
    After the professor finished her speech, a few members of the crowd began hissing and booing and shouting. The police soon arrived and attempted to break up the rally. Several members of the group, including the professor, were arrested for inciting a riot.
    Which of the following would be the most accurate statement with regard to the police halting the rally?
    (A) The police were justified, since the rally threatened imminent violence and serious disorder.
    (B) The police were justified in order to protect the group’s leaders.
    (C) The police violated the group’s First Amendment rights of assembly.
    (D) Since the group obtained the rally permit, the police were not permitted to interfere with the staging of the rally.
A
  1. (C) Under the circumstances, the police clearly violated the group’s 1st Amendment right of peaceful assembly. It is well established that assemblies or speeches that threaten imminent violence or serious disorder can be halted by the police to prevent physical injury, but unless the risk of disruption is clearly demonstrated, the gathering is protected. In the case at bar, the police would not be justified to break up the rally merely because of the jeering of a few members of the crowd. Moreover, if the risk of disruption is caused by a hostile crowd, the first duty of the police is to protect the speaker from the crowd, not to stop the speech and arrest the speaker. Therefore choice (C) is correct, and choice (A) is incorrect. Choice (B) is not the strongest answer because there is insufficient evidence to suggest that the leaders were in imminent danger. Choice (D) is incorrect. The mere possession of a permit does not confer absolute rights of free speech and assembly. If there is an imminent danger of violence or disorder, the police have a duty to protect the speakers and, if that is not possible, to halt the speech to avoid the imminent harm.
120
Q
  1. A professor employed by a state university is a well- known critic of foreign policy and has sometimes publicly stated that he wished for the United States to suffer some collective political tragedy in retribution for all the harms it does to the world. The professor was recently involved in a highly visible political protest against the government. The police eventually shut down the protest because they felt that it was becoming unruly. A television crew covered the entire rally.
    On the six o’clock news that evening, the film of the protest rally was shown during the broadcast. A news anchorman then made the following commentary: “It’s a shame that public funds are spent to pay the salaries of such university professors, who are not grateful for what this country has done for them. In my opinion, these people like the professor should be deported.”
    If the professor asserts a claim based on invasion of privacy against the television station and the anchorman for his television commentary, the most likely result is that the professor will
    (A) not prevail, because the criticism was not directed at the professor personally.
    (B) not prevail, because the broadcast was privileged as being in the public interest.
    (C) prevail, because the professor, as a private individual, was placed in a false light.
    (D) prevail, because the comments were made with actual malice.
A
  1. (B) In Time Inc. v. Hill, 385 U.S. 374 (1967), a case involving this particular invasion of privacy branch, the Supreme Court held that the 1st Amendment prohibited recovery for invasion of privacy in cases where the published matter was in the public interest, unless the plaintiff established that the defendant acted with malice. Malice here, as in New York Times v. Sullivan, goes to knowledge of falsity or reckless disregard for the truth. Moreover, choice (B) is the best answer because the 1st Amendment constitutional privileges likely encompass aLl pure opinions, whether false or not. As such, onlystatements of fact can be actionable as defamatory. In this example, the facts clearly state that the TV commentator, said, “In my opinion. . . these people like the professor should be deported.” Since the anchorman was merely voicing his opinion, choice (B) is a better answer than choice (D). Choice (A) is incorrect because it is irrelevant, whether or not the criticism was directed at the professor “personally.” What matters is whether the statement is a factual representation—a statement capable of being proved either true or false— because that is the only way to prove liability. Choice (C) is incorrect. A false light tort involves a defendant publishing factual information about the plaintiff; which tends to place the plaintiff in a false light; and which would be very offensive to a reasonable person were the latter in the plaintiff’s situation. In our example, there are no factual representations being made by the TV commentator, only opinions.
121
Q
  1. As a legislative aide to a U.S. Senator you are called upon to provide an analysis of the constitutionality of a bill pending congressional approval. The bill imposes a 15% tax upon the gross annual receipts from the sales of all birth control devices. The bill has the strong support of conservative and pro-life organizations. The stated purpose of the proposed measure is to raise revenue and spur population growth across the country.
    In your learned opinion, the proposed tax is probably
    (A) constitutional, because the fact that the tax applies to all sales of every type of birth control device invalidates any possible objection to the tax on the grounds that it violates the equal protection clause of the Fourteenth Amendment.
    (B) constitutional, because the fact that controversial policy motives may have induced the enactment of an otherwise reasonable measure calculated to raise revenue does not ipso facto invalidate the tax.
    (C) unconstitutional, because in inseverable aggregates, the domestic purchases and sales of birth control devices affect interstate and foreign commerce.
    (D) unconstitutional, because the tax burdens the fundamental right to privacy of users of birth control devices without establishing a compelling national interest for doing so.
A
  1. (B) Congress is granted broad powers of taxation by express constitutional provisions, namely, Article I, Section 8 (taxing and spending power) and the 16th Amendment (federal income tax without apportionment). The taxing power is virtually plenary. The standard used to analyze the validity of a federal tax is whether or not the dominant intent is fiscal. In other words, even if a federal tax does have some incidental regulatory effect, it will nevertheless be upheld if it does, in fact, raise revenue. The stated purpose of the proposed Senate bill for a 15% gross receipts tax is to raise revenue and spur population growth. Applying the aforementioned standard, the proposed federal tax will be constitutional. Choice (B) is correct, since it states the proper rationale. Choices (A), (C), and (D) are incorrect because validity of a federal tax is not analyzed under the principles of equal protection, the commerce clause, or the right to privacy.
122
Q
  1. A state university was the scene of campus protests against nuclear weapons. A group of students painted purple a statue of the university’s founder in protest to the university’s federally subsidized nuclear weapon experimental studies. The leader of this group of students was a first-year student from a neighboring state who established residency in the state after beginning classes.
    The group leader had been the recipient of a $5,000 annual grant from the state to finance his education. However, the aid was withdrawn because of a state statute, which provided: “Any student attending a state university who engages in disruptive campus activities will not be eligible for state aid.”
    The group leader was married to a fellow student. However, in light of his involvement in defacing the statue and the loss of his state aid, his wife left him. At that time, the group leader received a tax bill for $150. This tax was imposed uniformly by the city on all individuals over 19 years of age, with the exception that full-time female college students were exempted. The tax notice stated that his wife, 22 years of age, qualified for the exemption, and there was no bill enclosed for her.
    Subsequently, the group leader moved in with his new girlfriend and began making arrangements to secure a divorce. However, the group leader was not able to obtain a divorce, since he had not fulfilled the 12-month residency requirement as imposed by state law.
    In an action by the group leader against the state challenging the constitutionality of the state statute regarding disruptive campus activities in order to regain his $5,000 annual grant, the court will most likely declare the statute
    (A) constitutional, because it promotes a compelling state interest.
    (B) constitutional, because it is a proper exercise of state action designed to regulate the activities of state university students.
    (C) unconstitutional, because it is vague and overbroad.
    (D) unconstitutional, because it is discriminatory on its face.
A
  1. (C) The state statute providing that “any student.. .who engages in disruptive campus activities will not be eligible for state aid” would be declared unconstitutional for vagueness and overbreadth. Clearly, the court will invalidate such a statute because “disruptive campus activities” is too general and overbroad and can be read as prohibiting constitutionally protected activity. Therefore, for these reasons, choice (C) is correct, and choice (A) is incorrect. Choice (B) is incorrect. Idiomatically, there is no proper exercise of “state action.” The state action doctrine refers to the idea that the government has acted and is thus, theoretically, subject to the U.S. Constitution. Choice (D) is incorrect. The problem with the statute is not that it discriminates on the basis of viewpoint or content. The problem lies in that the statute suffers from substantial vagueness and overbreadth in its reference to “disruptive campus activities.”
123
Q
  1. In order to encourage college enrollment, a state provides a tax exemption of $200 for state taxes for those who are full-time students at colleges within the state. Female students who attend half-time are still permitted a $100 tax exemption, whereas male students who attend half-time are no longer permitted any tax exemption.
    Which of the following most accurately summarizes the correct rule of constitutional law regarding the state tax exemption for students who enroll half-time?
    (A) The tax exemption would be invalidated as a denial of due process.
    (B) The tax exemption would be invalidated as violative of the equal protection clause.
    (C) The tax measure would be upheld as within the area of substantive due process.
    (D) The tax measure would be upheld as within the power of a municipality to tax different classes of persons unequally.
A
  1. (B) Such laws explicitly distinguishing between males and females have been invalidated as violative of the Equal Protection Clause of the 14th Amendment unless they serve the objective of offsetting unequal opportunities for women (or men, as the case may be). In the present case, there is no important governmental interest available to support a tax exemption for only full-time female college students and not male students. Choice (A) is incorrect. Generally, a property interest is not protected under the 14th Amendment’s Due Process Clause unless there is a reasonable expectation of continued receipt of the benefit. [Board of Regents v. Roth, 408 U.S. 564 (1972)]. In our example, no such reasonable expectation of continued receipt of the benefit exists for the college men because the law had already informed them that failure to enroll fulltime will erase the tax exemption. Choice (C) is incorrect. There is no substantive fundamental right protected by the due process clause that has been denied by the tax exemption. Most substantive rights protected by the due process clause concern the rights of bodily and familial autonomy. Choice (D) is incorrect. States may have the power to tax different people in different ways, but it may not do so in a way that violates the Constitution. Here, the tax exemption—having a gender classification—will be subject to intermediate review and will likely fail such review.
124
Q
  1. A couple were married, and four months later, they wished to obtain a divorce. However, the state in which they were married prohibits a couple from obtaining a divorce until they have resided in the state for at least 12 months.
    In an action by the couple challenging the constitutionality of the residency requirement of the state divorce law, the court will most likely declare the provision
    (A) constitutional, because the requirement promotes a compelling state interest.
    (B) constitutional, because it is within the area of state action.
    (C) unconstitutional, because it constitutes a violation of the equal protection clause.
    (D) unconstitutional, because it constitutes a violation of the privileges and immunities clause of Article IV, Section 2.
A
  1. (A) Limited residency requirements (of one year or less) for obtaining divorces have been upheld as promoting a compelling state interest in the exercise of the state’s police powers to legislate to protect the health, safety, welfare, and morals of its citizens. Choice (B) is incorrect, since the concept of state action pertains to nullifying state legislation that impairs the privileges and immunities of citizens of the United States, orwhich injures them in life, liberty, or property without due process of law, orwhich denies to any of them the equal protection of the laws. Choice (C) is incorrect. While they may have been the subject of discrimination by the legislature, they are not members of a suspect or quasi-suspect class. Only rational review will be applied, which the state probably will pass with ease. Choice (D) is incorrect. In order to show a violation of the privileges and immunities clause, there must be an initial showing of discrimination by the state against out-of-staters. In our example, there is no such discrimination.
125
Q
  1. A privately owned shopping center leases retail store space to private retailers. A group of students from a local high school were distributing pamphlets commemorating a national holiday in the enclosed mall area of a privately owned shopping complex. The management of the shopping complex requested that the students cease distributing the pamphlets or leave the premises. When they refused, the police were summoned to disperse the students. Upon the arrival of the police, the students were removed from the premises.
    Subsequently, the students brought suit in federal court seeking an injunction that would order the shopping complex management to allow them to distribute the pamphlets within the mall. The students will
    (A) prevail, because pamphleteering is a speech- related activity, which is protected by the First and Fourteenth Amendments.
    (B) prevail, because there is not an anti-pamphleteering statute.
    (C) not prevail, because pamphleteering on private property is not a constitutionally protected activity.
    (D) not prevail, because pamphleteering may be prohibited as a public nuisance that invades the privacy interest of persons not wishing such communicative contact.
A
  1. (C) In accordance with Hudgens v. NLRB, 424 U.S. 507 (1971), the owner of a private shopping center may exclude persons who want to distribute pamphlets, since no “state action” is present. The Supreme Court held that, so long as the state does not aid, command, or encourage the suppression of free speech, the 1st Amendment would not be violated by the shopping center owners. The operation of the shopping complex was not part of a privately owned town and, therefore, did not involve the assumption of a public function by private persons. Since pamphleteering on private property is not a constitutionally protected activity, injunctive relief would not be granted to the students. Choices (A) and (B) are incorrect because there is no state action in this example; thus, the Constitution does not apply. Choice (D) is incorrect. The nuisance action is unnecessary, and it is superfluous to discuss the privacy interests of persons not wishing to be accosted by pamphleteers.
126
Q
  1. A state legislature has recently enacted a statute making it a misdemeanor to curse or revile or use obscene or opprobrious language toward or in reference to a police officer perfonning his duties. A student at a state university organized a demonstration on campus to protest the war. The rally was attended by a group of 50 students who shouted anti-war messages at cars passing by.
    To show his contempt for the United States, the student sewed the American flag to the rear of his jeans. When a police officer saw the flag sown on the student’s jeans, he approached and told him to remove the flag or he would be placed under arrest. The student became angered and shouted at the police officer, “Listen, you bastard, I’ll wear this rag anywhere I please.” The student was subsequently placed under arrest and charged with violating the state statute.
    The student subsequently brings suit in state court challenging the constitutionality of the statute. The strongest constitutional argument for the student is that
    (A) the statute is void for vagueness under the Fourteenth Amendment’s due process clause.
    (B) the statute is invalid because it violates the petitioner’s freedom of speech under the First Amendment.
    (C) the statute is an abridgment of freedom of speech under the First Amendment because less restrictive means are available for achieving the same purpose.
    (D) the statute is overbroad and consequently invalid under the First and Fourteenth
    Amendments.
A
  1. (D) The state statute wouLd be construed as overbroad and, thus, invalid under the 1st and 14th Amendments. The Supreme Court, in Lewis v. City of New Orleans, 415 U.S. 130 (1974), applied the overbreadth doctrine to a similar New Orleans ordinance, which was invalidated under the 1st and 14th Amendments. In that case, as well as in the factual situation here, the ordinance effectively punished all obscene and offensive speech, even though some of the speech may have been protected by the 1st Amendment. Since Section 1220 may have included constitutionally protected speech, the statute should have been more narrowly drawn to protect such 1st Amendment activities. Choice (A) is not the best answer. If a statute is unduly vague, it will be voided on its face. The rationale is that the audience will suffer a chilling effect by being afraid to speak at all. Here, the statute is vague in at least two respects: in the reference to “opprobrious language” and in the reference to “treat contemptuously.” Both references, however, are more objectionable because they suffer from overbreadth by prohibiting speech that is protected by the 1st Amendment. Choice (B) is not the strongest choice. This answer presumes that the statute is invalid only because it violates the 1st Amendment. The statute is actually invalid on its face, without having to determine whether it violates the speaker’s rights in this instance. Choice (C) is incorrect. This answer presumes that the statute, on its face, is valid and that the constitutional problem resides in its application to the instant facts. But the statute is void on its face.
127
Q
  1. A state legislature has recently enacted the following statute:
    “Statute 1221. It shall be unlawful for an individual to publicly mutilate, trample upon, deface, or treat contemptuously the flag of the United States. Whoever shall violate this statute shall be guilty of a misdemeanor.”
    To show his contempt for the United States, an antigovernment protestor sewed the American flag on the rear of his jeans. He was subsequently placed under arrest and charged with violating Statute 1221.
    If the protestor is subsequently prosecuted under Statute 1221 for his flag misuse, he
    (A) should be convicted.
    (B) should not be convicted, because the lack of ascertainable standards for defining “treat contemptuously” violates the equal protection clause of the Fourteenth Amendment.
    (C) should not be convicted, since the statutory language is void for vagueness under the First and Fourteenth Amendments.
    (D) should not be convicted, because the statute has a chilling effect on nonverbal forms of speech and, therefore, is invalid under the First and Fourteenth Amendments.
A

127.(C)or(D) In accordance with Smith v. Gognen, 415 U.S. 566 (1974), where the petitioner had been convicted of violating a similar flag-misuse statute for sewing a U.S. flag to the seat of his pants, the Supreme Court held that the statutory language was void for vagueness under the 1st and 14th Amendments. The flag-misuse statute was declared vague because no clear distinction had been made between what type of treatment of the flag was or was not criminal. Furthermore, the statutory terminology, “treat the flag contemptuously,” is lacking of any ascertainable standards and is, therefore, violative of the Due Process Clause of the 14th Amendment. Students should note that the police and courts should not be given such broad discretion in determining what constitutes flag contempt as to be violative of the 1st and 14th Amendments’ constitutional safeguards. Choice (B) is incorrect because this question presents a due process violation, not an equal protection issue. Choice (A) is incorrect because the statute here suffers from undue vagueness and, thus, will be voided on its face.

128
Q
  1. A state legislature recently enacted a statute legalizing harness racing. The statute authorized pari-mutuel betting at certain track locations within the state. A seven-member commission was established and empowered to supervise and regulate the sport’s activities. Under an inseparable provision of the statute, the commission was authorized to suspend the racing license of any trainer whose horse tested positive for illegal drugs. The statute permitted the commission to make the suspension without any prior hearing. However, suspended trainers were entitled to a prompt post- suspension hearing and decision on any issues in dispute.
    The racing season was inaugurated at the largest racetrack in the state. The featured race was a $1,000,000 harness race for 2-year-old trotters. After the awards presentation, the winning horse underwent a standard drug test and traces of cocaine were found in his urine sample. Immediately thereafter, the horse was disqualified and the commission suspended the horse’s trainer, without a prior hearing.
    Without seeking a post-suspension hearing as provided by statute, the trainer brings suit in federal district court challenging the constitutionality of the state harness racing law. The statute is probably
    (A) constitutional, because being granted a racing license is a privilege, not a right.
    (B) constitutional, because the state’s interest in suspending the license of horse trainers suspected of illegal drugging is sufficiently important to permit the suspension of any prior hearing.
    (C) unconstitutional, because the suspension provision unreasonably interferes with a trainer’s right to contract with horse owners and seek gainful employment.
    (D) unconstitutional, because the suspension provision violates due process by not affording a prior hearing.
A
  1. (B) The 14th Amendment’s procedural due process clause operates as a Limitation on state action by providing an individual the guarantees of both notice of the charges brought against him, as well as an opportunity to be heard, whenever the deprivation of any life, liberty, or property interest has occurred. The individual whose interests are affected must be granted a fair procedure to determine the factual basis and legality for such action. If the government terminates an individual’s ability to engage in a profession, a procedure must be afforded to determine the individual’s fitness to engage in that profession. Specifically, in Barry v. Barchi, 443 U.S. 555 (1979), the court held that the New York licensing system for horse training created a “property” interest in licensed trainers protected by the Due Process Clause and a post-suspension hearing was required. In this question, the state statute affording a post-suspension hearing will be upheld. Therefore, choice (B) is correct. Choice (A) is incorrect. The rights-privilege distinction has been abandoned by the Supreme Court. Instead, the Court looks to whether there is a reasonable expectation of the continuance of the government benefit. Here, the trainer will be able to show such expectation. Choice (C) is incorrect because there is no fundamental right to contract; nor is there a fundamental right to “seek gainful employment.” Finally, choice (D) is incorrect. Under a licensing scheme, a right to a hearing prior to revocation of the License is not required, although a right to a hearing is required after the revocation of the license.
129
Q
  1. Proposed legislation was offered to a state legislature that would reorganize the state police. The bill created a great deal of controversy, both in and outside the state government. Several leaders of the minority party in the legislature decided to oppose the legislation. One member of the minority party disagreed with his party’s opposition to the bill and publicly announced his support for the legislation.
    The minority party leaders called a caucus to discuss and determine their legislative strategy for floor debate on the bill. When the disagreeing member appeared at the door of the caucus room, he was denied admission because of his anti-party stance. He was also informed that he would be removed from all of his committee assignments.
    During the caucus, the party members discussed other means of disciplining the member for his party insubordination. It was suggested that they issue a press release in which the party would publicly castigate him for his actions. The leader of the party said that “the member is a cutthroat politician who is only looking out for where his next buck will come from.”
    Which of the following constitutional provisions would give the ousted member his best grounds for challenging his exclusion from the party caucus?
    (A) The equal protection clause of the Fourteenth Amendment.
    (B) The right of assembly as guaranteed by the First Amendment.
    (C) The speech and debate clause.
    (D) The due process clause of the Fourteenth Amendment.
A
  1. (D) The due process clause of the 14th Amendment would furnish the plaintiffs best grounds for challenging his exclusion from his party’s caucus. Whenever a governmental instrumentality acts so as to deprive someone of any interest, the first question to ask is whether the interest qualifies as “life,”“liberty,” or “property.” If so, the due process safeguards of notice and some form of hearing are required. In the present hypo, the plaintiffs exclusion from the caucus would be a deprivation of his property interest (i.e., stripping a duly elected public official of his right to participate in his party’s meeting). Choice (A) is incorrect, while the plaintiff may have been the subject of discrimination by the legislature, he is not a member of a suspect or quasi-suspect class, and only rational basis review will be applied. Choice (B) is not the strongest answer. The right to assemble would not be the best argument, since the party members do not wish to associate with the plaintiff. To force them to do otherwise probably would violate their right to assemble. Choice (C) is incorrect. There is no violation of the right of speech if there is a valid time, place, and manner regulation for the speech. Such a law regulates the secondary effects of the speech (not the speech) and provides reasonabLe alternatives for the speech. Here, one can argue that the minority party is not punishing Turner, per se.
130
Q
  1. During a Senate debate, members discussed how to punish a particular senator for having publicly rebuked the head of his party. One of the members suggested that the party inform television reporters that the senator in question is “an opportunist who has very little loyalty to his own party and will switch sides at the earliest convenience.”
    In determining whether the senator has a valid cause of action against the member for his remarks, which of the following most accurately reflects the applicable rule of law?
    (A) The senator must prove actual malice in order to recover for defamation.
    (B) Any remarks made during the debate were privileged.
    (C) The remarks violated the senator’s First Amendment right of privacy by placing him in a “false light.”
    (D) The remarks constitute a “fair and substantial” relation to “important governmental objectives.”
A
  1. (B) “The Senators and Representatives.., for any Speech or Debate in either House… shall not be questioned in any other Place” [U.S. Const. art. I, sec. 6, cI. 1]. It is important to note that virtually every state has adopted similar speech and debate clauses in their respective state constitutions. Choice (A) is incorrect. The senator did not offer any factual representations. The actual malice test requires that the speaker offer some factual representation (later proved inaccurate) either deliberately knowing of its inaccuracy or in reckless disregard. Choice (C) is incorrect. A false light tort involves a defendant publishing factual information about the plaintiff, which tends to place plaintiff in a false light, and which would be offensive to a reasonable person were the latter in plaintiff’s situation. In this question, there are no factual representations being made, only opinions. Choice (D) is incorrect because, even if this were factually true, it does not matter for legal purposes. Therefore, choice (B) is correct, because the statement would be privileged, since the Speech and Debate Clause protects Legislators and their aides against criminal or civil proceedings for “legislative acts.”
131
Q
  1. Congress, under intense lobbying pressure has enacted a statute prohibiting the sale of contraceptive devices to married persons. The act further prohibits the use of contraceptive devices by married persons. Congress claimed that the statute was passed because it might help deter illicit sexual relationships.
    The law is most likely
    (A) constitutional, because it is a regulation of interstate commerce.
    (B) constitutional, because it is a measure promoting the general welfare.
    (C) unconstitutional, because the law deprives the manufacturers of contraceptives of their property interest without just compensation.
    (D) unconstitutional, because it violates the right of privacy of contraceptive users.
A
  1. (D) In Griswold v. Connecticut, 381 U.S. 479 (1965), the U.S. Supreme Court invalidated a similar statute restricting the use of contraceptive devices by married couples as violating the right of privacy of married persons. The Court held that the statute violated the due process clause because it deprived these married persons the liberty protected by the 5th Amendment. Choice (A) is incorrect. Even if Congress is empowered under the Commerce Clause to enact this law, Congress would still run afoul of the fundamental right to contraceptives in the 5th Amendment’s due process clause. Choice (B) is incorrect. Article I, Section 8, reads: “The Congress shall have Power to lay and collect Taxes, Duties, Imposts and Excises, to pay the Debts and provide for the common defence and general welfare of the United States.” The Supreme Court has taken the reference to “general welfare” to mean that Congress enjoys great discretion in how it chooses to allocate money for the public [United States v. Butler, 297 U.S. 1 (1936)]. There is no such allocation of money in our example; just a regulation. Choice (C) is incorrect. In Lucas v. South Carolina Coastal Council, 505 U.S. 1003 (1992), the Court stated that a taking occurs where “regulation denies all economically beneficial or productive use of the land.” Here, the congressional law does not deprive the manufacturers all property interest in contraceptives, because the manufacturers may still seLl the contraceptives to people who are not married.
132
Q
132. A state legislature enacts a statute prohibiting the sale of contraceptive devices to married persons. This state statute prohibits the use of contraceptive devices by married persons. A physician who practices in the state brings suit in federal court challenging the constitutionality of the state contraceptive statute. The physician attacks the validity of the statute on the grounds that it prevents him from giving professional advice concerning the use of contraceptives to three patients, all of whom are married, whose condition of health might be endangered by child bearing.
The plaintiff is likely
(A) to have standing.
(B) to have standing jus tertii.
(C) not to have standing.
(D) not to have standing jus tertii.
A
  1. (D) In Tileston v. UlIman, 318 U.S. 44 (1943), the U.S. Supreme Court ruled that a medical doctor does not have third-party standing to attack a state anti-contraceptive statute on the grounds that it prevents him from giving his professional advice concerning the use of contraceptives to three patients whose condition of health might be endangered by child bearing. On the contrary, if the person is convicted of prescribing, selling, or giving away contraceptives, in the defense of that action, he may then raise the third-party rights of the recipients. [Eisenstadtv. Baird, 405 U.S. 438 (1972)]. Under this reasoning, choice (D) is correct and, therefore, choices (A) and (B) are incorrect. Choice (C) is incorrect. The physician would seem to have standing. First, if he tried to violate the state law, he would be convicted and, thus, would suffer iarm. Second, there’s causation, because the state is directly responsible for the potential prosecution and the physician is able to show that the state can remedy its harm by repealing its law.
133
Q
  1. The owner of a test prep company sent an advertisement regarding the company’s review courses to a local newspaper. In an accompanying letter, the owner instructed the newspaper to publish the ad in an upcoming edition. The ad was received by the newspaper’s advertising editor.
    The next day the editor phoned the owner and told her that he would not permit the newspaper to run the ad. When the owner asked for an explanation, the editor replied, “My daughter took your review course and scored low on her entrance exams to college and didn’t get into the college of her choice. Because of your instructors, she’s now working at the mall. That’s why I’m prohibiting the newspaper from publishing your ad.” The editor then forwarded a letter to the owner reiterating his newspaper’s refusal to have the ad published.
    In an appropriate action, the owner brings suit against the editor and the newspaper seeking an order that would require the newspaper to publish the advertisement. In such action, who will prevail?
    (A) The owner, because such advertising is protected by the First Amendment under the commercial speech doctrine.
    (B) The owner, because there is a constitutional right of advertising under the First and Fourteenth Amendments.
    (C) The editor and newspaper, because Congress is empowered to prohibit untruthful advertising, even where it urges the purchase of a legal, validly offered item.
    (D) The editor and newspaper, because there is no constitutional right of advertising under the First and Fourteenth Amendments.
A
  1. (D) There is no constitutional right of advertising under the 1st and 14th Amendments. Certainly, a private newspaper is not required under the Constitution to accept and publish all forms of advertising. Choice (A) is incorrect. There is a lack of state action because the newspaper is a private entity. Accordingly, the newspaper cannot violate a person’s constitutional rights, although other Legal remedies might be available. Choice (B) is not the best answer. There is a constitutional right to commercial speech, meaning that the government may not infringe on such a right. However, there is no constitutional right to “advertise” in a private newspaper. Choice (C) is incorrect. Congress’s powers are not legally relevant for this example. There is no constitutional right of advertising under the 1st and 14th Amendments. Certainly, a private newspaper is not required under the Constitution to accept and publish all forms of advertising.
134
Q
  1. A prisoner was serving a life sentence in a state prison as a result of his conviction for the murder of a child who had trespassed onto his farmland. The prisoner came from a family of farmers, dating back to at least 1750. His family believed that all nourishment comes from the ground and that one’s soul will be saved only if his diet consists totally of natural, farm-grown food. The prisoner followed that belief and ate only fresh fruits and vegetables. He further believes that a higher power has commanded him to eat only vegetarian foods.
    When the prisoner entered the prison state prison officials agreed to grant his wishes and served him only fresh fruits and vegetables for his meals. After six months, deciding that catering to his special diet was overly burdensome and administratively unworkable, the officials decided to stop giving the prisoner special treatment and began to serve him the same food as served to the rest of the prison population. Although nothing physically prohibited the prisoner from eating and surviving on the general prison population’s diet, he refused to eat the food that was not in conformity with his special diet.
    The prisoner’s best constitutional argument to support his claim of the right to a fresh fruit and vegetable diet is based on
    (A) the First Amendment.
    (B) the Eighth Amendment’s prohibition against cruel and unusual punishment, as applied to the states.
    (C) the Fourteenth Amendment’s substantive due process clause.
    (D) the Fourteenth Amendment.
A
  1. (D) The prisoner’s best constitutional argument will be based on the 1st Amendment’s free exercise clause, which is applicable to the states through the 14th Amendment. Even though it is not known whether his belief is properly classified as a religion for 1st Amendment purposes, choice (D) still provides the prisoner’s best argument. Although choice (A) seems correct, choice (D) is the better answer, since state action is involved. When there are apparently two correct answers to a question, you should select the alternative more on-point to the facts in the question. As illustrated in this question, in the area of Constitutional Law, when a particular question relates to one’s rights under any of the first eight Amendments to the U.S. Constitution, such rights are only afforded through the due process clause of the 14th Amendment. Therefore, in our hypo, choice (D) is the preferred answer, as it refers to both the 1st and 14th Amendments. Choice (B) is incorrect. The 8th Amendment guarantees that inmates shall be served adequate food, not food to their tastes. Choice (C) is incorrect. There is no fundamental right to religious expression or to vegetarianism under the 14th Amendment’s Substantive Due Process Clause.
135
Q
  1. Several states have enacted laws shielding reporters from being compelled to release the names of confidential sources. Now, Congress, under intense lobbying pressure from the press, proposes to enact legislation forbidding any state from requiring journalists to reveal the sources of their news articles in civil suits.
    Which of the following is the strongest constitutional argument in support of this proposed law?
    (A) Congress has the authority under the commerce clause to regulate the flow of news.
    (B) Acts of Congress are the supreme law of the land and take precedence over any conflicting state laws.
    (C) Congress is essentially reaffirming the free speech guarantees of the First and Fourteenth Amendments.
    (D) Under Article I, Section 8, Congress has the authority to secure to authors and inventors the exclusive right to their respective writings and discoveries.
A
  1. (A) Under the “affectation doctrine,” the U.S. Supreme Court has recognized that Congress has the power to regulate any activity, whether carried on in one state or many, which has anyappreciable effect—dire ctlyorindirectl y—upon interstate commerce. Choice (B) is incorrect. The Supremacy Clause cannot serve as a basis for a congressional statute, although once passed, the congressional statute, under the logic of the supremacy clause, can take precedence over a state statute. Choice (C) is incorrect because Congress has no legislative power under the 1st Amendment. Congress has legislative powers under Section 5 of the 14th Amendment but, in our example, it is not clear how the congressional statute would further some 14th Amendment right. Choice (D) is not the best answer. Article I, Section 8 states that Congress has the authority “To promote the Progress of Science and useful Arts, by securing for limited Times to Authors and Inventors the exclusive Right to their respective Writings and Discoveries.” This clause refers to Congress’s authority to bestow patents and copyright, not freedom of the press.
136
Q
  1. A motel advertises the showing of pornographic, or adult movies in the privacy of each room. The motel has a strict policy permitting adults only to occupy the rooms. The state has recently enacted a statute that prohibits the showing of any obscene film in an area open to the public.
    The owner of the motel is prosecuted for violating the statute by showing pornographic movies in the motel rooms. On appeal, the owner’s conviction will probably be
    (A) sustained, because a state can use its police power to prohibit the showing of pornography in public areas.
    (B) sustained, because a state may use local standards in determining whether a movie has redeeming literary, artistic, political, or scientific merit.
    (C) overturned, because his prosecution violates the right of consenting adults
    to view such films in private.
    (D) overturned, because the First and Fourteenth Amendments prohibit the suppression of sexually oriented materials on the basis of their allegedly obscene contents.
A
  1. (C) The Multistate examination tests not only your knowledge of the substantive rules of law; it also tests your reading comprehension ability. The state statute prohibits the showing of any obscene film “in an area open to the public.” Since the films were being shown in the privacy of the motel rooms (and occupancy was limited to consenting adults), the owner’s conviction would be overturned because the statute would be inapplicable. ALso, for Multistate purposes, students should be familiar with Stanley v. Georgia, 394 U.S. 557 (1969), in which the U.S. Supreme Court held that mere private possession of obscene matter is not a crime. Although the states retain broad power to regulate obscenity, that power simply does not extend to mere possession by the individual in the privacy of his home. Choice (A) is incorrect for the reasons stated above. Choice (B) is incorrect. The reference to literary, artistic, and the like refers to the Miller obscenity test. Obscenity as a legal category receives no constitutional protection. However, in this example, it’s not certain that we’re dealing with obscenity, but with pornography. Obscenity is defined as material that is (1) patently offensive; (2) appeals to prurient interests; and (3) the work, taken as a whole, lacks any serious literary, artistic, political, or scientific value [Miller v. California, 413 U.S. 15 (1973)]. Even if the material in the hotel is deemed obscene, students should be familiar with Stanley v. Georgia, 394 U.S. 557 (1969), in which the U.S. Supreme Court held that mere private possession of obscene matter is not a crime. Although the states retain broad power to regulate obscenity, that power simply does not extend to mere possession by the individual in the privacy of his home. Choice (D) is incorrect. As a general matter, materials deemed obscene do not enjoy hardly any constitutional protection. However, students should be familiar with Stanleyv. Georgia, 394 U.S. 557 (1969), in which the U.S. Supreme Court held that mere private possession of obscene matter is not a crime. Although the states retain broad power to regulate obscenity, that power simply does not extend to mere possession by the individual in the privacy of his home. Moreover, in this example, it’s not certain that we’re dealing with obscenity, but with pornography. Obscenity is defined as materiaL that is (1) patently offensive; (2) appeaLs to prurient interests; and (3) the work, taken as a whole, lacks any serious literary, artistic, political, or scientific value [Millerv. California, 413 U.S. 15(1973)].
137
Q
  1. Each year the state provides a number of non- interest-bearing loans andlor scholarships to candidates for the degree of L.L.B or J.D. at the state’s law school. The applicable state statute limits eligibility to citizens of the United States and aliens seeking U.S. citizenship. A candidate for a J.D. degree at the state law school applied for one of the non-interest-bearing loans. Since he did not intend to seek U.S. citizenship, he was refused the loan for ineligibility under the state statute.
    In a suit by the candidate challenging the constitutionality of the state statute, he will
    (A) win, because the statute is violative of the privileges or immunities clause of the Fourteenth Amendment.
    (B) win, because classifications by a state that are based on alienage are inherently suspect and subject to close judicial scrutiny.
    (C) lose, because the statute promotes a compelling state interest.
    (D) lose, because alienage classifications are not, per Se, unconstitutional under the Fourteenth Amendment’s equal protection clause.
A
  1. (B) In accordance with Nyquist v. Mauclet, 432 U.S. 1 (1977), the U.S. Supreme Court invalidated, under the equal protection clause of the 14th Amendment, a state law that granted aid for higher education to citizens and resident aliens who were or would be applying for citizenship. The Court found no “compelling state interest” in encouraging citizenship or limiting general programs to those who determine its policy. Therefore, choice (B) is correct, and choice (C) would be incorrect under our facts. Choice (A) is incorrect because the privileges or immunities clause of the 14th Amendment does not protect those who are non-U.S. Citizens. Choice (D) is incorrect. Technically, it is true that alienage classifications are not, per se, unconstitutional under the 14th Amendment’s equal protection cLause. However, such cLassifications will still be subject to strict scrutiny, and the state must show that its law contains compelling governmental interests [Graham v. Richardson, 403 U.S. 365 (1971)].
138
Q
  1. A state has a statute generally prohibiting the installation and use of radar detection devices in any motor vehicle operating on the roadways within the state. This prohibition was enacted to prevent motorists from evading radar or speed checkpoints on county and state highways. A neighboring state has no such regulation in effect. By the same token, Congress has taken no action on the matter.
    A resident of the neighboring state has installed a radar detection device in his automobile. While driving to visit his mother he is arrested and charged with violating the aforementioned state statute. The resident files a complaint challenging the constitutionality of the state statute.
    As applied to the resident, the state prohibition against the use of radar detection devices is likely to be held
    (A) constitutional, because it protects a legitimate state interest.
    (B) constitutional, because the commerce clause does not invalidate a state regulation relating to interstate commerce unless Congress takes express action to do so.
    (C) unconstitutional, because the state statute fails to give credit to the law of a neighboring state.
    (D) unconstitutional, because it unduly burdens interstate commerce.
A
  1. (A) As a generaL rule, state laws regulating roadways and highways are usually upheld as constitutional unless they unduly burden interstate commerce. Since the facts do not indicate that the state statute unduly burdens interstate commerce, choice (A) is preferred over choice (D). Anothervariation of how this ruLe can be tested involves the enactment of a state law prohibitingthe use of metalstuds or cleats on vehiculartires. Even though the facts may indicate that the cleats and studs give better traction in ice and snow, the statute may stilt be upheld as constitutional if it serves a legitimate state interest by reducing damage to state highways. Choice (B) is incorrect. The commerce clause of Article I, Section 8 can invalidate state regulation if such regulation presents an undue burden on interstate commerce. But that does not appear to be the case in this example, given that both in-staters and out-of-staters must comply with the same prohibition. Choice (C) is incorrect. Article IV, Section 1 contains the Full Faith and Credit Clause: “Full Faith and Credit shall be given in each State to the public Acts, Records, and judicial Proceedings of every other State. And the Congress may by general Laws prescribe the Manner in which such Acts, Records and Proceedings shall be proved, and the Effect thereof.” But the law in our example would not be covered under the Full Faith and Credit Clause.
139
Q
  1. Which of the following constitutional provisions is applicable to corporations?
    (A) The privileges and immunities clause of the Fourteenth Amendment.
    (B) The comity clause of Article IV.
    (C) The Fifth Amendment’s prohibition against compulsory self-incrimination.
    (D) The equal protection clause of the Fourteenth Amendment.
A
  1. (D) The equal protection clause of the 14th Amendment is applicable to corporations. Choices (A) and (B) are incorrect because neither the privileges and immunities clause nor the comity clause applies to corporations, because the term “citizen” does not include corporations. Choice (C) is incorrect because, under the 5th Amendment (the prohibition of compulsory self-incrimination), the word “person” fails to include corporations or other business entities [Bellis v. United States, 409 U.S. 322 (1973)].
140
Q
  1. A state has recently enacted a statute that provides no person or company may be awarded any state construction contract unless the person or company agrees to hire only citizens of the state. The primary purpose of the statute is to help alleviate the state’s high rate of unemployment.
    Which of the following, if established, is the strongest argument in support of the statute if it is attacked as violating the commerce clause?
    (A) The statute will help racial minorities living in the state obtain gainful employment.
    (B) The state has the highest unemployment rate in the country.
    (C) If the state uses its own taxpayer funds to purchase construction materials, it is responsible for creating demand for the required labor.
    (D) The statute was overwhelmingly adopted by the voters of the state.
A
  1. (C) As a general rule, the Commerce Clause prohibits a state from enacting regulations that discriminate or burden interstate commerce. In accordance with Dean Milk Co. v. City of Madison, 340 U.S. 349 (1951), a state may not create economic barriers to out-of-state products or impose on them costs that are more burdensome than those imposed on comparable local commerce in order to protect local interests. However, the Commerce Clause does not prevent the state, when acting itself as a purchaser or seller of goods, from buying only from or selling only to local business orfrom giving subsidies only to its residents. See Reeves v. Stake, 447 U.S. 429 (1980), where discrimination merely affects a market created by a state’s own purchases, and the state is, thus, a market participant, rather than a market regulator. Choices (A) and (B) are incorrect. A state has police powers under its 10th Amendment to create “gainful employment” for racial minorities. But there are constitutional limits to this aim, including the dormant Commerce Clause. As a general rule, the Commerce Clause prohibits a state from enacting regulations that discriminate or burden interstate commerce. In accordance with Dean Milk Co. v. City of Madison, 340 U.S. 349 (1951), a state may not create economic barriers to out-of-state products or impose on them costs that are more burdensome than those imposed on comparable local commerce in orderto protect local interests. Choice (D) is incorrect. Even if this were factually true, it wouLd not be legaLly relevant in overcoming the legal obstacles presented by the Commerce Clause.
141
Q
  1. In light of the current oil glut, many oil producing states have experienced extreme economic hardship. Due to the precipitous drop in oil prices, many oil companies have been forced to cut back on oil production and lay off many workers. As a result, unemployment has reached all-time high levels in several states. In order to alleviate this potentially catastrophic situation, the one of those state’s legislatures recently enacted a statute requiring that 10% of all oil produced within the state be purchased by the state and reserved for use by state residents. The purpose of the statute was twofold: (1) it was intended to stimulate the oil industry within the state by encouraging more production and exploration, and (2) it was designed to create an oil reserve so that state residents and industries would not suffer unduly from future oil shortages. Subsequently, Congress enacted a statute forbidding states to reserve local resources for local use in this manner.
    Is this state statute constitutional?
    (A) Yes, because Congress has not expressly forbidden states to reserve local resources for local use.
    (B) Yes, because the state statute requires that the oil be used for the general welfare of the people in emergency situations.
    (C) No, because a state may not protect its residents from out-of-state competition for its natural resources without the express permission of Congress.
    (D) No, because application of the statute denies non-oil producing companies to equal protection of the law, in violation of the Fourteenth Amendment.
A
  1. (C) In Constitutional Law, students must be familiar with the area of state regulation of interstate commerce. As a general rule, where Congress has not acted, and where no uniform national scheme of regulation exists, states are free to act in the regulation of interstate commerce, provided that the purpose or the effect of such regulation does not discriminate against interstate commerce. In this regard, states may not favor local interests by protecting them against out-of-state competition [Dean Milk Co. v. City of Madison, 340 U.S. 349 (1951)]. In addition, the Court has struck down state laws regulating the conservation of local natural resources. Since state laws enacted to protect local, publicly owned natural resources (e.g., minerals, wild animals) will generally be invalidated if they discriminate against interstate commerce, choice (C) is correct. Choice (A) is incorrect. Even in the absence of congressional prohibition, the commerce clause has a silently negative effect in its “dormant” condition. No state is permitted to pass Laws that unduly burden interstate commerce and, hence, violate the commerce clause of Article I, Section 8. In our example, the state appears to burden interstate commerce by hoarding local resources for itself, instead of permitting out-of-staters to buy them. Choice (B) is incorrect. A state enjoys the right under its 10th Amendment police powers to regulate welfare (as well as health, safety, and morals). However, the state may not invoke its police powers to violate the dormant commerce clause. In our example, the state appears to burden interstate commerce by hoarding local resources for itself, instead of permitting out-of-staters to buy them. Choice (D) is not the best answer. Non-oil-producing companies may have been the subject of discrimination by the legislature, but they are not members of a suspect or quasi-suspect class. Only rational review will be applied, which the state probably will pass with ease.

Basic Test
As a general rule, state regulation of interstate commerce is permissible if:
(1) the state reguLation does not discriminate against interstate commerce;
(2) the subject matter is not one that the Court concludes inherently requires uniform, national regulation; and
(3) the state interest underlying the regulation is not outweighed by the burden on interstate commerce (i.e., the “balance of interests” favors state, as opposed to national, interests).

142
Q
  1. A state built a racetrack that was specially designed and constructed for thoroughbred horseracing. State bonds were issued to finance the construction of the racetrack. The bond agreement provided that for the first five years the racetrack was in operation, at least $2 from each admission charge would be used to repay the bond debt. The bond agreement further stipulated that if the proceeds from the admission charges during the first five years were not sufficient to repay the bondholders, then at least $1 from each admission charge for the next five years would be applied to make the necessary bond payments.
    After the racetrack was built and in operation for two years, the state legislature passed a bill requiring the racetrack to admit all senior citizens over the age of 65 for the discounted admission fee of 50 cents. This law is probably
    (A) constitutional, because it is a justifiable exercise of the state’s police power.
    (B) unconstitutional, because it denies citizens under the age of 65 the equal protection of the law.
    (C) unconstitutional, because it impairs the obligation of the bondholders’ contract.
    (D) unconstitutional, because it is an ex post facto law.
A
  1. (C) Article I, Section 10 (Contract Clause) provides: “No State shall pass any Law impairingthe Obligation of Contracts.” In this example, the state was obligated under the terms of the bond agreement to apply at least $2 from each admission charge (to the racetrack) for the repayment of the bond debt. Consequently, it is an impairment of the Contract Clause for the state to pass a subsequent statute reducing the admission charge for senior citizens to 50 cents. By analogy, in United States Trust Co. v. New Jersey, 431 U.S. 1 (1977), the U.S. Supreme Court declared a New Jersey statute unconstitutional because the law impaired the state’s contractual obligation to the bondholders of the Port Authority of New York and New Jersey. Choice (D) is incorrect because the ex post facto clause applies to retroactive laws that are criminal in nature. Choice (A) is incorrect because it is an impairment of the Contract Clause forthe state to pass a subsequent statute reducing the admission charge for senior citizens to 50 cents. Choice (B) is incorrect; while those under 65 years may have been the subject of discrimination by the legislature, they are not members of a suspect or quasi-suspect class. Only rational review will be applied, which the state probably will pass with ease.
143
Q
  1. A state built a baseball stadium and issued bonds to finance its construction. The bond agreement provided that for the first five years the stadium was in operation, at least $2 from each admission charge would be used to repay the bond debt. The bond agreement further stipulated that if the proceeds from the admission charges during the first five years were not sufficient to repay the bondholders, then at least $1 from each admission charge for the next five years would be applied to make the necessary bond payments.
    Assume that three years after the stadium had been in operation, a subsequent session of the state legislature passed a bill entirely prohibiting baseball because four players were killed in playing mishaps. This statute is probably
    (A) constitutional, because it is a justifiable exercise of the state’s police power.
    (B) constitutional, because of the clear and present danger of baseball playing in the state.
    (C) unconstitutional, because it impairs the obligation of the bondholders’
    contract.
    (D) unconstitutional, because it violates the due process rights of the baseball team owners.
A
  1. (A) It is within the state’s police powers to enact legislation for the protection of the health, safety, and welfare of its citizens. Clearly, most state regulations place some burden on interstate commerce. In such situations, the Court balances the nature and extent of the burden (which the state regulation would impose on interstate commerce) against the merits and purposes of the regulation. Choice (B) is incorrect because the clear and present danger test applies to the abridgement or restraint of freedom of speech where there is a substantial threat of violence. This doctrine is not applicable in our case because the question does not relate to freedom of speech. Choice (C) is incorrect. Technically, the bondholders may still enforce the contract against the stadium because the prohibition against baseball does not necessarily impair the contractual obligations accrued by the state. Bondholders can still sue the state for payment, and the state is still liable to honorthose obligations. Choice (D) is incorrect. Generally, a property interest is not protected under the 14th Amendment’s Due Process Clause unless there is a reasonable expectation to continued receipt of the benefit [Board of Regents v. Roth, 408 U.S. 564 (1972)1. In our example, no such reasonable expectation to continued receipt of the benefit exists for the owners because there was nothing to guarantee that the stadium would be a success or that people would attend games.
144
Q
  1. A state built a casino and issued bonds to finance its construction. On five occasions, there were episodes of violence in various casinos in the state. The state police attributed the violence to greed and fear at the casinos.
    To prevent such violence, the state legislature passes a statute prohibiting all gambling at privately owned casinos in the state. Is this law likely to be held constitutional if most casinos in the state were owned by those from out-of-state?
    (A) Yes, because the act was expressly authorized by the state legislature.
    (B) Yes, but only if the local interest in safety outweighs the burden of interstate commerce.
    (C) No, because out-of-state casinos are part of interstate commerce.
    (D) No, because the statute violates the due process rights of the owners of the casinos.
A
  1. (B) As noted in the previous question, the Court generally attempts to balance the nature and extent of the burden (which the state regulation imposes on interstate commerce) against the merits and purposes of the regulation. Accordingly, choice (B) is the preferred answer. Choice (A) is incorrect because, even if this were factually true, it would not overcome the legal obstacles presented by the dormant commerce clause of Article I, Section 8. Under the dormant commerce clause, a state is not permitted to place undue burdens on interstate commerce. Choice (C) is incorrect. This fact, in itself, would not necessarily prohibit the state from regulating out-of-state casinos as long as it was done in a way that did not undu’y burden interstate commerce. Choice (D) is incorrect. Generally, a property interest is not protected under the 14th Amendment’s Due Process Clause unless there is a reasonable expectation to continued receipt of the benefit [Board of Regents v. Roth,408 U.S. 564 (1972)]. In our example, no such reasonable expectation to continued operation appears to exist.
145
Q
  1. A high school student was suspended for five days by school officials after he came to school wearing a beard. The school had a rule prohibiting any student from growing a beard or mustache while attending classes. The rule required an automatic five-day suspension in such cases. The student, who was aware of the rule prohibiting beards and mustaches, requested a trial-type hearing before the suspension was imposed.
    If the school board denies the student’s request for a trial-type hearing, which of the following statements is most accurate?
    (A) The suspension violated the student’s due process rights because it deprived him of his entitlement to an education.
    (B) The denial of a trial-type hearing violated the student’s due process rights because the suspension was arbitrarily imposed.
    (C) The denial of a trial-type hearing did not violate the student’s due process rights because under the circumstances, he had no right to a hearing.
    (D) There was no violation of the student’s due process rights because his conduct could be deemed so injurious to school discipline that it warranted suspension prior to a hearing.
A
  1. (C) Another key Multistate testing area deals with educational rights of students. In Goss v. Lopez, 419 u.s. 565 (1975), the U.S. Supreme Court held that fair procedures had to be established for determining the basis of the suspension of students from pubLic school systems. As a general rule, a student is not entitled to a trial-type hearing when his dismissal or suspension is with just cause. Certainly, a rule prohibiting students from growing beards or mustaches would seem to be compatible with the orderly operation of the school. On the other hand, the Court in Goss v. Lopez did note that when the suspension or termination of a student’s educational benefits may affect his employment or associational opportunities in the future, there may be a due process violation involved. Choice (D) is incorrect because the student was not entitled to a hearing under the circumstances. Choice (A) is incorrect. There is no fundamental right to an education under the 14th Amendment’s due process clause [Plylerv. Doe, 457 U.S. 202 (1982)] if harm to the student inheres not in his rightto an education but in harm to his reputation, which is protected by the 14th Amendment’s due process clause. See Goss v. Lopez, 419 U.S. 565 (1975). Choice (B) is incorrect. It’s not clear that the student’s right was denied arbitrarily. Even if the right were arbitrarily denied, the Supreme Court has not recognized that a student would be entitled to a trial, although at least a hearing by the school wouLd be required [Goss v. Lopez, 419 U.S. 565 (1975)].
146
Q
  1. A high school junior was charged by the school administration with violating certain sections of the disciplinary code, specifically, he was charged with being disrespectful to a teacher by using profanity and with using abusive language to a fellow student.
    The principal, sent the student’s parents a letter notifying them of the three-day suspension for the above-mentioned charges. The suspension was to take effect on February 1. The principal also included a copy of the disciplinary code in the letter. On January 19, the student and his mother met with the principal in his office to discuss the matter, and the student admitted that he used abusive language to a fellow student.
    On January 22, the student’s parents received a letter informing them that his teacher had upheld the school administration’s decision to suspend their son. They were then notified of a hearing on the recommended suspension to be held at the school. The parents did not attend this hearing, but were advised that the school board upheld the suspension, effective February 1.
    Which of the following most accurately summarizes the applicable rule of constitutional law with respect to the student’s suspension?
    (A) The student’s suspension deprived him of liberty and property without due process, as guaranteed by the Fourteenth Amendment.
    (B) The student’s conduct was protected under the First Amendment’s guarantee of freedom of speech.
    (C) The student’s suspension did not constitute a denial of due process.
    (D) The disciplinary code violated the student’s right to a compulsory school education.
A
  1. (C) In Hiliman v. Elliott (1977), the U.S. Supreme Court held that due process with respect to a three-day suspension of a student from public high school required that the student be given notice of charges, an explanation of evidence against him if he denied the charges, and an opportunity to present his version of the incident. In the instant case, the student was afforded such due process safeguards. Choice (A) is incorrect for the reasons stated above. Choice (B) is incorrect. A high school is not a public forum where speakers enjoy broad rights of free speech. Further, the school code that prohibited the student’s speech would not be subject to strict scrutiny, but only an easy rational review if it were deemed to be a proper time, place, and manner regulation. A proper time, place, and manner regulation does not prohibit the speech, but only its secondary effects, and such a regulation leaves reasonable alternatives for the speech. Here, the school code can be interpreted as prohibiting not the student’s criticism of both the teacher and his fellow student, but prohibiting the potential for distraction and disorder that his speech may cause. Plus, the student can probably air the same sort of message outside the school. Choice (D) is incorrect because there is no fundamental right to an education under the 14th Amendment’s due process clause (or anywhere else). [Plyler v. Doe, 457 U.S. 202 (1982)].
147
Q
  1. A state legislature passed a law requiring state universities to have male and female students live in separate dormitory facilities. The law was passed in order to curtail the increasing number of co-ed pregnancies on college campuses.
    Two students at a state university are engaged to be married and wish to share a dormitory room together. Citing the law, university housing officials refuse to allow them to live together in a campus dormitory facility.
    The students bring an appropriate action to challenge the constitutionality of the law. Which of the following correctly states the applicable burden of persuasion?
    (A) Since the law deals with the fundamental right of privacy, the state must show that it furthers a compelling state interest.
    (B) Since the law deals with the freedom of association, the state must show that it furthers a compelling state interest.
    (C) Since the law involves gender discrimination, the state must prove that the law is substantially related to an important state interest.
    (D) Since the law does not affect a fundamental right or involve gender discrimination, the burden is on the plaintiffs to show that it is not rationally related to a legitimate governmental interest.
A
  1. (D) Gender classifications will merit intermediate review. The court will ask whether there are important governmental interests and whether the means are substantially related. Paramount in gender cases is whether the statute is advancing some debilitating stereotype regarding females. Here, there is an important government interest (preventing pregnancies), but it’s not clear that such debilitating stereotypes are being advanced. Therefore, choice (D) is correct and choice (C) is incorrect. Choice (A) is incorrect because, since the students are not married, the privacy issue of related individuals living together is unripe. Choice (B) is incorrect. There is no fundamental right to associate in this example. It is similar to Village of Belle Terre v. Boraas, 416 U.S. 1 (1974). There, the Court subjected to an easy rational review a city code that prohibited the cohabitation by groups of three or more persons unrelated by blood or marriage. The Court stressed that the unrelated person— unlike married persons—do not enjoy a fundamental right to associate with each other. Similarly, in our example, the students are not yet married and, thus, have no fundamental rights to exercise.
148
Q
  1. The prime minister of a foreign country was assassinated by a group of right wing extremists. The prime minister’s death triggered a civil war between rival factions within the country. To prevent U.S. involvement in this country’s political crisis, Congress passed an appropriations bill prohibiting any funding for U.S. military operations in this foreign country.
    The appropriations bill was passed over a presidential veto. Thereafter, the President issued an executive order directing U.S. Navy and Army troops to the foreign country to restore order.
    Is this executive order constitutional?
    (A) Yes, because the President, as commander- in-chief of the armed forces, has the power to authorize such military expeditions.
    (B) Yes, because the President has inherent power to regulate foreign affairs as long as there is no formal declaration of war.
    (C) No, because the President cannot usurp Congress’s spending power by taking action that supersedes a congressional appropriations bill.
    (D) No, because the executive order was neither authorized by federal statute nor ratified by the Senate.
A
  1. (C) Another frequently tested Constitutional Law area is, to what extent does an executive agreement (or order) override an earlier enacted federal statute? According to Nowak, an executive agreement does not supersede inconsistent provisions of earlier acts of Congress. To be sure, the appropriations bill passed by Congress prohibiting funding for military operations in the foreign country will be controlling. The President does not have the power to override this congressional act by issuing a subsequent executive order. Choice (A) is incorrect. The President does enjoy Article II powers to act as a commander-in-chief. But he or she must first be summoned into action by Congress and, second, the President does not have the constitutional power to override a congressionaL statute in his or her capacity as commander-in-chief. Similarly, choice (B) is incorrect because, while the President does enjoy Article II powers to engage in various aspects of foreign policy, the President does not have the constitutional power to override a congressional statute in his or her role as a participant in foreign affairs. Choice (D) is incorrect. The chief problem is not that the executive order was not authorized by the Senate or Congress. The chief problem is that the President is not authorized by the Constitution to override an otherwise valid law; which would be a violation of the principle of the separation of powers.
149
Q
  1. A recent law school graduate took and passed the bar examination. Before the swearing-in ceremony, however, the graduate received a letter from the bar examiners indicating that his admission would be delayed until a character fitness investigation had been completed. The examiners also requested information as to whether the graduate had ever been arrested, convicted, or placed on probation for a criminal offense. The graduate had been arrested as a juvenile for possession of marijuana. He was placed in a special drug education program for first- time juvenile offenders and was told that the arrest would be expunged. Since the graduate believed that he had been totally exonerated of criminal liability, he didn’t report the arrest on his bar application form.
    The bar examiners had, in fact, received an anonymous letter making reference to the graduate’s juvenile arrest and suggesting that he was not morally fit to be admitted as an attorney. In addition, the letter provided detailed information about the facts related to the offense. As a result, the examiners hired a special investigator, to look into the matter. As part of the investigator’s inquiry, he went to the clerk of the juvenile court where the offense occurred and demanded access to the records concerning the graduate. The clerk refused, citing a state statute that required all court and police records relating to juveniles be sealed. After all other attempts to gain access to the records proved futile, the bar examiners filed suit in federal district court demanding access to the relevant documents.
    The court should rule that, as applied to this case, the state statute regarding the sealing of juvenile records is
    (A) constitutional, because juveniles are entitled to the equal protection of the laws.
    (B) constitutional, because the state has a strong and legitimate interest in rehabilitating juvenile offenders, and this is furthered by protecting them from embarrassment in later life through revelation ofjuvenile offenses.
    (C) unconstitutional, because the bar examiners, as a quasi-judicial agency, is entitled to have
    access to all relevant public documents.
    (D) unconstitutional, because it hinders the interests of justice by preventing the bar examiners from determining the fitness of candidates to practice law.
A
  1. (B) Juveniles are neither a suspect nor a quasi-suspect class of persons. Therefore, state laws dealing with juveniles will be scrutinized by the court under neither the compelling state interest nor the middle tier standard of review. The mere rationality test applies. Specifically, if the statute in question is rationally related to the furtherance of a legitimate state interest, the court will uphold it as constitutional. Choice (B) is correct because the sealing of court and police records of juveniles clearly serves the legitimate state interest in rehabilitation of juvenile offenders so as to be free from embarrassment in later life. Choice (C) is incorrect because by satisfying the requisite scrutiny of the rational basis test, the state law supersedes the right of the Committee of Bar Examiners to have access to otherwise public documents. Choice (A) is incorrect, juveniles may have been the subject of discrimination by the legislature, but they are not members of a suspect or quasi-suspect class. Only rational review will be applied, which the government probably will pass with ease. Choice (D) is incorrect because “hindering the interests of justice,” as used in this answer, is not a LegaLly relevant claim for purposes of constitutional analysis.
150
Q
  1. A city enacted an ordinance prohibiting the singing and chanting of songs from 1:00P.M. to 1:00A.M. by two or more persons if such noise is audible off the premises where the singing takes place. The ordinance expressly exempted from its purview the singing or chanting of religious songs.
    After the ordinance was enacted and went into effect, several college students were walking down the street on their way to a victory celebration following a big college football game. They began loudly chanting, “We’re number 1.. .we’re number 1.” One of the residents who lived on the street was greatly disturbed by the noise and reported the incident to the police, who were immediately summoned to the scene. The students who engaged in the chanting were arrested and charged with violating the ordinance.
    The students who were prosecuted now bring suit challenging the constitutionality of the city ordinance. Which of the following would constitute their WEAKEST argument against enforcement of the city ordinance?
    (A) The ordinance deprives persons of their freedom of expression, in violation of the First and
    Fourteenth Amendments.
    (B) The ordinance deprives persons of their liberty without due process of law because the ordinance is not related to any legitimate community interest.
    (C) The statutory language of the ordinance is vague and overbroad since it attempts to encompass all forms of singing.
    (D) The ordinance deprives the persons of the equal protection of the laws by impermissibly distinguishing between religiously inspired singing and all forms of singing.
A
  1. (B) The city ordinance may be attacked as a violation of protected 1st Amendment freedom of expression, since it prohibits all singing and chanting for 12 hours every day in areas that are traditionally viewed as public forums. Time, place, manner limitations on speech-related conduct are permitted when achieved by reasonable content-neutral regulations that further a significant governmental purpose. Such an ordinance must be narrowly drawn so as not to establish a total ban on protected rights of free speech. Since the city ordinance does not appear to satisfy this standard, a 1st Amendment free speech attack by the students will be a strong challenge. Therefore, choice (A) is incorrect. Choice (C) is also incorrect because it presents a strong challenge in the form of the vagueness and overbreadth doctrines. By proscribing protected as well as prohibited speech for half of each day everywhere in the city, the ordinance is clearly overbroad on its face. Similarly, due to the uncertainty-producing effect as to what conduct is restricted by the words “singing and chanting of songs” and “audible off the premises,” a vagueness challenge should be successful. Choice (D), another strong argument, is incorrect since nonreligious songs—which are certainly areas of protected speech—are being treated differently from religious songs, thereby raising an equal protection challenge to be reviewed using the strict scrutiny standard. By process of elimination, choice (B) is correct because the due process argument it presents is the weakest basis to attack the ordinance.
151
Q
  1. A citizen of a state was arrested and charged under a state statute making it a felony for “a male to sell or give alcoholic beverages to a female under the age of 14.” At his trial, the citizen attacked the validity of the state statute on federal constitutional grounds.
    The court will likely hold the statute to be
    (A) constitutional, because under the Twenty First Amendment, a state has exclusive authority to regulate the use and sale of intoxicating liquors.
    (B) constitutional, because the state could reasonably believe that young females need more protection than young males under these circumstances.
    (C) unconstitutional, because it lacks a legitimate purpose and, therefore, is violative of the Fourteenth Amendment.
    (D) unconstitutional, because the law treats males and females differently without adequate justification and, therefore, is violative of the Fourteenth Amendment.
A
  1. (D) Classifications based on gender are not “suspect,” but neither are they judged by the traditional (or rational basis) test. On the contrary, intentional discrimination against members of one sex are “quasi suspect” and violate equal protection unless they are substantially related to important government objectives. In Craig v. Boren, 429 U.S. 190 (1976), the U.S. Supreme Court invalidated an Oklahoma law permitting the sale of beer to women at age 18 but required males to be 21. The Court held that classification by gender must serve important governmental objectives and must be substantially related to the achievement of those objectives.
    Choice (A) is incorrect. The 21st Amendment does not maintain that a “state has exclusive authority to regulate the use and sale of intoxicating liquors.” Even if this were legally true, the state law should not violate other parts of the Constitution. Choice (B) is incorrect because this is not the standard under which we determine whether a state should be allowed to discriminate based on gender. Choice (C) is incorrect. When considering gender classifications, the Supreme Court takes particular offense at those governmental means that perpetuate a debilitating gender stereotype for girls and women. Such regulations are not likely to pass intermediate review, the review to which all gender classifications are subject.
152
Q
  1. During a hotly contested gubernatorial election, a local newspaper endorsed the candidacy of a corporate official of a chemical company. Inspired by a progressive revolt against the chemical company’s domination of the state government, the corporate official’s opponent, won by an extremely close margin.
    After the new governor took office, he vowed to get back at the company that owned the local newspaper for its newspaper’s endorsement of the corporate official. Using his influence, the new governor was instrumental in getting the state legislature to pass a bill that imposed a special tax on the sale of ink and paper used in the publication of newspapers and periodicals of general circulation. The tax bill was signed into law by the governor.
    The strongest constitutional basis upon which to challenge the validity of the tax would be the
    (A) equal protection clause of the Fourteenth Amendment.
    (B) bill of attainder provision under Article I, Section 10.
    (C) privileges or immunities clause of the Fourteenth Amendment.
    (D) First Amendment, as incorporated in the Fourteenth Amendment.
A
  1. (D) In Minneapolis Star& Tribune Co. v. Minnesota Commission of Revenue, 460 U.S.
    575 (1983), the Court found that a state tax statute imposing a “use tax” on paper and ink products consumed in the production of certain publications violated the 1st Amendment. Justice O’Connor found that the tax was not the result of an impermissible or censorial motive but that the tax violated the 1st Amendment because it singled out the press for special taxation and had the effect of targeting a small group of newspapers for a special tax. The majority did not employ equal protection analysis. In light of the Minneapolis Star & Tribune decision, the special Print Tax imposed on the publication of newspapers would best be challenged as a violation of the 1st Amendment. Choice (D) is, therefore, preferred over choice (A). Choice (B) is incorrect. ArticLe I, Section 10 prohibits a bilL of attainder, a Law passed by Congress to sentence someone for a crime in the absence of a trial. No such laws were passed in the example. Choice (C) is incorrect. The privileges or immunities clause of the 14th Amendment protects only U.S. citizens. A newspaper company is not a U.S. citizen.
153
Q
  1. After months of negotiations, the United States and Canada entered into a tax treaty that provided that neither country would impose income taxes on citizens of the other nation. The treaty, which was ratified by the Senate, was supported by professional baseball and hockey players. Many Canadian hockey players, who were employed by American teams and lived in the United States during the hockey season, lobbied for passage of the treaty, since they were subject to the payment of both U.S. and Canadian income taxes. In like manner, many American baseball players, who lived in Canada during the baseball season, objected to the same dual taxation.
    In violation of the treaty, a Canadian court convicted a U.S. citizen who resided in Canada, of illegally evading the payment of Canadian income taxes. The U.S. citizen, who was a member of a Canadian baseball team, resided in Canada during the baseball season. Following his conviction, the President announced that the previously effective tax treaty would no longer be abided by the United States. The President proclaimed that Canada’s refusal to honor the treaty rendered it invalid. As a result, the President ordered the Internal Revenue Service to begin collecting income taxes from Canadian citizens residing in the United States in the same manner that it collects taxes from other residents of this country.
    A Canadian citizen and resident of the United States, sues in an appropriate federal court, seeking a declaratory judgment that the treaty with Canada remains valid and effective. Therefore, he contends that the Internal Revenue Service may not collect U.S. income taxes from him.
    Which of the following is the strongest constitutional basis that may be urged in support of this claim?
    (A) The President’s unilateral termination of a treaty benefiting residents of the United States cannot be effective until a hearing is afforded to persons who would be affected by such action.
    (B) The courts have exclusive authority to determine whether a particular treaty has, in fact, been breached by another nation.
    (C) A ratified treaty is the supreme law of the land and, therefore, remains effective until superseded by another treaty or statute.
    (D) The treaty created a property right in Canadian citizens residing in the United States that cannot be taken away without just compensation.
A
  1. (C) Since a treaty is viewed on an equal footing with the Constitution and Acts of Congress, it is considered to be “the supreme Law of the land” (Article VI, Section 2). When a treaty is approved by two-thirds of the Senate, the President then ratifies it and the treaty becomes an agreement binding as an international obligation. Nowak, Constitutional Law, pg. 183. A treaty can be superseded by another treaty, properly ratified, or by subsequent Act of Congress. This result stems from the rule that where there is a conflict between a treaty and an Act of Congress, they are of equal weight, and the last one in time will control [Chae Chan Ping v. United States, 130 U.S. 581 (1889)]. Therefore, the strongest argument that the treaty between the United States and Canada is valid and effective is choice (C). The due process arguments presented in choices (A) and (D) are weaker choices, since nonpayment of income tax has never been regarded as an entitlement (i.e., welfare or disability benefit) for which a hearing or payment of just compensation must be afforded. Choice (B) is incorrect, since exclusive authority is not vested in the courts to determine whether another nation has breached a treaty. A nation might remain accountable to other parties to the treaty under the “law of nations” or even under the President’s broad authority to represent the United States in foreign relations [Clark v.Allen, 331 U.S. 503 (1947)].
154
Q
  1. The United States and Mexico entered into a tax treaty that provided that neither country would impose income taxes on citizens of the other nation. The treaty was ratified by the Senate. Recently, the President, angry over Mexico’s perceived failure to abide by the terms of the treaty, has decided that the United States would not honor any of the terms of the treaty. The President then ordered the Internal Revenue Service to begin collecting income taxes from Mexican citizens residing in the United States in the same manner that it collects taxes from other residents of this country.
    A Mexican citizen and resident of the United States sues in an appropriate federal court, seeking a declaratory judgment that the treaty with Mexico remains valid and effective. Therefore, he contends that the Internal Revenue Service may not collect U.S. income taxes from him.
    Which of the following is the strongest constitutional grounds for the federal court to refuse to decide the suit on its merits?
    (A) The citizen has no standing to bring his suit.
    (B) The case presents a nonjusticiable political question.
    (C) The case is moot because the President has already taken definitive action with respect to the effectiveness of this treaty.
    (D) The citizen is not entitled to a federal adjudication of this case because as a resident alien, he is not protected by the privileges or immunities clause of the Fourteenth Amendment.
A
  1. (B) Under Article III, Section 2, the Court possesses substantial power in the areas of international affairs and foreign policy. However, historically the Court often defers to the judgment of Congress and the President when a conflict involving foreign affairs arises. Nowak, Constitutional Law, pg. 179. The political question doctrine has been used as a means of evading judicial review when issues in the area of foreign relations are presented. In Bakery. Carr, 369 U.S. 186 (1962), Justice Brennan cited the following passage that “The conduct of the foreign relations of our Government is committed by the Constitution to the Executive and Legislative—the ‘political’—Departments of the Government, and the propriety of what may be done in the exercise of this political power is not subject to judicial inquiry or decision” [Oetjen v. Central Leather Co., 246 U.S. 297 (1918)]. Because the validity and enforcement of the treaty is an issue of international affairs, the political doctrine appears to be the strongest basis for the Court to refuse to decide the claim. The Baker case noted, however, that in the absence of any conclusive congressional or executive action, the Supreme Court will allow review of issues of foreign affairs, while still retaining its character of judicial independence and integrity. Choice (A) is incorrect. The requirements for standing would seem to be present: The citizen can show injury, since he will have to pay income tax to the United States; he can show causation because the President caused him to be taxed in the United States; and he can show redressability by showing that the President can prevent his injury by honoring the treaty. Choice (C) is incorrect. The case is not moot because the issues have not been resolved. The citizen will continue to be taxed in the United States. Choice (D) is incorrect because while it is true that only U.S. citizens are protected under the privileges or immunities clause of the 14th Amendment, aliens, even illegal aliens, are entitled other forms of constitutional protection.
155
Q
  1. A state has enacted a criminal statute prohibiting the mailing of obscene materials to any person. The owner of a publication company was prosecuted and convicted of violating the state obscenity law by mailing adults sexual literature that appealed to their prurient interests.
    During the owner’s trial, the judge instructed the jury that determining if the mailed materials were obscene depended in part on whether they were offensive to the average or normal person under contemporary community standards, and that the community standards test must be considered in light of the fact that many children reside in the community. Furthermore, the judge instructed the jury that in determining whether the materials were obscene, it could also consider evidence of pandering, or whether the materials were marketed purposely to appeal to the recipients’ prurient interest in sex.

The owner appealed her conviction, alleging a denial of her First Amendment rights. Which of the following is the strongest argument why the appellate court should reverse the owner’s conviction?
(A) The method by which materials are marketed or advertised is not probative of whether they are obscene.
(B) It is an unconstitutional invasion of privacy for the government to interfere with the content of closed mailings intended for the private use by consenting adults.
(C) Under the First Amendment, the community’s standards for children may not be applied in determining what constitutes obscenity for adults.
(D) Obscenity is to be determined by applying national standards, not contemporary community standards.

A
  1. (C) Choice (A) is incorrect because evidence of “pandering” on the part of the defendant (commercial exploitation for the sake of prurient appeal) may be probative on whether the material is obscene [Ginzburg v. United States, 383 U.S. 643 (1966)]. Choice (B) is incorrect because although private possession of obscenity at home is not a crime [Stanleyv. Georgia, 394 U.S. 557 (1969)], transportation and importation of obscene materials for either public or private use may be prohibited [United Statesv. Onto, 413 U.S. 139 (1973); United States v. Twelve 200-Ft. Reels, 413 U.S. 123 (1973)]. Choice (D) is, likewise, not our best answer because under the first two prongs of the Roth—Miller test, obscenity should be determined in accordance with “community standards,” not necessarily a “national standard.” In Jenkins v. Georgia, 418 U.S. 153 (1974), it was held that a juror may draw on knowledge of the vicinity from which he comes, and the court may either direct the jury to apply “community standards” without specifying what “community” or defining the standard in more precise geographic terms.
156
Q
  1. A 2-week-old baby had developed a severe case ofjaundice. A pediatrician informed the infant’s father that unless his daughter received immediate medical treatment, she would die. The father, who was very religious, refused to permit the pediatrician to administer the necessary treatment. He explained that his faith in his religion would restore his daughter to good health. As a consequence, the pediatrician sought an order from the state court, where the father was present, permitting the pediatrician to provide the necessary medical treatment to the infant.
    Which of the following is the father’s strongest constitutional argument against the court order?
    (A) The order violates the due process clause of the Fourteenth Amendment.
    (B) The order violates the equal protection clause of the Fourteenth Amendment.
    (C) The order violates the free exercise clause of the First Amendment, as incorporated by the Fourteenth Amendment.
    (D) The order violates the privileges or immunities clause of the Fourteenth Amendment.
A
  1. (C) American courts have upheld the right of the state to protect the health and safety of minor children over the religiously based objections of the children or their parents. Thus, the courts have uniformly appointed guardians to consent to necessary medical treatment (such as blood transfusions) for children even though the treatment violates the child’s or parent’s religion. It should be noted that when objection to medical treatment is based on religious principles, a serious free exercise clause problem is presented. However, if the individual’s preference is not religious in nature (and there is only a conflict between personal choice relating to health and state medical regulations), the decision would be resolved under the due process clause. Choice (A) is incorrect because the father was afforded due process because a judge reviewed the facts and issued an order. Choice (B) is incorrect. The equal protection clause provides heightened protection for groups whose legal classifications are suspect. One of those categories is religion. However, the court order would not seem to be directed at religionists while leaving non-religionists alone. That is, the court would probably have issued the same order for an atheist parent who objected to the doctor’s treatment on secular philosophical grounds. Choice (D) is incorrect. The privileges or immunities clause protects people against discrimination by states against out-of-staters. That does not appear to be the case in our example.
157
Q
  1. A 3-week-old baby, came down with a very high fever. The baby’s pediatrician informed the mother that unless her son received immediate medical treatment, he would die. The mother objected on religious grounds. She claimed that it would be better for her son to die if that was his fate. The pediatrician sought an order from the state court. The mother was present with her lawyer at the court.
    In deciding whether it may issue such an order in face of all relevant constitutional defenses by the mother, which of the following must the state court consider?
    (A) Whether medical treatment is necessary to save the baby’s life.
    (B) Whether the mother’s refusal to authorize medical treatment is justified on the basis of current knowledge.
    (C) Whether the hospital is owned and operated by the state.
    (D) Whether the mother is a taxpayer of the state in which the court is located.
A
  1. (B) This is an example of a very tricky Multistate question because the test maker knows that many students will incorrectly choose choice (A). Clearly, choice (A) is arguably correct. Choice (B), however, is the best answer because, in deciding whether to order medical treatment for a minor, the courts will balance the objections of the parent against the health and safety of the minor. See Jehovah’s Witnesses v. King CountyHospital, 278 F. Supp. 598 (1968) fora more detailed analysis of the modern balancing test. Choice (C) is not the best answer. This would not be a relevant legal consideration for purposes of analysis under the free exercise clause; a publicly owned hospital would not be permitted to violate free exercise clause rights. Choice (0) is incorrect because even if Janet had never paid her taxes, she would still be entitled to her free exercise clause rights.
158
Q
  1. A state statute provides that an illegitimate child may not inherit from his father’s property. The state law, however, does permit illegitimate children to inherit from and through their mothers. A man died intestate, leaving neither spouse nor any children other than an illegitimate son. The man’s wife and his daughter died one year later in a motor vehicle accident. The illegitimate son filed suit in an appropriate court alleging that the state statute, which bars an illegitimate child from sharing in his father’s estate is invalid, and that he should be declared the lawful heir of the man’s estate.
    In challenging the validity of the state statute, will the illegitimate son prevail?
    (A) Yes, because most state laws that discriminate against illegitimate children have been invalidated to ensure that a state’s concern over illicit relationships is not the basis for punitive measures against the product of such a relationship.
    (B) Yes, because he has been deprived of property without due process, since his fundamental right to inherit has been compromised without a compelling state need.
    (C) No, because a state may promote the just and expeditious disposition of property at death by denying intestate succession to all illegitimate children.
    (D) No, because discrimination against illegitimate children is not suspect and, therefore, the law does not violate the equal protection clause if it is substantially related to a legitimate state interest.
A
  1. (A) Iliegitimacyis a “quasi-suspect” classification (as in gender) and, thus, subject to an intermediate level of scrutiny. Therefore, whenever a state (or other non-federal government entity) passes a law discriminating against illegitimates, the burden of persuasion is on the state. In such cases, the state must demonstrate that the law furthers “an important state interest,” otherwise it is deemed unconstitutional. According to the holding in Trimble v. Gordon, 430 U.S. 762 (1977), a state cannot absolutely exclude illegitimate children from inheriting from their intestate fathers. Therefore, (A) is correct, and choice (C) is incorrect. Choice (D) is partly correct inasmuch as illegitimacy is not a “suspect” classification, though choice (0) is wrong because in Trimble, a similar Illinois statute excluding illegitimate children inheriting from their father was deemed unconstitutional as not being substantially related to a legitimate state interest. Choice (B) is incorrect. There is no fundamental right to inherit under the due process clause.
159
Q
  1. A state has recently enacted a statute prohibiting the disposal of any nuclear wastes within the state. This law does not contravene or conflict with any federal statutes. A man operates a company in the state that is engaged in the disposal of nuclear wastes. Subsequent to the passage of the state statute, the man, not yet aware of the new law, entered into contracts with many out-of-state firms to dispose of their nuclear wastes in the state. On account of this new law, however, the man will be unable to perform these contracts.
    Assume that the man has standing to challenge this state law. Which of the following presents his strongest constitutional grounds to challenge the state law prohibiting the disposal of nuclear wastes within the state?
    (A) The commerce clause.
    (B) The equal protection clause of the Fourteenth Amendment.
    (C) The privileges and immunities clause of Article IV, Section 2.
    (D) The contract clause.
A
  1. (A) The strongest constitutional basis to challenge the state Law is the Commerce Clause. The disposal of nuclear waste is an item of interstate commerce, as is the disposal of garbage. In City of Philadelphia v. New jersey, 437 U.S. 617 (1978), the Court held a New Jersey statute prohibiting out-of-state waste disposal in New Jersey dump sites to be invalid under the Commerce Clause since the disposal of garbage was an item in interstate commerce. By analogy, the state statute is discriminatory because it creates an undue burden on out-of-state nuclear firms by prohibiting them from entering the state to dispose of their waste. Contract Clause is a much weaker argument for the man since private contracts can be validly modified by state legislation, which is necessary to achieve a legitimate public interest. Choice (A) is the best answer. Choice (B) is incorrect. The man may have been the subject of discrimination by the legisLature, but the man is not a member of a suspect or quasi-suspect class. Only rational review will be applied, which the state probably will pass with ease. Choice (C) is incorrect. The privileges and immunities clause of Article IV, Section 2 prohibits states from discriminating against nonresidents. The statute here, however, does not; it prohibits the disposal of any nuclear wastes within the state, which does not discriminate against non residents. Choice (D) is incorrect. The contract clause of Article I, Section 10 prohibits states from impairing the obligation of contracts. However, the contracts clause applies only to contracts that were made prior to the state statute that ostensibly impairs fulfiLlment of their obligations.
160
Q
  1. A shrimp fishery is located in the coastal waters of a state. A large part of the catch is either frozen or canned, and distributed nationwide. The state legislature passed a statute requiring payment of a license fee of $25 for each shrimp boat owned by a resident and $2,500 for each boat owned by a nonresident.
    A resident of a neighboring state was a commercial shrimp fisherman who was denied a license after refusing to pay the $2,500 fee. The resident brought suit in federal court challenging the constitutionality of the state shrimp boat licensing statute. The federal court should
    (A) hear the case on its merits.
    (B) dismiss the suit because the resident lacks standing.
    (C) dismiss the suit because it involves a question of state law.
    (D) abstain from jurisdiction because the constitutional issue should be litigated first in a state court.
A
  1. (A) Choice (B) is incorrect because the resident can show that enforcement of the patently unconstitutional statute causes him injury in fact, economic and otherwise. Choice (C) is incorrect because in the leading case of Toomer v. Witsell, 334 U.S. 385 (1948), the U.S. Supreme Court ruled a similar South Carolina law unconstitutional of violating the privileges and immunities clause of Article IV. The test is whether there are valid reasons for a state to make distinctions based on one’s state citizenship and whether the degree of discrimination bears a “close relation” to these reasons. Choice (D) is incorrect because the suit does not in Large part turn on an unsettled question of state law. Moreover, the abstention doctrine is inappropriate where a state court ruling would not be helpful in a determination of the constitutional issue.
161
Q
  1. There is a thriving source of crawfish that live in a state. The state owns a fleet of boats that trawl for crawfish. The state is willing to sell the crawfish to in-staters for $1 per pound and to out-of-staters who come to the state for $5 per pound. The state felt that the increased fee for out-of-staters was a reasonable contribution toward the protection they received from the state.
    Assume that the federal court decided to hear the case. The statute is likely to be found
    (A) constitutional, because it is a valid exercise of the state’s police power.
    (B) constitutional, because the fee was a reasonable contribution toward the protection that the state government gave nonresidents.
    (C) unconstitutional, because it places a discriminatory burden on interstate commerce.
    (D) unconstitutional, because it constitutes a violation of the privileges and immunities clause under Article IV.
A
  1. (D) Choice (D) is the best answer in light of the Toomer decision. In the leading case of Toomer v. WitseIl, 334 U.S. 385 (1948), the U.S. Supreme Court ruled a similar South Carolina law unconstitutional of violating the privileges and immunities clause of Article IV. The test is whether there are valid reasons fora state to make distinctions based on one’s state citizenship and whether the degree of discrimination bears a “close relation” to these reasons. Choice (A) is incorrect. The state does have police powers under the 10th Amendment to regulate welfare, including environmental and economic welfare. However, in exercising these police powers, the state may not violate the Constitution. Here, State will violate the privileges and immunities clause of Article IV. Choice (B) is incorrect because it is not a legally relevant consideration. Choice (C) is not the best answer. The state is acting like another market participant who is selling crawfish and, thus, immune from analysis under the dormant commerce clause.
162
Q
  1. A state has passed a law that provides that only residents of the state who are citizens of the United States can own agricultural land in the state. A out- of-state farmer who is a U.S. citizen has contracted, subsequent to the aforementioned law, to purchase a farm from a landowner, which is located in the state. The landowner, who is a resident of the state, has been informed by his attorney that his sales agreement with the farmer is null and void under state law.
    Which of the following is the best constitutional argument to contest the validity of the state statute?
    (A) The contract clause prohibition against a state from enacting any law that will impair the obligation of contracts.
    (B) The privileges and immunities clause of the Fourteenth Amendment.
    (C) The privileges and immunities clause under Article IV, Section 2.
    (D) The national property power provision under Article IV, Section 3.
A
  1. (C) There are two privileges and immunities clauses contained in the U.S. Constitution. First, Article IV, Section 2 provides: “The citizens of each state shall be entitled to all Privileges and Immunities of citizens in the several states.” This clause prohibits state discrimination against nonresidents (of the state) in respect to “essential activities” or “basic rights” unless the discrimination is closely related to a substantial purpose. Since ownership of property involves a “basic right,” this constitutional provision is most applicable. The second Privileges or Immunities provision is contained in the 14th Amendment and provides in part: “No state shall make or enforce any law which shall abridge the privileges or immunities of citizens of the United States.” These privileges and immunities have been limited to those rights that arise out of the relationship of the individual and the national government (e.g., the right to travel freely from state to state; the right to petition Congress for redress of grievances; the right to vote for national officers; and the right to assemble peaceably). Therefore, choice (C) is stronger than choice (B). Choice (A) is incorrect because the contracts clause applies only to those contracts made prior to the enactment of the state statute that ostensibly impairs the obligation of contracts. Choice (D) is incorrect. The national property clause in Article IV, Section 3 mentions nothing about the right of local governments to enact the sort of legislation referenced in our example.
163
Q
  1. Recently, Congress enacted a statute requiring all boat owners to register their boats with a newly created federal boat registry. Among the purposes of the statute are the prevention of theft of boats in coastal waters and the protection of the rights of individual boat owners throughout the United States.
    Congress enacted the statute despite the fact that all states require boat owners to register their craft with the state department of motor vehicles. In addition, there is uncontradicted evidence that most stolen boats are kept or resold in the state in which the theft occurred. Nonetheless, an increasing number of boats are transported to other states and other countries for resale.
    Is the statute likely to be held constitutional?
    (A) No, because most stolen boats remain within the state in which they were stolen.
    (B) No, because the registration of boats is a matter reserved to the states by the Tenth Amendment.
    (C) Yes, because Congress could determine that the transportation of stolen boats affects interstate commerce.
    (D) Yes, because Congress has the power to regulate property for the general welfare.
A
  1. (C) Under the “affectation doctrine,” Congress has the power to regulate any activity—whether carried on in one state or many—that has an appreciable effect upon interstate commerce. Although federal power over interstate commerce is potentially “all-persuasive,” it is not necessarily exclusive. Thus, Congress has absolute authority to define the distribution of federal and state regulatory power over interstate commerce. Just as Congress may permit state regulation of interstate commerce, it may also prohibit state regulation of any part of interstate commerce. Choice (A) is incorrect because even if this were factually true, Congress’s regulation would still be valid under the Commerce Clause of Article I, Section 8. Choice (B) is incorrect. The 10th Amendment permits the states to exercise police powers for purposes of regulating health, safety, welfare, and morals. However, Congress may also regulate in these areas if it can find authorization in the Constitution. In this example, Congress can find such power in the Commerce Clause. Finally, choice (D) is incorrect. Article I, Section 8, reads: “The Congress shall have Power to lay and collect Taxes, Duties, Imposts and Excises, to pay the Debts and provide for the common Defence and general Welfare of the United States.” The Supreme Court has taken the reference to “general Welfare” to mean that Congress enjoys great discretion in how it chooses to allocate money for the public [United States v. Butler, 297 U.S. 1 (1936)]. There is no such allocation of money in our example; just a regulation.
164
Q
  1. A state legislature has recently enacted a statute requiring all prospective voters in state elections
    who wish to write-in a candidate to print the candidate’s full name, and designate the office for which the candidate is running. The statute provides that such information must be written on the ballot in ink in an appropriate space.
    A write-in candidate for the office of Attorney General is a Chinese-American. The candidate is of the opinion that he needs a large turnout of Chinese voters in order to win the election. As a result, his campaign manager decides to mail to every registered Chinese voter a tear-off sticker, which bears the candidates name along with the office of Attorney General. Since many native Chinese people are not proficient in reading and writing English, the campaign manager believes that many of the voters will have difficulty writing the candidate’s name and office on the ballot. As a result, the campaign manager has mounted an extensive media campaign to inform voters on how to apply the stickers to the ballot.
    Five months prior to the election an election official notifies the candidate’s campaign committee that the tear-off stickers do not comply with the state statute. In her letter, the official explains that state election officials are of the opinion that it is necessary for potential voters to write the candidate’s name in ink. Therefore, she concludes that the stickers do not comply with statutory requirements.
    Three weeks later, the candidate filed suit in federal district court against the election officials, claiming that their interpretation of the state statute violates the U.S. Constitution. Thereafter, one of the candidate’s opponents filed suit in state court seeking to prevent state election officials from counting any write-in ballots with stickers. The state court has now scheduled a prompt hearing on this matter. In addition, the state court has indicated that it hopes to render a decision on the merits within the next three weeks.
    Which of the following statements is correct concerning the federal court’s adjudication of the candidate’s suit?
    (A) The federal court should hear the case on the merits.
    (B) The federal court should refuse to hear the case because it presents a nonjusticiable political question.
    (C) The federal court should refuse to hear the case because of the abstention doctrine.
    (D) The federal court should remand the case to the state court to decide the constitutional issue presented.
A
  1. (C) Here’s a reaLly difficult choice between abstention and justiciability. Generally speaking, a federal court will “abstain” or temporarily “stay its hand” whenever presented with an unsettled question of state law. Thus, federaL courts will permit the state courts to resolve such issues before exercising jurisdiction. Choice (B) is less preferred because questions involving political rights (such as deprivation of the right to vote or legislative apportionment) are not necessarily “political questions.” They may involve claims under the Equal Protection Clause of the 14th Amendment or even the 15th Amendment, which have well-developed judicial standards. Refer to Bakery. Carr, 369 U.S. 186 (1962), for a more detailed analysis of justiciability and factors regarding political questions. Finally, choices (A) and CD) are not the best answers because, as is stated above, the court will abstain from hearing this matter.
165
Q
  1. A state has recently enacted a statute requiring all prospective voters in state elections who wish to write-in a candidate to print the candidate’s full name, and designate the office for which the candidate is running. The statute provides that such information must be written on the ballot in ink in
    an appropriate space.
    A write-in candidate is a German-American and is of the opinion that he needs a large turnout of German voters in order to win the election. As a result, his campaign manager decides to mail to every registered German voter a tear-off sticker, which bears the candidate’s name and office for which he is running. Since many native German people are not proficient in reading and writing English, the campaign manager believes that many of the voters will have difficulty writing the candidate’s name and office on the ballot. As a result, the campaign manager has mounted an extensive media campaign to inform voters on how to apply the stickers to the ballot.
    Five months prior to the election an election official notifies the candidate’s campaign committee that the tear-off stickers do not comply with the state statute. In her letter, the official explains that state election officials are of the opinion that it is necessary for potential voters to write the candidate’s name in ink. Therefore, she concludes that the stickers do not comply with statutory requirements.
    Three weeks later, the candidate filed suit in federal district court against state election officials, claiming that their interpretation of the state statute violates the U.S. Constitution.
    Which of the following sets forth the strongest constitutional argument the candidate could make against the interpretation of the statute by the state officials?
    (A) It unreasonably discriminates against German voters who lack a proficiency in the English language.
    (B) It unreasonably discriminates against write-in candidates for public office.
    (C) It unreasonably interferes with the exclusive federal election power as embodied in the Fifteenth Amendment.
    (D) It unreasonably interferes with the 1965 Voting Rights Act outlawing literacy tests.
A
  1. (B) From a standpoint of standing, choice (B) is a better answer than choice (A). This is because a person asserting the violation of a constitutional or statutory right must show a “direct and immediate personal injury” due to the challenged action. Choice (B) is correct because the candidate will argue that he is adversely affected by the election official’s interpretation of the statute. On the other hand, it is more difficult for a claimant to have standing to assert the rights of third persons. That’s why the candidate will be in a better position to bring suit for himself rather than on behalf of the German voters in choice (A). Choices (C) and (D) are wrong because federal election laws do not prevent a state from regulating absentee ballots. In fact, in McDonald v. Board of Elections, a state law granting the right to vote by absentee ballot to only certain classes of people was upheld. The Court ruled that the right to vote was not involved because the state did not preclude the appellants from voting by other means.
166
Q
  1. A write-in candidate is a Mexican-American and is of the opinion that he needs a large turnout of Mexican voters in order to win the election. As a result, his campaign manager, decides to mail to every registered Mexican voter a tear-off sticker, which bears the candidate’s name and office for which he is running. Since many native Mexican people are not proficient in reading and writing English, the campaign manager believes that many of the voters will have difficulty writing the candidate’s name and office on the ballot. As a result, the campaign manager has mounted an extensive media campaign to inform voters on how to apply the stickers to the ballot.
    Five months prior to the election an election official notifies the candidate’s campaign committee that the tear-off stickers do not comply with the state statute. In her letter, the official explains that state election officials are of the opinion that it is necessary for potential voters to write the candidate’s name in ink. Therefore, she concludes that the stickers do not comply with statutory requirements.
    Three weeks later, the candidate filed suit in federal district court against state election officials claiming that their interpretation of the state statute violates the U.S. Constitution.
    Which of the following, if established, sets forth the strongest constitutional argument supporting the election official’s interpretation of the statute?
    (A) Voter turnout among registered Mexican American voters who lack proficiency in English has been less than 1% of the entire state total in recent elections.
    (B) A state statute requires that each voting booth be equipped with an ink pen to facilitate the writing in of votes.
    (C) Since large numbers of write-in votes generally increase the time needed to count all votes, it is necessary to cast such votes in a uniform manner.
    (D) The potential for voting fraud is substantially greater with preprinted stickers.
A
  1. (D) Sometimes on the Multistate it is necessary to simply use common sense. Here, for example, choice (D) states that the decision should be upheld because it minimizes the potential for election fraud. Clearly, under a “strict scrutiny” standard of review, this will serve a compelling state interest and best justify the election official’s action. Choice (A) is incorrect because even were this factuaLly true, the right to vote is a fundamental right, and the state must try to protect it. Choice (B) is incorrect because this would not necessarily resolve the problem about lack of English proficiency. Choice (C) is not the best answer. Even if this were factually true, the right to vote is a fundamental right, and the state must try to protect it. Trying to save time in this manner would hardly seem a compelling state interest.
167
Q
  1. A state assemblyman made a visit to a foreign country. To protest U.S. foreign policy, the assemblyman and the foreign country’s leader issued a joint statement criticizing the United States’ involvement in the political affairs of neighboring countries.
    Following the assemblyman’s return to the United States, he was prosecuted under a federal criminal statute making it unlawful for any citizen not specifically authorized by the President to negotiate with a foreign government for the purpose of influencing the foreign government in relation to a dispute with the United States. The law further provides that “any citizen who knowingly counsels, aids, or abets a foreign government in a dispute with the United States.., shall, upon conviction… be punished by imprisonment for not more than five years or a fine of not more than $10,000, or by both fine and imprisonment….”
    Which of the following is the strongest constitutional basis for upholding the validity of the aforementioned federal statute?
    (A) Federal criminal laws dealing with international affairs need not be as specific as those dealing with domestic affairs.
    (B) Under its enumerated powers, Congress may legislate to preserve the monopoly of the national government over the conduct of U.S. foreign affairs.
    (C) The President’s inherent power to negotiate for the U.S. with foreign countries authorizes him to punish citizens who engage in such negotiations without permission, even in the absence of statutory authorization.
    (D) Article I, Section 8 of the Constitution grants Congress concurrent power with the President to regulate external affairs with foreign countries.
A
  1. (B) The power of the federal government in regard to foreign affairs is excLusive. Powers, such as those to make treaties, to declare war, to conclude peace, and to maintain diplomatic relations with other nations are deemed “necessary concomitants of nationality,” which has been interpreted to mean that they would have vested in the federal government even if they had never been mentioned in the Constitution, and are as broad as similar powers held by other sovereign nations [United States v. Laws, 163 U.S. 258 (1896)]. Choice (A) is incorrect. Federal criminal laws dealing with international affairs, like those dealing with domestic affairs, must meet the same requirements for specificity insofar as both must adhere to constitutional norms of due process. Choice (C) is incorrect. The President does not enjoy constitutional authority to punish citizens in the absence of congressional authorization. If the President possessed such powers, he would violate the principle of the separation of powers by exercising rights that belong to the legislature as well as those that belong to the judiciary. Choice (D) is not the best answer. Congress and the President may enjoy concurrent powers to regulate external affairs with foreign countries, but the powers are not the same, and Congress may invoke its own constitutional powers under Article Ito regulate foreign affairs.
168
Q
  1. Which of the following executive orders will most likely be found unconstitutional?
    (A) The President issued an executive order requiring all executive branch employees to use exclusively one brand of ballpoint pens and pencils as their writing utensils. According to a study, the federal government could save in excess of $250,000 a year in office supply costs if all executive agencies were to use standardized pens and pencils.
    (B) The President issued an executive order requiring all executive branch employees to wear only white shirts and blouses during regular working hours. According to the President’s directive, executive employees are prohibited from wearing colored (e.g., blue or yellow) or striped shirts and blouses while on duty. The President issued the order in an effort to establish a uniform dress code for all executive employees.
    (C) In 1887, Congress passed a law establishing a federal Commission of Birdwatchers. The Commission, which is still in effect, consists of seven members who are appointed by the President. The Commission’s main function is to go on periodic retreats to photograph and study North American birds and their migratory habits. Believing that the Commission is archaic, the President decides that any future funding will simply be a waste of money. He thus executes an executive decree abrogating the Commission of Birdwatchers.
    (D) A devastating hurricane damages an island which is part of a foreign country. The storm destroys many homes, resulting in death and injury to thousands. In response to a request from the foreign government for emergency aid, the President, without seeking the advice and consent of the Senate, issues an executive decree authorizing U.S. Army troops to the island to provide medical and humanitarian assistance.
A
  1. (C) The President may properly issue an executive order to ensure efficient operation of all executive agencies. The President’s executive order to require all executive branch employees to use exclusively one brand of ballpoint pens and pencils could be upheld as a cost-efficient measure falling within the permissible scope of the President’s power as Chief Executive. President Ford issued an executive order requiring all executive agencies to use only letter-size 8-1/2” X 11” paper. This cost-saving measure was held constitutional. Choice (A) is, therefore, incorrect. Choice (B) is also incorrect because it invites a similar type of regulation, namely that all executive branch employees wear only white shirts and blouses during regular working hours. Such an executive order could further efficient operation of executive departments based on the notions of teamwork and single-mindedness, which a uniform dress code inspires. Choice (D) is incorrect because, under the President’s broad emergency powers as Commander-in-Chief, he may summon U.S. Army troops (or the National Guard) by executive decree for medical and humanitarian purposes without advice or consent of the Congress. Choice (C) is correct because an executive order may not be issued to supersede an earlier act of Congress. Nowak, Constitutional Law, 3rd Ed., p. 211. Since the Commission of Bird- watchers was established by federal law, the President’s executive order to abolish the Commission would be unconstitutional.
169
Q
  1. The President announced that a newly created military rapid deployment force would be engaging in joint training exercises with the military forces in a foreign country. Following the President’s announcement, Congress enacted a statute that unequivocally prohibited “the U.S. Armed Forces from conducting military exercises with foreign military forces unless Congress has received notice of such intention to take such action at least three months before the joint military exercises become effective.”
    This statute is most likely
    (A) constitutional, because the President, in this instance, has not been called by Congress into actual service as Commander-in-Chief.
    (B) constitutional, because of Congressional power under the War Powers Act.
    (C) unconstitutional, because of the President’s authority to execute the laws of the United States.
    (D) unconstitutional, because of the President’s authority as Commander-in-Chief of the Armed Forces.
A
  1. (A) Article II, Section 2 states: “The president shall be Commander in Chief of the Army and Navy of the United States, and of the Militia of the several States, when called into the actualservice of the United States. . - .“ Congress has not called the President into actual service in our example. Therefore, choice (A) is correct, and choice (D) is incorrect. Choice (B) is incorrect because the war powers resolution applies to situations when the armed forces are engaged in hostilities or where there is imminent involvement in hostilities. The facts in our case do not indicate that the armed forces were involved in hostilities. Choice (C) is incorrect. The President has the authority under Article II to execute laws passed by Congress, not to create his or her own.
170
Q
  1. Congress recently enacted a statute permitting a governmental agency to make a gift of federal property to private individuals or organizations, provided that it be used “to further good relations and better understanding with foreign governments.”
    The Secretary of Defense planned to give an old military cargo plane to a national organization who supports future pilots. Before making the gift, the Secretary was approached by the head of a church. The church leader indicated that he would be sponsoring a worldwide crusade and suggested to the Secretary that such an undertaking would serve to strengthen relations with foreign governments. The Secretary donated the plane to the church instead of the organization.
    Who would have the best standing to challenge the Secretary’s action?
    (A) A citizen of the United States.
    (B) A taxpayer of the United States.
    (C) The national organization.
    (D) A state within the United States.
A
  1. (C) Here, you must be familiar with the decision in Valley Forge Christian College v. Americans United, 102 S.Ct. 752 (1982), in which a federal taxpayer was denied standing to challenge a gift of federal surplus property to a church college asviolating the establishment clause. The Court held that the challenge was not to a federal expenditure under Congress’s taxing and spending power, but to an exercise of Congress’s power “to dispose of property belonging to the United States” under Article IV, Section 3, Clause 2. Many students will incorrectly choose choice (B) based on the holding in Flast v. Cohen, 392 U.S. 83 (1968). According to Flast, a federal taxpayer has standing to challenge a federal appropriation and spending measure if she can establish the challenged measures as (1) enacted under Congress’ taxing and spending powers, and (2) it exceeds some specific limitation on the taxing and spending powers. Note: Be advised, however, that taxpayers have not been successful where the appropriation measure is challenged on grounds other than the 1st Amendment’s establishment clause. Choice (A) is, likewise, incorrect because persons have no standing as citizens to claim that federal statutes violate the Constitution. Similarly, choice (D) is incorrect because absent congressional authorization of such a suit, a state has no standing to attack a federal statute on the grounds that Congress has exceeded its delegated powers. Under such circumstances, the state has not suffered any injury, and the matter is purely a “political question” [Massachusetts v. Mellon, 262 U.S. 447 (1923)]. Once again, by process of elimination, choice (C) is the strongest answer.
171
Q
  1. Congress recently passed a law that would grant, for free, federally owned buses to a religious organization. The congressional statute stipulates that the religious organization must use the buses to travel across America to spread the moral message of sexual abstinence to teenagers at school assemblies.
    Which of the following is the strongest constitutional grounds for invalidating the gift of the buses to the religious organization?
    (A) The gift violates the equal protection rights of secular organizations.
    (B) The gift violates the establishment clause.
    (C) The gift is a taking of federal property without just compensation.
    (D) The gift violates the commerce clause.
A
  1. (B) In order not to violate the establishment clause, a statute (or other government action) must (1) have a secular purpose; (2) have a principal or primary effect that neither advances nor inhibits religion; and (3) not foster excessive government entanglement with religion [Lemon v. Kurtzman, 403 U.S. 602 (1971)]. As a practical matter, the Establishment Clause bars government sponsorship of religion, government financial support of religion, and active involvement in religious activities. Choice (A) is incorrect. Legal classifications that discriminate against religious groups would trigger strict scrutiny. But the classification in our example discriminates against secularists. Thus, the classification will be subject to rational review, which the federal government should be able to pass with ease. Choice (C) is incorrect. The 5th Amendment’s takings clause prohibits the government from taking private property without just compensation. In our example, the property belongs to the federal government. Choice (D) is incorrect. Idiomatically, Congress cannot violate the Commerce Clause; rather, Congress either has authority under the Commerce Clause or it does not. In our example, Congress may pass this legislation under the Commerce Clause because its stipulation regarding the terms of the devise are related to interstate commerce.
172
Q
  1. A city has granted a license to a private utility company to provide electrical service to the residents in the city. After approving the license to the utility company, the city council then passed a measure by which the utility company was required to insert in its monthly billing statements a letter from a private consumer group criticizing the high cost of electrical service. The cost of printing and mailing the monthly letter was paid entirely by the consumer group. Nonetheless, the utility company vehemently objected to having such a critical letter enclosed in their monthly billing statements. However, the city council warned the utility company that unless it complied with the directive, the city would revoke its license for electrical service. The utility company filed suit in federal court seeking a hearing prior to its license being revoked by the city council.
    Which of the following constitutional provisions would provide the utility company with the strongest grounds with which to challenge the city council measure?
    (A) The due process clause.
    (B) The equal protection clause.
    (C) The privileges and immunities clause of Article
    Iv.
    (D) The commerce clause.
A
  1. (A) The 14th Amendment’s Due Process Clause provides procedural safeguards against arbitrary deprivation whenever a governmental agency acts to deprive a person of her “life, liberty, or property” interests. Since a corporation or company is considered a “person,” due process protection would extend to General Electric in this problem. Choice (A) is clearly correct because the Parkview City CounciL is threatening to revoke its license for electrical service unless General Electric agrees to enclose the consumer letter in its monthly billing statements. Thus, if General Electric failed to comply with the city directive, it would face losing its electrical contract which, in turn, would effectively constitute a property deprivation. Choice (B) is incorrect. General Electric may have been the subject of discrimination by the legislature, but General Electric is not a member of a suspect or quasi-suspect class. Only rational review will be applied, which city probably will pass with ease. Choice (C) is incorrect because the privileges and immunities clause of Article IV applies only to U.S. citizens. General Electric is not a U.S. citizen. Choice CD) is incorrect. In order to find a violation of the dormant commerce clause, there must be a showing that the local government has placed undue burdens on interstate commerce. In our example, there is no evidence to suggest that the city’s law protects utility companies like GE that reside in the city while burdening those outside the city.
173
Q
  1. A student at a private university was receiving financial aid from the university based on a financial aid application he completed at the time of his enrollment. During finals at the end of the fall semester, the student was caught cheating. His chemistry professor saw him looking at a fellow student’s exam paper and copying the answers to three multiple choice questions. The professor notified the honor committee, which immediately found an honor code violation and ordered that the student receive a failing grade in the course. In addition, in accordance with the rules of the university, the student’s financial aid was terminated.
    The student filed a lawsuit seeking reinstatement of his financial aid. The student’s only argument was that his due process rights were violated in that he did not receive a notice of, or a hearing on, the alleged honor code violation.
    Which of the following facts, if true, would be most helpful to the student?
    (A) The university was in financial difficulties and could not meet its payroll expenses.
    (B) The university did not re-allocate the student’s financial aid to another deserving student.
    (C) The university received support from the state.
    (D) The honor committee sent the student a letter to advise him of the hearing date.
A
  1. (C) Choice (C) is most accurate because it is the only alternative that is helpful to the student. The fact that the university received support from the state is evidence of the “joint contract” between the university and the government. Under the concept of state action, where the private actor and government can be said to be in a “symbiotic relationship,” the private actor will be subject to constitutional restraints. As a consequence, since the student did not receive notice or a hearing before his financial aid was terminated, the actions of the university (which would be treated as a government agent) were violative of the due process clause of the 14th Amendment. Choices (A) and (B) are incorrect because these facts would not carry legal relevance for our purposes. Choice (D) is incorrect because this would tend to hurt the student’s argument insofar as it demonstrates that the university is trying to afford the student due process.
174
Q
  1. A woman was employed as a state trooper. Although the state provides both sexes with equal pay and benefits, the state has adopted a policy that prohibits the assignment of female officers to its special undercover narcotics division. This is a moderate risk position that sometimes involves violent encounters with drug dealers. Since the special narcotics division was first established, five undercover agents have been killed in the line of duty. It is because of the state’s concern with the safety and well-being of its female officers that it has adopted such a policy.
    The woman, who desired to be a member of the narcotics division, filed an application for assignment as a special drug agent. After she was rejected for the position, the woman sued the state in federal court to enjoin enforcement of its stated policy on the grounds that it is unconstitutional.
    As a matter of constitutional law, which of the following results in this suit is most appropriate?
    (A) Judgment for the woman, because the facts asserted do not demonstrate that the particular classification contained in this policy is substantially related to the advancement of an important state interest.
    (B) Judgment for the woman, because the terms and conditions of state government employment are privileges or immunities of state citizenship that may not be abridged by the state on the basis of gender.
    (C) Judgment for the state, because it is within a state’s police power to insulate the terms and conditions of governmental employment.
    (D) Judgment for the state, because the state has articulated a rational basis for this classification and, therefore, a court may not substitute its judgment for that of responsible state officials.
A
  1. (A) As noted in the following chart, gender classifications are subject to a “middle-tier” standard of review. The burden of persuasion is on the government to demonstrate that the classification (or discrimination) is substantially related to an important governmental interest. Choice (A) correctly states the applicable standard of review for gender classifications. Choice (C) is incorrect to the extent that it relies on the decision in Foley v. Connellie, which allowed a state to exclude “aliens” from its police force. In Foley, the Supreme Court held that New York can prevent aliens from becoming state troopers. According to the Foley doctrine, a state could prevent aliens from holding certain state positions that involve a governmental function (e.g., public school teachers, police officers, and probation officers). Because this decision was specific to “aliens,” it wouLd be inapplicable to this question, which pertains to discrimination on the basis of gender classification, not alienage. Choice (B) is incorrect because the privileges or immunities clause of the 14th Amendment protects U.S. citizens from discrimination against out-of-staters, not discrimination based on gender. Choice (D) is incorrect. The reference to “rational basis” suggests that rational review is in order. But we have a gender classification, which wiLl trigger intermediate review [Craig v. Boren, 429 U.S. 190 (1976)].
175
Q
  1. An employee is an orthodox member of his faith. He has recently been hired as a forest ranger by the state. In accordance with the orthodox tradition of his religion, the employee wears a covering on his head throughout the day. The director of the state forestry department has notified the employee that he will not be permitted to wear his head covering while on duty. A state forestry regulation provides that all forest rangers are required to wear only standard headgear in the interests of maintaining a uniform dress code conducive to the furtherance of the department’s morale, efficiency, and professional standard of conduct. Although the employee wants to pursue a career as a forest ranger, he feels that his religious beliefs should not be compromised by a governmental agency.
    In trying to resolve his dilemma, the employee seeks your legal advice. You should advise him that in light of relevant U.S. Supreme Court decisions, the state regulation is probably
    (A) constitutional, because although the employee has a constitutional right to the free exercise of his religion, a state may impose reasonable regulations that are rationally related to the furtherance of a state interest.
    (B) constitutional, because the interest of the state in vindicating a carefully considered professional judgment by the forestry director that wearing such religious apparel would interfere with the department’s morale and efficiency is sufficient to contravene the wishes of its members to wear headgear required by their religious beliefs.
    (C) unconstitutional, because in accordance with the free exercise clause of the First Amendment, a state has no power to regulate religious beliefs.
    (D) unconstitutional, because an individual has a fundamental right to seek gainful employment, and a state cannot arbitrarily and unreasonably regulate against such economic activity.
A
  1. (B) In Goldman v. Weinberger, 106 S.Ct. 1310 (1986), the Court ruled against the free exercise claim of an Orthodox Jewish Air Force captain to wear a yarmulke while on duty. Giving broad deference to the professional judgment of the military authorities in determining the need to place restrictions on certain aspects of religiously motivated conduct, the Court stated that the use of standardized uniforms “encourages the subordination of personal preferences and identifies in favor of the overall group mission.” Choice (B) is the best answer, since it closely approximates the specific language used by the Court in the Goldman case. While refusal by the military to accommodate an individual’s free exercise claim is subject only to rational basis standard review, based on the Court’s broad deference, the strict scrutiny standard applies to state regulation of free exercise claims. Therefore, choice (A) is incorrect. Choice (C), a true statement, is incorrect because the wearing of a head covering is a form of religious conduct, not a religious belief. The procedural due process argument in choice (D) is incorrect because the state regulation in question directly affects the employee’s free exercise of religion and is not depriving him of his right to seek gainful employment.